Header Graphic
Apps for iPad

FAA Glossaries

Touring Machine Company

Safety Pilot may log time as SIC or PIC

April 7th, 2019

There are two legal interpretations that allow you to log SIC or PIC time when acting as a safety pilot. The first, Beaty (2013), addressed logging cross-country time for the safety pilot, but explicitly says that the safety pilot can log the time as PIC if they are acting as PIC.

In your first scenario, Pilot A and Pilot B, who hold private pilot certificates and ratings appropriate to the aircraft, take a flight. Pilot A acts as the pilot-in-command (PIC). During a portion of the flight, Pilot B acts as the safety pilot and second-in-command (SIC) while Pilot A operates in simulated instrument flight. You ask whether Pilot B may log SIC and/or cross-country time for the portion of the flight during which Pilot B acts as safety pilot.

Section 61.51 (f) governs the logging of SIC time and states, in relevant part, that a person may log SIC time only for that flight time during which that person holds the appropriate ratings for aircraft being flown and “more than one pilot is required under the type certification of the aircraft or the regulations under which the flight is being conducted.” When a pilot is operating an aircraft in simulated instrument flight, 14 C.F.R. § 91.109(b), in relevant part, requires that a safety pilot, who possesses at least a private pilot certificate with category and class ratings appropriate to the aircraft, occupy the other control seat. Accordingly, Pilot B may log SIC time for the portion of the flight during which Pilot B acts as safety pilot because Pilot B was a required flight crewmember for that portion of the flight under § 91.1 09(b). The FAA previously has interpreted that a person acting as safety pilot for a portion of a flight may not log cross-country time because that person is not a required flight crewmember for the entire flight. See Legal Interpretation to Jeff Gebhart (June 22,2009) (copy enclosed). Accordingly, Pilot B may not log cross-country time for any portion of the flight.

Gebhart (2009) also says you may log the time as safety pilot as PIC.

Section 61.51(e) governs the logging of PIC time and states, in relevant part, that a sport, recreational, private, or commercial pilot may log PIC time for the time during which that pilot is “the sole manipulator of the controls of an aircraft for which the pilot is rated or has privileges” or “acting as pilot in command of an aircraft on which more than one pilot is required under … the regulations under which the flight is conducted.”

Hicks (1993) is even more explicit about logging time as SIC, so this is not a new interpretation.

Section 61.51(e) governs the logging of PIC time and states, in relevant part, that a sport, recreational, private, or commercial pilot may log PIC time for the time during which that pilot is “the sole manipulator of the controls of an aircraft for which the pilot is rated or has privileges” or “acting as pilot in command of an aircraft on which more than one pilot is required under … the regulations under which the flight is conducted.”

Responding specifically to your inquiry, the pilot that is under the hood may log PIC time for that time in which he is the sole manipulator of the controls of the aircraft, provided that he or she is rated for that aircraft. The appropriately rated safety pilot may concurrently log as second-in-command (SIC) that time during which he or she is acting as safety pilot.

However, the two pilots may, prior to initiating the flight, agree that the safety pilot will be the PIC responsible for the operation and safety of the aircraft during the flight. If this is done, then the safety pilot may log all the flight time as PIC time in accordance with FAR 1.1 and the pilot under the hood may log, concurrently, all of the flight time during which he is the sole manipulator of the controls as PIC time in accordance with FAR 61.51(c)(2)(i). In order to assist you further in this regard, enclosed please find a prior FAA interpretation concerning the logging of flight time under simulated instrument flight conditions.

CFR §61.127 (b) (x) Flight proficiency. High-altitude operations;

April 5th, 2019

There are several areas that you need to consider when operating at high altitudes. Most of my flying is done at altitudes around 5,000′ so I don’t really worry about the high altitude stuff, but if you are flying commercially you will probably be flying more capable airplanes and trying to minimize flight time. That means flying where oxygen is required and potentially above 18,000′ where flight rules are different. There are five major things to consider for high altitude operations: physiology, training, regulations, aircraft systems, and aircraft performance.

Physiology

The Pilots Handbook of Aeronautical Knowledge does a good job of covering this area.

Symptoms of Hypoxia
High-altitude flying can place a pilot in danger of becoming hypoxic. Oxygen starvation causes the brain and other vital organs to become impaired. The first symptoms of hypoxia can include euphoria and a carefree feeling. With increased oxygen starvation, the extremities become less responsive and flying becomes less coordinated. The symptoms of hypoxia vary with the individual, but common symptoms include:

• Cyanosis (blue fingernails and lips)
• Headache
• Decreased response to stimuli and increased reaction time
• Impaired judgment
• Euphoria
• Visual impairment
• Drowsiness
• Lightheaded or dizzy sensation
• Tingling in fingers and toes
• Numbness

As hypoxia worsens, the field of vision begins to narrow and instrument interpretation can become difficult. Even with all these symptoms, the effects of hypoxia can cause a pilot to have a false sense of security and be deceived into believing everything is normal.

Treatment of Hypoxia
Treatment for hypoxia includes flying at lower altitudes and/or using supplemental oxygen. All pilots are susceptible to the effects of oxygen starvation, regardless of physical endurance or acclimatization. When flying at high altitudes, it is paramount that oxygen be used to avoid the effects of hypoxia. The term “time of useful consciousness” describes the maximum time the pilot has to make rational, life-saving decisions and carry them out at a given altitude without supplemental oxygen. As altitude increases above 10,000 feet, the symptoms of hypoxia increase in severity, and the time of useful consciousness rapidly decreases. Since symptoms of hypoxia can be different for each individual, the ability to recognize hypoxia can be greatly improved by experiencing and witnessing the effects of it during an altitude chamber “flight.”

Altitude-Induced Decompression Sickness (DCS)
Decompression sickness (DCS) describes a condition characterized by a variety of symptoms resulting from exposure to low barometric pressures that cause inert gases (mainly nitrogen), normally dissolved in body fluids and tissues, to come out of physical solution and form bubbles. Nitrogen is an inert gas normally stored throughout the human body (tissues and fluids) in physical solution. When the body is exposed to decreased barometric pressures (as in flying an unpressurized aircraft to altitude or during a rapid decompression), the nitrogen dissolved in the body comes out of solution. If the nitrogen is forced to leave the solution too rapidly, bubbles form in different areas of the body causing a variety of signs and symptoms. The most common symptom is joint pain, which is known as “the bends.”

What to do when altitude-induced DCS occurs:
• Put on oxygen mask immediately and switch the regulator to 100 percent oxygen.
• Begin an emergency descent and land as soon as possible. Even if the symptoms disappear during descent, land and seek medical evaluation while continuing to breathe oxygen.
• If one of the symptoms is joint pain, keep the affected area still; do not try to work pain out by moving the joint around.
• Upon landing, seek medical assistance from an FAA medical officer, AME, military flight surgeon, or a hyperbaric medicine specialist. Be aware that a physician not specialized in aviation or hypobaric medicine may not be familiar with this type of medical problem.
• Definitive medical treatment may involve the use of a hyperbaric chamber operated by specially-trained personnel.
• Delayed signs and symptoms of altitude-induced DCS can occur after return to ground level regardless of presence during flight.

Training

Pressurized Aircraft
§61.31 Type rating requirements, additional training, and authorization requirements.
(g) Additional training required for operating pressurized aircraft capable of operating at high altitudes. (1) Except as provided in paragraph (g)(3) of this section, no person may act as pilot in command of a pressurized aircraft (an aircraft that has a service ceiling or maximum operating altitude, whichever is lower, above 25,000 feet MSL), unless that person has received and logged ground training from an authorized instructor and obtained an endorsement in the person’s logbook or training record from an authorized instructor who certifies the person has satisfactorily accomplished the ground training. The ground training must include at least the following subjects:

(i) High-altitude aerodynamics and meteorology;
(ii) Respiration;
(iii) Effects, symptoms, and causes of hypoxia and any other high-altitude sickness;
(iv) Duration of consciousness without supplemental oxygen;
(v) Effects of prolonged usage of supplemental oxygen;
(vi) Causes and effects of gas expansion and gas bubble formation;
(vii) Preventive measures for eliminating gas expansion, gas bubble formation, and high-altitude sickness;
(viii) Physical phenomena and incidents of decompression; and
(ix) Any other physiological aspects of high-altitude flight.

(2) Except as provided in paragraph (g)(3) of this section, no person may act as pilot in command of a pressurized aircraft unless that person has received and logged training from an authorized instructor in a pressurized aircraft, or in a full flight simulator or flight training device that is representative of a pressurized aircraft, and obtained an endorsement in the person’s logbook or training record from an authorized instructor who found the person proficient in the operation of a pressurized aircraft. The flight training must include at least the following subjects:

(i) Normal cruise flight operations while operating above 25,000 feet MSL;
(ii) Proper emergency procedures for simulated rapid decompression without actually depressurizing the aircraft; and
(iii) Emergency descent procedures.

RVSM
You can read the details in the FARs and AIM but basically in order to operate in airspace above FL290 the airplane and the pilot need to be certified for RVSM operations.

Part 91 Appendix G to Part 91—Operations in Reduced Vertical Separation Minimum (RVSM) Airspace
Section 1. Definitions
Reduced Vertical Separation Minimum (RVSM) Airspace. Within RVSM airspace, air traffic control (ATC) separates aircraft by a minimum of 1,000 feet vertically between FL 290 and FL 410 inclusive. Air-traffic control notifies operators of RVSM airspace by providing route planning information.

AIM Section 6. Operational Policy/Procedures for Reduced Vertical Separation Minimum (RVSM) in the Domestic U.S., Alaska, Offshore Airspace and the San Juan FIR

Regulations

There are several FARs dealing with high-altitude operations and some of them overlap with items mentioned previously. You are probably familiar with the oxygen requirements above 12,500′ but may not know that there are additional requirements at higher altitudes for pressurized aircraft. From your private pilot training (and every Flight Review since then) you also know that a flight plan and IFR rating are required for flight in Class A airspace (airspace from 18,000′ MSL up to and including FL 600) and a transponder is required above 10,000′ MSL. After January 1, 2020 ADS-B is required in all the airspace where transponders are currently required. In Class A airspace only 1090 ES is allowed.

§91.135 Operations in Class A airspace.
Except as provided in paragraph (d) of this section, each person operating an aircraft in Class A airspace must conduct that operation under instrument flight rules (IFR) and in compliance with the following:

(a) Clearance. Operations may be conducted only under an ATC clearance received prior to entering the airspace.

(b) Communications. Unless otherwise authorized by ATC, each aircraft operating in Class A airspace must be equipped with a two-way radio capable of communicating with ATC on a frequency assigned by ATC. Each pilot must maintain two-way radio communications with ATC while operating in Class A airspace.

(c) Equipment requirements. Unless otherwise authorized by ATC, no person may operate an aircraft within Class A airspace unless that aircraft is equipped with the applicable equipment specified in §91.215, and after January 1, 2020, §91.225.

(d) ATC authorizations. An operator may deviate from any provision of this section under the provisions of an ATC authorization issued by the ATC facility having jurisdiction of the airspace concerned. In the case of an inoperative transponder, ATC may immediately approve an operation within a Class A airspace area allowing flight to continue, if desired, to the airport of ultimate destination, including any intermediate stops, or to proceed to a place where suitable repairs can be made, or both. Requests for deviation from any provision of this section must be submitted in writing, at least 4 days before the proposed operation. ATC may authorize a deviation on a continuing basis or for an individual flight.

§91.211 Supplemental oxygen.
(a) General. No person may operate a civil aircraft of U.S. registry—
(1) At cabin pressure altitudes above 12,500 feet (MSL) up to and including 14,000 feet (MSL) unless the required minimum flight crew is provided with and uses supplemental oxygen for that part of the flight at those altitudes that is of more than 30 minutes duration;

(2) At cabin pressure altitudes above 14,000 feet (MSL) unless the required minimum flight crew is provided with and uses supplemental oxygen during the entire flight time at those altitudes; and

(3) At cabin pressure altitudes above 15,000 feet (MSL) unless each occupant of the aircraft is provided with supplemental oxygen.

(b) Pressurized cabin aircraft. (1) No person may operate a civil aircraft of U.S. registry with a pressurized cabin—
(i) At flight altitudes above flight level 250 unless at least a 10-minute supply of supplemental oxygen, in addition to any oxygen required to satisfy paragraph (a) of this section, is available for each occupant of the aircraft for use in the event that a descent is necessitated by loss of cabin pressurization; and

(ii) At flight altitudes above flight level 350 unless one pilot at the controls of the airplane is wearing and using an oxygen mask that is secured and sealed and that either supplies oxygen at all times or automatically supplies oxygen whenever the cabin pressure altitude of the airplane exceeds 14,000 feet (MSL), except that the one pilot need not wear and use an oxygen mask while at or below flight level 410 if there are two pilots at the controls and each pilot has a quick-donning type of oxygen mask that can be placed on the face with one hand from the ready position within 5 seconds, supplying oxygen and properly secured and sealed.

(2) Notwithstanding paragraph (b)(1)(ii) of this section, if for any reason at any time it is necessary for one pilot to leave the controls of the aircraft when operating at flight altitudes above flight level 350, the remaining pilot at the controls shall put on and use an oxygen mask until the other pilot has returned to that crewmember’s station.

§91.215 ATC transponder and altitude reporting equipment and use.
(i) In all airspace of the 48 contiguous states and the District of Columbia at and above 10,000 feet MSL, excluding the airspace at and below 2,500 feet above the surface;

§91.225 Automatic Dependent Surveillance-Broadcast (ADS-B) Out equipment and use.
(a) After January 1, 2020, and unless otherwise authorized by ATC, no person may operate an aircraft in Class A airspace unless the aircraft has equipment installed that—

(1) Meets the performance requirements in TSO-C166b, Extended Squitter Automatic Dependent Surveillance-Broadcast (ADS-B) and Traffic Information Service-Broadcast (TIS-B) Equipment Operating on the Radio Frequency of 1090 Megahertz (MHz); and

RVSM was discussed in the training section.

DME is required above FL240 although that requirement is probably not applicable on most Part 91 aircraft since GPS can be used instead.

§91.205 Powered civil aircraft with standard category U.S. airworthiness certificates: Instrument and equipment requirements.
(e) Flight at and above 24,000 feet MSL (FL 240). If VOR navigation equipment is required under paragraph (d)(2) of this section, no person may operate a U.S.-registered civil aircraft within the 50 states and the District of Columbia at or above FL 240 unless that aircraft is equipped with approved DME or a suitable RNAV system. When the DME or RNAV system required by this paragraph fails at and above FL 240, the pilot in command of the aircraft must notify ATC immediately, and then may continue operations at and above FL 240 to the next airport of intended landing where repairs or replacement of the equipment can be made.

Aircraft Systems

Normally aspirated engines lose power as you climb because the air is thinner. To solve this problem engineers have come up with several ways to pressurize the air so that the engine gets the same amount of oxygen as it would at sea level. The most common ways for GA aircraft are tubocharging and turbo-normalization where exhaust gases drive a turbine that compresses the air. Military planes in WWI used supercharging where the engine drives a pump to compress the air.

Chapter 11 of the Airplane Flying Handbook explains the details fairly well.

Turbocharging
The turbocharged engine allows the pilot to maintain sufficient cruise power at high altitudes where there is less drag, which means faster true airspeeds and increased range with fuel economy. At the same time, the powerplant has flexibility and can be flown at a low altitude without the increased fuel consumption of a turbine engine. When attached to the standard powerplant, the turbocharger does not take any horsepower from the engine to operate; it is relatively simple mechanically, and some models can pressurize the cabin as well.

The turbocharger is an exhaust-driven device that raises the pressure and density of the induction air delivered to the engine. It consists of two separate components: a compressor and a turbine connected by a common shaft. The compressor supplies pressurized air to the engine for high-altitude operation. The compressor and its housing are between the ambient air intake and the induction air manifold. The turbine and its housing are part of the exhaust system and utilize the flow of exhaust gases to drive the compressor. [Figure 11-9]
The turbine has the capability of producing manifold pressure in excess of the maximum allowable for the particular engine. In order not to exceed the maximum allowable manifold pressure, a bypass or waste gate is used so that some of the exhaust is diverted overboard before it passes through the turbine.

The position of the waste gate regulates the output of the turbine and therefore, the compressed air available to the engine. When the waste gate is closed, all of the exhaust gases pass through and drive the turbine. As the waste gate opens, some of the exhaust gases are routed around the turbine through the exhaust bypass and overboard through the exhaust pipe.

The waste gate actuator is a spring-loaded piston operated by engine oil pressure. The actuator, which adjusts the waste gate position, is connected to the waste gate by a mechanical linkage.

The control center of the turbocharger system is the pressure controller. This device simplifies turbocharging to one control: the throttle. Once the desired manifold pressure is set, virtually no throttle adjustment is required with changes in altitude. The controller senses compressor discharge requirements for various altitudes and controls the oil pressure to the waste gate actuator, which adjusts the waste gate accordingly. Thus the turbocharger maintains only the manifold pressure called for by the throttle setting.

Ground Boosting Versus Altitude Turbocharging
Altitude turbocharging (sometimes called “normalizing”) is accomplished by using a turbocharger that maintains maximum allowable sea level manifold pressure (normally 29–30 “Hg) up to a certain altitude. This altitude is specified by the airplane manufacturer and is referred to as the airplane’s critical altitude. Above the critical altitude, the manifold pressure decreases as additional altitude is gained. Ground boosting, on the other hand, is an application of turbocharging where more than the standard 29 inches of manifold pressure is used in flight. In various airplanes using ground boosting, takeoff manifold pressures may go as high as 45 “Hg.

Aircraft designed for high-altitude flying have additional systems that the pilot needs to become familiar with as well.

Environmental systems.
In an aircraft, the systems, including the supplemental oxygen systems, air conditioning systems, heaters, and pressurization systems, which make it possible for an occupant to function at high altitude.

High-performance airplanes are usually certified for flight into known icing conditions. These aircraft are equipped to handle limited amounts of ice.

Anti-Icing/Deicing
Anti-icing/deicing equipment is frequently installed on multiengine airplanes [and turbo-props] and consists of a combination of different systems. These may be classified as either anti- icing or deicing, depending upon function. The presence of anti-icing and deicing equipment, even though it may appear elaborate and complete, does not necessarily mean that the airplane is approved for flight in icing conditions. The AFM/POH, placards, and even the manufacturer should be consulted for specific determination of approvals and limitations. Anti-icing equipment is provided to prevent ice from forming on certain protected surfaces. Anti-icing equipment includes heated pitot tubes, heated or non- icing static ports and fuel vents, propeller blades with electrothermal boots or alcohol slingers, windshields with alcohol spray or electrical resistance heating, windshield defoggers, and heated stall warning lift detectors. On many turboprop engines, the “lip” surrounding the air intake is heated either electrically or with bleed air. In the absence of AFM/POH guidance to the contrary, anti-icing equipment should be actuated prior to flight into known or suspected icing conditions.

Deicing equipment is generally limited to pneumatic boots on wing and tail leading edges. Deicing equipment is installed to remove ice that has already formed on protected surfaces. Upon pilot actuation, the boots inflate with air from the pneumatic pumps to break off accumulated ice. After a few seconds of inflation, they are deflated back to their normal position with the assistance of a vacuum. The pilot monitors the buildup of ice and cycles the boots as directed in the AFM/ POH. An ice light on the left engine nacelle allows the pilot to monitor wing ice accumulation at night.

Aircraft Performance

I already covered reduced engine performance in the previous section, but there are some other ways in which flying in thinner air affects aircraft performance.

As explained in The Pilots Handbook of Aeronautical Knowledge does a good job of covering this area V speeds vary with altitude. Refer to the AFM for your aircraft for specific values.

VY—the speed at which the aircraft obtains the maximum increase in altitude per unit of time. This best ROC speed normally decreases slightly with altitude.

VX—the speed at which the aircraft obtains the highest altitude in a given horizontal distance. This best AOC speed normally increases slightly with altitude.

Mach Speed
High-speed airplanes designed for subsonic flight are limited to some Mach number below the speed of sound to avoid the formation of shock waves that begin to develop as the airplane nears Mach 1.0. These shock waves (and the adverse effects associated with them) can occur when the airplane speed is substantially below Mach 1.0. The Mach speed at which some portion of the airflow over the wing first equals Mach 1.0 is termed the critical Mach number (Mcr). This is also the speed at which a shock wave first appears on the airplane.

There is no particular problem associated with the acceleration of the airflow up to Mach Crit, the point where Mach 1.0 is encountered; however, a shock wave is formed at the point where the airflow suddenly returns to subsonic flow. This shock wave becomes more severe and moves aft on the wing as speed of the wing is increased and eventually flow separation occurs behind the well-developed shock wave.

If allowed to progress well beyond the MMO for the airplane, this separation of air behind the shock wave can result in severe buffeting and possible loss of control or “upset.”

Commercial Pilot ACS VIII. High Altitude Operations

Most of the items in this section are covered above, however there are a couple of things not covered that ar explained in The Pilots Handbook of Aeronautical Knowledge.

Task A. Supplemental Oxygen
c. Time of useful consciousness (TUC)

All pilots are susceptible to the effects of oxygen starvation, regardless of physical endurance or acclimatization. When flying at high altitudes, it is paramount that oxygen be used to avoid the effects of hypoxia. The term “time of useful consciousness” describes the maximum time the pilot has to make rational, life-saving decisions and carry them out at a given altitude without supplemental oxygen. As altitude increases above 10,000 feet, the symptoms of hypoxia increase in severity, and the time of useful consciousness rapidly decreases.

Time of Useful Consciousness

Demonstrating how to use the oxygen system or pressurization in the practical test only apply if you provide an airplane with oxygen or pressurization. However, you are still responsible for the knowing how they work.

a. Characteristics, limitations, and applicability of continuous flow, demand, and pressure-demand oxygen systems
Diluter-Demand Oxygen Systems
Diluter-demand oxygen systems supply oxygen only when the user inhales through the mask. An automix lever allows the regulators to automatically mix cabin air and oxygen or supply 100 percent oxygen, depending on the altitude. The demand mask provides a tight seal over the face to prevent dilution with outside air and can be used safely up to 40,000 feet. A pilot who has a beard or mustache should be sure it is trimmed in a manner that will not interfere with the sealing of the oxygen mask. The fit of the mask around the beard or mustache should be checked on the ground for proper sealing.

Pressure-Demand Oxygen Systems
Pressure-demand oxygen systems are similar to diluter demand oxygen equipment, except that oxygen is supplied to the mask under pressure at cabin altitudes above 34,000 feet. Pressure-demand regulators create airtight and oxygen-tight seals, but they also provide a positive pressure application of oxygen to the mask face piece that allows the user’s lungs to be pressurized with oxygen. This feature makes pressure demand regulators safe at altitudes above 40,000 feet. Some systems may have a pressure demand mask with the regulator attached directly to the mask, rather than mounted on the instrument panel or other area within the flight deck. The mask-mounted regulator eliminates the problem of a long hose that must be purged of air before 100 percent oxygen begins flowing into the mask.

Continuous-Flow Oxygen System
Continuous-flow oxygen systems are usually provided for passengers. The passenger mask typically has a reservoir bag that collects oxygen from the continuous-flow oxygen system during the time when the mask user is exhaling. The oxygen collected in the reservoir bag allows a higher aspiratory flow rate during the inhalation cycle, which reduces the amount of air dilution. Ambient air is added to the supplied oxygen during inhalation after the reservoir bag oxygen supply is depleted. The exhaled air is released to the cabin.

Electrical Pulse-Demand Oxygen System
Portable electrical pulse-demand oxygen systems deliver oxygen by detecting an individual’s inhalation effort and provide oxygen flow during the initial portion of inhalation. Pulse demand systems do not waste oxygen during the breathing cycle because oxygen is only delivered during inhalation. Compared to continuous-flow systems, the pulse- demand method of oxygen delivery can reduce the amount of oxygen needed by 50–85 percent. Most pulse-demand oxygen systems also incorporate an internal barometer that automatically compensates for changes in altitude by increasing the amount of oxygen delivered for each pulse as altitude is increased.

b. Differences between and identification of “aviator’s breathing oxygen” and other types of oxygen
High pressure oxygen containers should be marked with the psi tolerance (i.e., 1,800 psi) before filling the container to that pressure. The containers should be supplied with oxygen that meets or exceeds SAE AS8010 (as revised), Aviator’s Breathing Oxygen Purity Standard. To assure safety, periodic inspection and servicing of the oxygen system should be performed.

c. Necessary precautions when using supplemental oxygen systems
Certain precautions should be observed whenever aircraft oxygen systems are to be serviced. Oxygen system servicing should be accomplished only when the aircraft is located outside of the hangars. Personal cleanliness and good housekeeping are imperative when working with oxygen. Oxygen under pressure creates spontaneous results when brought in contact with petroleum products. Service people should be certain to wash dirt, oil, and grease (including lip salves and hair oil) from their hands before working around oxygen equipment. It is also essential that clothing and tools are free of oil, grease, and dirt. Aircraft with permanently installed oxygen tanks usually require two persons to accomplish servicing of the system. One should be stationed at the service equipment control valves, and the other stationed where he or she can observe the aircraft system pressure gauges. Oxygen system servicing is not recommended during aircraft fueling operations or while other work is performed that could provide a source of ignition. Oxygen system servicing while passengers are on board the aircraft is not recommended.

O2 gear: Care and feeding from AOPA.

  • Store portable oxygen tanks securely so they can’t fall over or become a projectile. This could damage the regulator and set the stage for cracks.
  • Don’t keep portable oxygen tanks in hot, enclosed areas, such as the inside of an airplane on a hot day, or in the trunk of a car. Compressed gases can expand, causing dangerous pressure rises and the chance of a tank explosion.
  • Keep oxygen equipment clean. Dirt particles can contaminate regulators and valves and create sparks at altitude, where the ambient air is dry.
  • Store masks and cannulas in their containers, and out of the sun.
  • Have your tank inspected every five years, as per FAA or manufacturer rules. This includes a hydrostatic test to check the tank’s strength and integrity, just like the tests administered to scuba tanks.
  • Allow no smoking around oxygen equipment Oxygen burns robustly!.
  • Use no petroleum-based lip balms, lipstick, sun block, or makeup when using oxygen. In the presence of oxygen, these products can burn.
  • Make sure your mask and regulator connectors are of a compatible design. In order to have leak-free connections, all components must be compatible, and a mask connector that works with one regulator may not properly fit another.
  • When having your tank filled, make sure it’s filled slowly.

Task B. Pressurization
Fundamental concepts of airplane pressurization system, to include failure modes.

Chapter 7 of The Pilots Handbook of Aeronautical Knowledge covers this in detail. Several points of note are:

In a typical pressurization system, the cabin, flight compartment, and baggage compartments are incorporated into a sealed unit capable of containing air under a pressure higher than outside atmospheric pressure. On aircraft powered by turbine engines, bleed air from the engine compressor section is used to pressurize the cabin. Superchargers may be used on older model turbine-powered aircraft to pump air into the sealed fuselage. Piston-powered aircraft may use air supplied from each engine turbocharger through a sonic venturi (flow limiter). Air is released from the fuselage by a device called an outflow valve. By regulating the air exit, the outflow valve allows for a constant inflow of air to the pressurized area.

A cabin pressurization system typically maintains a cabin pressure altitude of approximately 8,000 feet at the maximum designed cruising altitude of an aircraft. This prevents rapid changes of cabin altitude that may be uncomfortable or cause injury to passengers and crew.

The cabin air pressure safety valve is a combination pressure relief, vacuum relief, and dump valve. The pressure relief valve prevents cabin pressure from exceeding a predetermined differential pressure above ambient pressure. The vacuum relief prevents ambient pressure from exceeding cabin pressure by allowing external air to enter the cabin when ambient pressure exceeds cabin pressure.

Decompression
During an explosive decompression, there may be noise, and one may feel dazed for a moment. The cabin air fills with fog, dust, or flying debris. Fog occurs due to the rapid drop in temperature and the change of relative humidity. Normally, the ears clear automatically. Air rushes from the mouth and nose due to the escape of air from the lungs and may be noticed by some individuals.

Rapid decompression decreases the period of useful consciousness because oxygen in the lungs is exhaled rapidly, reducing pressure on the body. This decreases the partial pressure of oxygen in the blood and reduces the pilot’s effective performance time by one-third to one-fourth its normal time. For this reason, an oxygen mask should be worn when flying at very high altitudes (35,000 feet or higher). It is recommended that the crewmembers select the 100 percent oxygen setting on the oxygen regulator at high altitude if the aircraft is equipped with a demand or pressure demand oxygen system.

CFR §61.127 (b) (xi) Flight proficiency. Postflight

April 5th, 2019

Commercial applicants are required to log ground or flight training in 11 areas. One of them involves post-flight.

(b) Areas of operation.
(1) For an airplane category rating with a single-engine class rating:
(xi) Postflight procedures.

This is another area that you have been doing since your first flight and Chapter 2 of the Airplane Flying Handbook tells you exactly what is required by this part. So all you have to do is go over it with your CFI and log the ground time.

If you have been doing touch-and-goes you may have gotten into bad habits with flaps, trim, and cowl. The preferred procedure is to wait until you are clear of the runway before you mess with anything. This is especially important with tailwheel planes.

Clear of Runway and Stopped
Because of different configurations and equipment in various airplanes, the after-landing checklist within the AFM/POH must be used. Some of the items may include:

• Power—set to the AFM/POH values such as throttle 1,000 rpm, propeller full forward, mixture leaned.
• Fuel—may require switching tanks and fuel pumps switched off.
• Flaps—set to the retracted position.
• Cowl flaps—may be opened or closed depending on temperature conditions.
• Trim—reset to neutral or takeoff position.
• Lights—may be switched off if not needed, such as strobe lights.
• Avionics—may be switched off or to standby, such as the transponder and frequencies changed to contact ground control or Common Traffic Advisory Frequency (CTAF), as required.

I use BC-FLAGS: Boost Pump Off, Cowl Flaps Open, Flaps Up, Lean Aggressively, Contact Ground, and get some Air into the cockpit.

From the Airplane Flying Handbook:

Post-Flight
A flight is not complete until the engine is shut down and the airplane is secured. A pilot should consider this an essential part of any flight.

Securing and Servicing
After engine shutdown and deplaning passengers, the pilot should accomplish a post-flight inspection. This includes a walk around to inspect the general condition of the aircraft. Inspect near and around the cowling for signs of oil or fuel streaks and around the oil breather for excessive oil discharge. Inspect under wings and other fuel tank locations for fuel stains. Inspect landing gear and tires for damage and brakes for any leaking hydraulic fluid. Inspect cowling inlets for obstructions.

Oil levels should be checked and quantities brought to AFM/ POH levels. Fuel should be added based on the immediate use of the airplane. If the airplane is going to be inactive, it is a good operating practice to fill the fuel tanks to prevent water condensation from forming inside the tank. If another flight is planned, the fuel tanks should be filled based on the flight planning requirements for that flight.

The aircraft should be hangared or tied down, flight controls secured, and security locks in place. The type of tie downs may vary significantly from chains to well-worn ropes. Chains are not flexible and as such should not be made taught as to allow the airplane some movement and prevent airframe structural damage. Tie down ropes are flexible and may be reasonably cinched to the airplane’s tie down rings. Consider utilizing pitot tube covers, cowling inlet covers, rudder gust locks, window sunscreens, and propeller security locks to further enhance the safety and security of the airplane.

Hangaring is not without hazards to the airplane. The pilot should ensure that enough space is allocated to the airplane so it is free from any impact to the hangar, another aircraft, or vehicle. The airplane should be inspected after hangaring to ensure that no damage was imparted on the airplane.

Pilots Handbook of Aeronautical Knowledge Chapter 2:
Another important tool—overlooked by many pilots— is a good post­flight analysis. When you have safely secured the airplane, take the time to review and analyze the flight as objectively as you can. Mistakes and judgment errors are inevitable; the most important thing is for you to recognize, analyze, and learn from them before your next flight.

I add a few items to my post-flight procedures. In planes with low-compression engines, I always do a runup before shutting down. That way if I didn’t lean enough on the flight I can clean the plugs before the next flight—rather than finding out in the runup area. I also refuel so I am ready to go for the next flight. I have a can of dollar store furniture polish that I use to get the bugs off of the wings and cowl.

CFR §61.127 (b) (i) and (ii) Flight proficiency. Preflight

April 5th, 2019

Commercial applicants are required to log ground or flight training in 11 areas. Two of them involve procedures prior to beginning the flight.

(b) Areas of operation.
(1) For an airplane category rating with a single-engine class rating:
(i) Preflight preparation;
(ii) Preflight procedures;

From the Pilots Handbook of Aeronautical Knowledge Chapter 2:
Preflight your passengers by preparing them for the possibility of delay and diversion, and involve them in your evaluation process.

From your private pilot training you are probably familiar with two pneumonics.

IMSAFE Checklist
1. Illness—Am I sick? Illness is an obvious pilot risk.
2. Medication—Am I taking any medicines that might affect my judgment or make me drowsy?
3. Stress—Stress causes concentration and performance problems. While the regulations list medical conditions that require grounding, stress is not among them.
4. Alcohol—Have I been drinking within 8 hours? Within 24 hours?
5. Fatigue—Am I tired and not adequately rested? Fatigue continues to be one of the most insidious hazards to flight safety, as it may not be apparent to a pilot until serious errors are made.
6. Emotions-A pilot who experiences an emotionally upsetting event should refrain from flying until the pilot has satisfactorily recovered.

The PAVE Checklist
Pilot in command (PIC), Aircraft, enVironment, and External pressures

Fortunately Chapter 2 of the Airplane Flying Handbook tells you exactly what is required by the aircraft part of PAVE. So all you have to do is go over it with your CFI and log the ground time.

Preflight Assessment of the Aircraft
The owner/operator is primarily responsible for maintenance, but the pilot is (solely) responsible for determining the airworthiness (and/or safety) of the airplane for flight.

• Annual inspection within the preceding 12-calendar months (Title 14 of the Code of Federal Regulations (14 CFR) part 91, section 91.409(a))
• 100-hour inspection, if the aircraft is operated for hire (14 CFR part 91, section 91.409(b))
• Transponder certification within the preceding 24-calendar months (14 CFR part 91, section 91.413)
• Static system and encoder certification, within the preceding 24-calendar months, required for instrument flight rules (IFR) flight in controlled airspace (14 CFR part 91, section 91.411)
• 30-day VHF omnidirectional range (VOR) equipment check required for IFR flight (14 CFR part 91, section 91.171)
• Emergency locator transmitter (ELT) inspection within the last 12 months (14 CFR part 91, section 91.207(d))
• ELT battery due (14 CFR part 91, section 91.207(c))
• Current status of life limited parts per Type Certificate Data Sheets (TCDS) (14 CFR part 91, section 91.417)
• Status, compliance, logbook entries for airworthiness directives (ADs) (14 CFR part 91, section 91.417(a) (2)(v))
• Federal Aviation Administration (FAA) Form 337, Major Repair or Alteration (14 CFR part 91, section 91.417)
• Inoperative equipment (14 CFR part 91, section 91.213)

A couple of things to note. Your ELT inspection is done at each annual, but the battery replacement time can get out of sync with the annual. There aren’t many life-limited parts on GA aircraft—that’s something you get into with turbines and jets. There are some hoses that are limited to 7 years but they should be caught at annual. Some ADs require replacement of parts, e.g. air filters, but again they are caught at annual and are not what is normally referred to as life-limited parts. Part 43 has the definition.

If your aircraft is not used for hire, then you don’t need a 100-hour inspection, but there are lots of 100-hour ADs out there, so be sure to check them.

CFR §43.10 Disposition of life-limited aircraft parts.
(a) Definitions used in this section. For the purposes of this section the following definitions apply.

Life-limited part means any part for which a mandatory replacement limit is specified in the type design, the Instructions for Continued Airworthiness, or the maintenance manual.

Life status means the accumulated cycles, hours, or any other mandatory replacement limit of a life-limited part.

Once you have gone over the books, it is time to look at the required documents. These are the familiar ARROW documents and two more that should be there.

It must be determined by the pilot that the following documents are, as appropriate, on board, attached, or affixed to the airplane:

• Original Airworthiness Certificate (14 CFR part 91, section 91.203)
• Original Registration Certificate (14 CFR part 91, section 91.203)
• Radio station license for flights outside the United States or airplanes greater than 12,500 pounds (Federal Communications Commission (FCC) rule)
• Operating limitations, which may be in the form of an FAA-approved AFM/POH, placards, instrument markings, or any combination thereof (14 CFR part 91, section 91.9)
• Official weight and balance
• Compass deviation card (14 CFR part 23, section 23.1547)
• External data plate (14 CFR part 45, section 45.11)

The Pre-Flight Procedures is just your standard pre-flight that you have been doing since your first flight. Most POHs or AFMs have a detailed checklist and many pilots make their own. I have a flow that I use every time and then check the major items before I get into the plane.

Pre-Taxi
P – Prop Did I check for nicks and leaks?
C – Clean Windows?

C – Cowl Is it closed, Oil & Oil Filler cap?
F – Fuel: Water and levels?
A – Pitot-static ports clear? Pitot Heat?
R – Remove tiedowns and towbar?
T – Tires and brakes look OK?
S – Safety
Stall warning
Beacon and lights all work?

Visual Preflight Assessment
The inspection should start with the cabin door. If the door is hard to open or close, does not fit snugly, or the door latches do not engage or disengage smoothly, the surrounding structure, such as the door post, should be inspected for misalignment which could indicate structural damage. The visual preflight inspection should continue to the interior of the cabin or cockpit where carpeting should be inspected to ensure that it is serviceable, dry, and properly affixed; seats belts and shoulder harnesses should be inspected to ensure that they are free from fraying, latch properly, and are securely attached to their mounting fittings; seats should be inspected to ensure that the seats properly latch into the seat rails through the seat lock pins and that seat rail holes are not abnormally worn to an oval shape; [Figure 2-5] the windshield and windows should be inspected to ensure that they are clean and free from cracks, and crazing. A dirty, scratched, and/or a severely crazed window can result in near zero visibility due to light refraction at certain angles from the sun.
AFM/POH must be the reference for conducting the visual preflight inspection, and each manufacturer has a specified sequence for conducting the actions. In general, the following items are likely to be included in the AFM/POH preflight inspection:

• Master, alternator, and magneto switches are OFF
• Control column locks are REMOVED
• Landing gear control is DOWN
• Fuel selectors should be checked for proper operation in all positions, including the OFF position. Stiff fuel selectors or where the tank position is not legible or lacking detents are unacceptable.
• Trim wheels, which include elevator and may include rudder and aileron, are set for takeoff position.
• Avionics master OFF
• Circuit breakers checked IN

• Flight instruments must read correctly. Airspeed zero; altimeter when properly set to the current barometric setting should indicate the field elevation within 75 feet for IFR flight; the magnetic compass should indicate the airplane’ s direction accurately; and the compass correction card should be legible and complete. For conventional wet magnetic compasses, the instrument face must be clear and the instrument case full of fluid. A cloudy instrument face, bubbles in the fluid, or a partially filled case renders the compass unusable. The vertical speed indictor (VSI) should read zero. If the VSI does not show a zero reading, a small screwdriver can be used to zero the instrument. The VSI is the only flight instrument that a pilot has the prerogative to adjust. All others must be adjusted by an FAA-certificated repairman or mechanic.

• Mechanical air-driven gyro instruments must be inspected for signs of hazing on the instrument face, which may indicate leaks.
• If the airplane has retractable gear, landing gear down and locked lights are checked green.
• Check the landing gear switch is DOWN, then turn the master switch to the ON position and fuel qualities must be noted on the fuel quantity gauges and compared to a visual inspection of the tank level. If so equipped, fuel pumps may be placed in the ON position to verify fuel pressure in the proper operating range.
• Other items may include checking that lights for both the interior and exterior airplane positions are operating and any annunciator panel checks.

The rest of the section goes into detail about what to look for regarding cracks, wear, and leaks and is worth a read.

I’m not sure if the runup and radio/nav setup counts as pre-flight, but presumably it does, since the next section of the FAR is takeoffs.

Before-Takeoff Check
Most airplanes have at least the following systems checked and set:

• Fuel System—set per the AFM/POH and verified ON and the proper and correct fuel tanks selected.
• Trim—set for takeoff position which includes the elevator and may also include rudder and aileron trim.
• Flight Controls—checked throughout their entire operating range. This includes full aileron, elevator, and rudder deflection in all directions. Often, pilots do not exercise a full range of movement of the flight controls, which is not acceptable.
• Engine Operation—checked to ensure that temperatures and pressures are in their normal ranges; magneto or Full Authority Digital Engine Control (FADEC) operation on single or dual ignition are acceptable and within limits; and, if equipped, carburetor heat is functioning. If the airplane is equipped with a constant speed or feathering propeller, that its operation is acceptable; and at minimum idle, the engine rpm continues to run smoothly.
• Electrical System—verified to ensure voltages are within operating range and that the system shows the battery system charging.
• Vacuum System—must show an acceptable level of vacuum, which is typically between 4.8 and 5.2 inches of mercury (“Hg) at 2,000 rpm. Refer to the AFM/POH for the manufacturer’s values. It is important to ensure that mechanical gyroscopic instruments have adequate time to spool up to acceptable rpm in order for them to indicate properly. A hasty and quick taxi and run-up does not allow mechanical gyroscopic instruments to indicate properly and a departure into instrument meteorological conditions (IMC) is unadvisable.
• Flight Instruments—rechecked and set for the departure. Verify that the directional gyro and the magnetic compass are in agreement. If the directional gyro has a heading bug, it may be set to the runway heading that is in use or as assigned by air traffic control (ATC).
• A vionics—set with the appropriate frequencies, initial navigation sources and courses, autopilot preselects, transponder codes, and other settings and configurations based on the airplane’s equipment and flight requirements.
• Takeoff Briefing—made out loud by the pilot even when no other person is there to listen. A sample takeoff briefing may be the following:
“This will be normal takeoff (use normal, short, or soft as appropriate) from runway (use runway assigned), wind is from the (direction and speed), rotation speed is (use the specified or calculated manufacturer’s takeoff or rotation speed (VR)), an initial turn to (use planned heading) and climb to (use initial altitude in feet). The takeoff will be rejected for engine failure below VR, applying appropriate braking, stopping ahead. Engine failure after VR and with runway remaining, I will lower pitch to best glide speed, land, and apply appropriate braking, stopping straight ahead. Engine failure after VR and with no runway remaining, I will lower pitch to best glide speed, no turns will be made prior to (insert appropriate altitude), land in the most suitable area, and apply appropriate braking, avoiding hazards on the ground as much possible. If time permits, fuel, ignition, and electrical systems will be switched off.”

IFR Checkride Oral

March 10th, 2019

The day before the checkride we took advantage of a break in the weather to move the the checkride airplane from KSBP to KSMX. I found a hole in the clouds and did some scud running to get out of KSBP but KSMX was clear. On the way back we tried to practice some holds but the clouds came in fast and we ended up climbing to 7,000′ to get above them. One lesson I learned is that you can pick up ice even in light mist, so be careful when flying in air that is below freezing. The holes in the clouds filled in so I got to do GPS approach in actual back to the field.

The next day we drove down and the skies were clear. Unfortunately, by the time the oral was over lots of puffy clouds arrived at 2,600′ right over both ends of the runway. Since approaches start higher than that, there was no way to do a checkride without either lots of vectoring or going IFR, so we cancelled the practical portion. So I have 60 days to finish and the ride is scheduled for the next time he will be in town, either March 30 or 31.

The paperwork portion of the exam took a while, but the oral portion only took an hour and forty-five minutes. Make sure you bring your IACRA userID and password because you will need it to e-sign the paperwork.

I messed up two things, but otherwise did OK. He gave me a clearance from KLBG to KVNY with fly runway heading, radar vectors, V264, climb and maintain 4,000′, expect 5,000 five minutes after departure, V264, V394, SLI, 120.4, 3412.

Checkride KVNY to KLGB

Current weather was SKC ceiling 12,000′. You depart the airport and lose comms. What do you do? Easy question, you return to the airport. You are at 500′ and in the clouds what do you do? Remember MEA AVEnue F. The assigned is runway heading, so fly that. You don’t want to fly that forever, since you were going to be vectored to V264. You are expecting vectors to DARTS so that’s where you fly to. Altitude is the highest of Minimum IFR, Expect, or Assigned. So for the first 5 minutes, you fly 4,000′, then climb to 5,000′. It’s 11 miles to DARTS so you’ll probably be at 5,000′ for a little while before reaching DARTS. At DARTS, you join V186 where the MEA is 5,500′ so you climb. The MEA drops to 4,000′ on V394 and then 3,000′ after the turn but you stay at 5,500, which is the highest of Minimum IFR, expected, or assigned. At SLI there is a feeder route to the ILS approach. The feeder joins the approach at an angle of more than 90° so you will need to do the procedure turn. If you are early the rules say that you hold there until your ETA, but he agreed that ATC expects you to begin the approach. If you are high you can cut the power and get down in one circuit of the holding pattern.

I didn’t remember that you can enter Victor airways in Foreflight (you can’t do that on the GNS430), so I wasted some time looking up the fixes for the route. Then he reminded me that the route was one that he had given us beforehand, so I pulled it up. I also had printed the route so we used that version for the discussion.

The one thing I was unprepared was the departure from South Lake Tahoe. As I wrote last year in this post the SHOLE2 departure is the one to use. The issue, like the issue with the missed approach is can you depart with a minimum climb rate of 300′ per NM to 9000′. I got tangled up in calculating whether the C172 could make—which I know how to do, just didn’t pull up that info when we were talking.

MY CFII uses this examiner all the time, so he knows how he operates. He doesn’t ask clear questions so you have to guess what he wants and then expand on the answers. That tripped me up a couple of times when I gave him the rote answer and he asked, what else. We’ll there was nothing else, so I just started talking about things that I knew the ACS wants covered and that seemed to satisfy him. I have access to three airplanes, a Cherokee, a Cessna 210, and a Piper Arrow, so my strategy going in was to use them as examples for different scenarios. The CFI exams stress that the objective of the practical tests is to go past the rote memorization to understanding so I tried to give examples of the things I was regurgitating.

He started off asking me about the PAVE acronym. I hate memorizing stupid FAA acronyms and this one is no exception. I remembered that it is Pilot, Aircraft, and Environment, but forgot about Emotion. It makes me mad when I do that. He said we were going to use the acronym during the oral, but if we did, I don’t see how.

The first question was what does it take for you as a pilot to be legal for IFR. There is an acronym for that, but I don’t know what it is. I just remember that you need to have a Flight Review, medical, and IFR rating and 6 approaches, holding patterns, and intercepting courses in the previous six months. This is the first time he asked “What else?”. I guessed that he was asking about carrying passengers so I gave him that info. He asked again, so I started talking about proficiency versus currency and talked about the ways you could be current but not proficient. This came up a couple of times in the exam. I also digressed into Basic Med and how pilots on any medical need to self-certify before each flight. Talked a little about how Basic Med works, e.g. I can fly any of the planes I have access to up to 18,000′ but not to Mexico or Canada (Update: Basic Med is now accepted by Mexico and the Bahamas). He needs a third-class or better to be a DPE but a CFI only needs Basic Med (Update: If acting as PIC, e.g. Instrument training, Flight Review with a person whose Flight Review was more than 24 months ago, or Student Pilot. No medical is required of not acting as PIC.). I also digressed a bit to talk about how there are even STCs for converting a 7-seat Saratoga to a 6-seater so you can fly it with Basic Med.

Then we went on to what equipment you need to fly IFR. I talked about the GRABCARD items and as far as I know that’s it. He wanted to know what else, and I was stumped, but as I was writing this, I think he might have been referring to all of the dar VFR items that are required as well, TOMATOFLAAMES. That didn’t occur to me so I started talking about how you aren’t required to have a heated pitot-tube and my Cherokee doesn’t but it would be a good idea if flying for any time in the clouds. I touched a bit on anti-icing systems and de-ice systems and how our planes are not certified for known-ice. Since I ran out of things that are required I started talking about how to placard inoperative items and how an old LORAN in the Arrow was placarded since it isn’t worth $150 to have it taken out. Also how I left an old radio in the Cherokee and placarded the nav side.

I think he got bored listening to me so he switched to systems. The first was the pitot-static system and how it works. I like the explanation Andy Munnis gives to I talked about that. This was a good place to bring up the Air France crash when they mis-identified clogged pitot for loss of airspeed and kept pulling up. As you know, if the pitot tube is clogged, the airspeed acts like an altimeter. So as you pull up, the altitude increases and the airspeed increases, so you pull up some more. It’s not clear why the crew didn’t cross check other instruments, especially the pitch and power, but they didn’t and stalled the aircraft.

Then he moved to the vacuum system. I brought up the fact that non-catastrophic failure is especially hard to catch because it happens gradually. I blanked for a moment on how to detect a failure, confusing the alternator warning light for the vacuum gauge, but then I remembered.

We moved on to the fuel system. The system on the C172 is dead simple. You only have a choice of L/R/Both and normally fly with both. I told him a story about a pilot who told his passenger to pull the red know to get air if they got hot. A little while later, the engine died. Turns out that the passenger pulled the fuel shut-off knob. I also talked about how the fuel selector on the Cherokee doesn’t have detents and you have to be careful to watch fuel flow after changing tanks. And finally talked about a pilot taking off from Catalina who rushed to get ahead of a fog bank that was rolling in. For some reason, he turned his fuel selectors off when shutting down and when he started up, didn’t remember to turn one engine back on. He had enough fuel in the lines to start and taxi, but when he took off her ran out a few hundred feet off the ground, rolled the airplane, and ended up at the bottom of a cliff—on fire.

He then moved on to talking about the flight plans. I covered this in a previous post and he is using the same basic flight plans now, except that he skipped Paso Robles and Whiteman.

We didn’t talk about much more than what is covered in that post. We had some interesting weather the week before and I printed out some of the icing and turbulence charts for the route. Icing up to 7,000′ was not forecast from KSMX to 3O8 but there was a 25% probability at 9,000′. I mentioned that it was legally flyable but that I wouldn’t do it in the 172, but I probably would in the Turbo 210, since I would have enough power to turn around and get back to an area without ice. I also mentioned that icing layers are usually not too thick, only three or four thousand feet, but climbing to a layer above where there is no icing isn’t usually possible in the planes we fly. I think this is where he asked about the kinds of icing and I just talked about rime, clear, and mixed and how rime was dangerous because it was usually caused by super-cooled liquid droplets and smooth was dangerous because it was hard to see. Either one will cause the airplane to stall because it disrupts the flow of air over the wing. One thing I didn’t mention is that in airplanes with altitude hold or select, you should disable the altitude mode of the autopilot. The autopilot could keep increasing the angle of attack to compensate for the ice, until it reaches a point where the autopilot kicks off and the airplane stalls.

For the last question he told me that we were departing Van Nuys at 10:00 PM and the destination was Camarillo, the entire area was 600′ overcast and Camarillo was 1,100 overcast 2 mile visibility. Did I need an alternate. Yes. Can I use Oxnard if the ceiling is 700 and 3 miles, There is an ILS there and the standard minimum would be 600 and 2 but there is a note on the chart for non-standard minimums. The note says that the ILS is not available if the tower is closed. Then he asked if I can use Camarillo as an alternate if the ceiling was 800′. At first I said no, since the tower was closed, but then I looked more carefully at the note and it only applied to the ILS—not the GPS approaches. To use a GPS approach as an alternate, you need to have visibility greater than the LNAV or circling minimums and 800′ meets that standard.

As I was doing this he was looking out the runway and after an hour and forty-five minutes he asked if the weather was good enough to fly today. The ceiling was few at 2,600 and 10 miles vis, wind right down the runway a 8 kts. So it was perfect for flying but both ends of the runway were covered in clouds so my answer was that unless he was willing to vector me a lot on the approaches, then we couldn’t to the flying portion. The clouds looked like they were moving in from the south so the chance of it clearing in the near future were slim, so we cancelled.

On the way home, I tracked the localizer outbound and couldn’t get higher than 2,000′ and still stay below the clouds so it was a good decision.

What are the minimum requirements to file and fly IFR?

March 3rd, 2019

The oral portion of the checkride starts with Preflight Preparation and the applicant is expected to demonstrate an understanding of certification requirements, recency of experience, and recordkeeping.

Qualified
The first part applies to anyone flying an airplane. Do you have a current flight review and medical? Are you endorsed for the aircraft? Tailwheel, high-performance, complex?

Current
For day flights with passengers you need to have three take-offs and landings in the same category and class of aircraft—to a full stop for night flight. Where night is 1 hour after sunset to 1 hour before sunrise.

For flight under IFR, you need 6 approaches, tracking courses, and holding patterns in the previous six months.

Capable
Just because you are legal to fly doesn’t mean you should. The FAA loves acronyms and one of them is IMSAFE.

IMSAFE Checklist

  1. Illness—Am I sick? Illness is an obvious pilot risk.
  2. Medication—Am I taking any medicines that might affect my judgment or make me drowsy?
  3. Stress—Stress causes concentration and performance problems. While the regulations list medical conditions that require grounding, stress is not among them.
  4. Alcohol—Have I been drinking within 8 hours? Within 24 hours?
  5. Fatigue—Am I tired and not adequately rested? Fatigue continues to be one of the most insidious hazards to flight safety, as it may not be apparent to a pilot until serious errors are made.
  6. Emotions-A pilot who experiences an emotionally upsetting event should refrain from flying until the pilot has satisfactorily recovered.

Personal Minimums
Right now I am night current, having flown three takeoffs and landings at night two months ago in my Cherokee. But my last cross-country night flight was two and a half years ago with an instructor. While it would be legal to fly with passengers on a cross-country flight in a Piper Arrow that I have flown a couple of times, it wouldn’t be prudent.

I’m finishing up my IFR rating and would be comfortable popping through the marine layer in my Cherokee to go somewhere the day after I get my rating. It doesn’t have an autopilot or GPS so I wouldn’t want to fly very long or to minimums in it. I only have a few minutes of actual in my logbook, so I would want to get some more practice in the system popping through puffy clouds before I attempted to fly hard IFR.

If I had a plane that I was familiar with and just needed to drop through the clouds to get home, my personal minimums right now would be 1,000′ AGL ceiling, 5 miles visibility, and minimal winds 10-15 kts.

RAIM and the Service Availability Prediction Tool

March 3rd, 2019

Non-WAAS GPS units (and WAAS units if WAAS is not available) have the ability to self-detect whether they can accurately depict the aircraft’s position. This is called Receiver Autonomous Integrity Monitoring (RAIM). The GPS does a bunch of calculations using various combinations of GPS satellites and if the answers match, you are good to go. If they don’t match, then you will get an alert and must rely on ground-based navigation to complete your flight.

The GPS requires five satellites to make the calculations. If the unit has baro-aiding, and most do, then they only need four. Since satellites follow a know orbit, the availability of four or five for doing calculations can be predicted in advance. Barring failure of the satellite, you can know up to three days in advance of your trip whether the GPS signals will be adequate for the en route, terminal, or approach phase of the flight. The Service Availability Prediction Tool (SAPT) website has maps the display service availability.

As you can see from the maps, if you have a unit with baro-aiding, then RAIM outages are no an issue. With baro-aiding there are occasionally some areas of south Florida that have issues.

RAIM Summary Pages

RAIM Outages NPA No Baro-Aiding

RAIM Outages NPA Baro-Aiding

AC 90-108: GPS in Lieu of Ground Based Nav

February 25th, 2019

Guidance on using RNAV systems, what the AIM refers to as GPS (non-WAAS) and WAAS, is scattered around various publications. AC 90-108 addresses using GPS in place of equipment that you probably no longer have in your panel—ADF and DME—and for determining fixes from cross-radials. Basically, you can use GPS to determine distances and fixes on an approach but you may not use it as the sole means of lateral guidance for a localizer based approach or VOR approach past the FAF. The exception is when the approach is labelled “… OR GPS”.

NOTE: This AC does not address the use of RNAV systems on RNAV routes and RNAV terminal procedures. The current edition of AC 90-100, U.S. Terminal and En Route Area Navigation (RNAV) Operations, applies to those operations. This AC also does not address the use of RNAV systems on instrument approach procedures (IAP) titled, RNAV (GPS) and GPS. The current edition of AC 90-105, Approval Guidance for RNP Operations and Barometric Vertical Navigation in the U.S. National Airspace System, applies to those operations.

7. USES OF SUITABLE RNAV SYSTEMS.
a. Usage of Suitable RNAV Systems. Subject to the operating requirements in this AC,
operators may use a suitable RNAV system in the following ways.

(1) Determine aircraft position relative to or distance from a VOR (see first note in subparagraph 7b), TACAN, NDB, compass locator (see second note in subparagraph 7b), DME fix; or a named fix defined by a VOR radial, TACAN course, NDB bearing, or compass locator bearing intersecting a VOR or Localizer (LOC) course.

(2) Navigate to or from a VOR, TACAN, NDB, or compass locator. (3) Hold over a VOR, TACAN, NDB, compass locator, or DME fix. (4) Fly an arc based upon DME

8. USES OF SUITABLE RNAV SYSTEMS NOT ALLOWED BY THIS AC.
An otherwise suitable RNAV system cannot be used for the following:

a. NOTAMed Procedures. Unless otherwise specified, navigation on procedures that are identified as not authorized (“NA”) without exception by a NOTAM. For example, an operator may not use a RNAV system to navigate on a procedure affected by an expired or unsatisfactory flight inspection, or a procedure that is based upon a recently decommissioned NAVAID.

b. Substitution on a Final Approach Segment (FAS). Substitution for the NAVAID (for example, a VOR or NDB) providing lateral guidance for the FAS.

c. Lateral Navigation on LOC-Based Courses. Lateral navigation on LOC-based courses (including LOC back-course guidance) without reference to raw LOC data.

There is an update in the AIM which allows to fly the final approach segment or VOR, TACAN or NDB approaches with GPS lateral course guidance provided that the underling navaid is monitored.

AIM 1−2−3. Use of Suitable Area Navigation (RNAV) Systems on Conventional Procedures and Routes
NOTE−
2. These operations do not include lateral navigation on localizer−based courses (including localizer back−course guidance) without reference to raw localizer data.

5. Use of a suitable RNAV system as a means to navigate on the final approach segment of an instrument approach procedure based on a VOR, TACAN or NDB signal, is allowable. The underlying NAVAID must be operational and the NAVAID monitored for final segment course alignment.

IFR Use of GPS: Database Requirements

February 25th, 2019

2. IFR Use of GPS

(b) Database Requirements. The onboard navigation data must be current and appropriate for the region of intended operation and should include the navigation aids, waypoints, and relevant coded terminal airspace procedures for the departure, arrival, and alternate airfields.

(1) Further database guidance for terminal and en route requirements may be found in AC 90-100, U.S. Terminal and En Route Area Navigation (RNAV) Operations.

(2) Further database guidance on Required Navigation Performance (RNP) instrument approach operations, RNP terminal, and RNP en route requirements may be found in AC 90-105, Approval Guidance for RNP Operations and Barometric Vertical Navigation in the U.S. National Airspace System.

(3) All approach procedures to be flown must be retrievable from the current airborne navigation database supplied by the equipment manufacturer or other FAA−approved source. The system must be able to retrieve the procedure by name from the aircraft navigation database, not just as a manually entered series of waypoints. Manual entry of waypoints using latitude/longitude or place/bearing is not permitted for approach procedures.

(4) Prior to using a procedure or waypoint retrieved from the airborne navigation database, the pilot should verify the validity of the database. This verification should include the following preflight and inflight steps:

Rental Rates in 1979

February 20th, 2019

I ran across this in the files of an old pilot. For context, I was working in my first job out of college as an insurance underwriter around this time and making $205 per week.

Rental Rates 1979

http://touringmachine.com/images/%20Rental_Rates_1979.jpeg

Barometric Aiding

February 18th, 2019

I have been flying a Cessna 172SP with an old-school Bendix stack and unlike the Garmin GPS and S-Tec autopilots that I am used to flying, I need to enter the altimeter setting into the KLN 94 GPS and the KAP 140 Autopilot. Most of the GPS systems in GA aircraft use Barometric Aiding to assist in detecting integrity anomalies. Most Garmin units interface directly with the altitude encoder to provide baro-aiding, but other units require the pilot to enter the local altimeter setting. AOPA has an article that lists non-WAAS GPSs that have baro-aiding. WAAS-enabled GPS units do not rely on RAIM so they do not rely on baro-aiding.

Barometric aiding is an integrity augmentation that allows a GPS system to use a non-satellite input source (e.g. the aircraft pitot-static system) to provide vertical reference and reduces the number of required satellites from five to four. Baro-aiding requires four satellites and a barometric altimeter input to detect an integrity anomaly. The current altimeter setting may need to be entered into the receiver as described in the operating manual. Baro-aiding satisfies the Receiver Autonomous Integrity Monitoring (RAIM) requirement in lieu of a fifth satellite.

As usual, John D Collins has an excellent discussion of this.

The RAIM algorithm can be improve upon by using the aircraft pressure altitude, as the vertical position, although less accurate than the lateral position, can substitute for one of the satellites. This is called Baro Aiding and probably all IFR GPS installations use this. With Baro Aiding, only four satellites are needed to determine a value of RAIM.

With WAAS, the calculations are determined by the WAAS system as each ground station that measures its position via the GPS satellites is at a know location, and the error can be determined one satellite at a time. This integrity data is uplinked to the WAAS satellites and is broadcast as part of the WAAS correction messages. So the WAAS system itself is determining the integrity information via explicit measurement and passing it on to the WAAS receiver to determine the HPL.

The autopilot has an altitude capture and hold function where you can select an altitude and climb or descend rate and the autopilot will adjust the elevators to capture the rate and altitude. You still need to set the appropriate power for a climb or descent. You need to enter the altimeter setting so it knows where to stop or hold.

Checkride Prep: KSMX Approaches

February 4th, 2019

I am taking the checkride at KSMX and we have been practicing the approaches and holds that the examiner likes to use. There are five different approaches there so you get to practice just about everything you might encounter in the wild.

Any time you are flying with your iPad, it would be a good idea to have paper backups of your destination and potential alternate IAPs. You can buy the charts, but they go out of date quickly and are expensive. The FAA website has all of them for free on their Digital Products Page. You can download the entire chart supplement and terminal procedures publications or search for just the airport you need and the approaches you airplane is equipped for.

I searched for the KSMX Approaches and printed them out. ForeFlight doesn’t have the Airport Legend as a separate document, so I downloaded it and added it to my documents. Likewise, while you can always find the alternate minimums and takeoff minimums in the airport page of ForeFlight, it doesn’t save the page, so I downloaded them as well, saved just the KSMX page, and added it to my ForeFlight documents.

The first thing to note on the IAPs is the date at the bottom left corner of the plate. You don’t need to print every plate each cycle, but if the date has changed since the last cycle, you need to update your printed version. Unlike Jepp charts, these don’t tell you what changed, so if you use the approach frequently, you might want to make sure you are aware of significant changes. If you are used to flying an approach and the date has changed, make sure that you don’t fly the old numbers by memory e.g. minimums or frequencies. You can also look at the FLAG column on the download page. If it is blank, there has been no change. KSMX has had no changes since the last cycle, so we are good. You can check for all IAPs that have been changed, added, or deleted in your area by using the advanced search feature of the website.

All IAPs follow the same format, but there are some things that you might miss if you aren’t paying close attention. The date is underlined at the bottom. This approach has a NOTAM for higher minimums which suggests that that the chart may change or that when the temporary obstruction is moved the minimums will revert back. There are a whole bunch of words related to using the Vandenberg altimeter setting and you might miss that RWY 2 is NA at night. When you have a note like this, you can often figure out why and keep the information in mind when landing on other runways. In this case there is an obstruction at 340′ MSL just off the end of the runway. If you are making left traffic for RWY 12, you should be aware of it even though you shouldn’t be 120′ above the airport at that point anyway.

The missed approach is always described at the right of the remarks section. A graphical depiction is always on the profile view and it is depicted on the plan view. If the Missed Approach Hold is off the chart, it is depicted in a box near the holding point.

The three red boxes are all related. In the profile view it tells you to remain within 10 nm of the VOR when executing the procedure turn. You can use your GPS to determine this distance, but if you notice the box around GASRE, the chart tells you that GASRE is 10nm from the VOR and you can begin your stepdown there if you started your approach from the MQO VOR. If you started the approach at the GVO VOR, then you can begin your stepdown to 1500 when inbound after the procedure turn.

If you are using GPS for your approach, it is easy to identify GASRE. But you can also identify it by the intersection of the MQO 131° radial and FLW 254° radial. If you have DME, it is 11.5 DME from MQO and 10 DME from GVO on the depicted radials.

One other thing to note on this approach, that I didn’t highlight, is that there is a maximum altitude of 6000′ MSL on the procedure turn outbound. In a radar environment that won’t be an issue but in a lost comms scenario you will need to pay attention to it.

KSMX VOR 12 Annotated

You can find the same kinds of things on the Back Course IAP. In this IAP the plan view contains the location of the Missed Approach hold so it is not contained in a box. There are no NOTAMs for this approach. There are two notes in the profile view which should be obvious. First, disregard any glide slope indications. The second is that you can use DME from the front course to determine stepdown fixes.

Back Course

I figured out why you were high.

February 2nd, 2019

I was invited to go along an a flight and when I got to the airport I found out that we were going to Monterey (KMRY). I had been there 20 years ago for a stop and taxi-back with another pilot but I didn’t remember anything about the airport.

As we approached the airport the pilot asked me what the traffic pattern altitude was and I looked it up in ForeFlight—1757 MSL. That seemed a bit high to me since, while I have never landed there, I have flown over it a bunch of times and I thought it was right on the ocean. We approached from the south and the airport was hidden by mountains. As the airport came into view, I thought that maybe the reason the TPA was so high was because the mountains on the south side were fairly high and that accounted for the high TPA. We were vectored up the coast for a right downwind to 28L. As we approached the runway on final we were really high. I attributed part of it to the odd sight picture. The surrounding terrain on the north side was at sea level and the airport is up on an island with a really long runway. I figured that he just got the sight picture wrong and called for a go-around. On the second try we were still hight but not too high to land.

When I got back I took a look at the charts and Chart Supplement and two things jumped out at me. Do you see the same things?

KMRY Chart Supplement

I have never been to an airport with a TPA that is 1500′ above the airport and neither had the pilot. So you need to drop another 500′ from what you are used to. The runway is the same width as what I am used to but it has an upslope of 1.4% which throws off your sight picture and it is longer than any runways I normally use which also throws off the sight picture.

Also notice that 28L and 28R both have right traffic, so the higher TPA isn’t explained by the higher terrain to the south—in fact the right pattern is probably to keep you out of the mountains.

Cessna 172P

January 22nd, 2019

I’m taking a checkride in a stock Cessna 172P that came from the factory with Bendix/King avionics. It has a GPS and autopilot that I am not familiar with. This post has some videos and pilot information manuals that you might find helpful if you fly one of these.

The GPS is a KLN 94 non-WAAS GPS only device. If you are familiar with other GPSs the operation is pretty straight-forward. These videos are useful for understanding the operation.

KLN 94 GPS
Pilots Guide
Quick Reference

KMD 550 Multi-Function Display
Pilot’s Guide
Quick Reference

Here’s how to use it to fly an approach.

This is a longer overview of the GPS that highlights some features that you might find useful.

And a quick take on using it in flight.

KLN 94 Startup Checklist

Nav Mode Button set to GPS
Altimeter Setting input
DME reading 34.5 and 2.5 dot deflection
Deflection on OBS also 2.5 dot FROM
Change OBS course and note change on unit
Verify current airport in reference section
Verify current database

KLN 94 Hints
Nav4 Page – Press menu button
Select Waypoints: Pull and turn the small knob. Press Direct to to go to the selected waypoint.

Flight Plan: Large knob to select FPL, small knob to select FPL 0

AUX 3 page predicts if and when RAIM (Receiver Autonomous Integrity Monitoring) capability will be available when you are ready to shoot a non-precision approach at your destination airport.

KAP 140 Autopilot

This is a good overview in X-Plane in a Cessna 172 so you can get an overview of the features of the autopilot.

This is detailed explanation of all the features of the autopilot. Skip the first three minutes since it discusses how to set parameters if you are building an airplane in X-Plane.

This is a longer more detailed look in a sim with a Garmin 1000 that goes into more details of the features of the autopilot.

Pilot’s Guide
Quick Reference
Autopilot Tutorial

Nav/Comm
If you have flown a variety of airplanes, you’ve probably run across the Bendix/King KX155 or KX 165 before. The basic operation is simple, just remember to pull out the small knob to get frequencies ending in 25 or 75 e.g. For 127.725 tune to 127.7 then pull out the small know to get the last two digits. Pilot’s Guide

The AFM for these is serial number specific, but there is a copy of one that a flight school has put online that contains information that applies to all Cessna 172SPs. AFM

Filing an IFR Alternate with GPS

January 16th, 2019

The rules governing approaches for GPS RNAV systems and WAAS-capable RNAV systems are different so it is important to be able to decipher the jargon when reading the AIM. The first section quoted below refers to GPS systems i.e. non-WAAS. The point of the special rules using GPS as an alternate has to do with the fact that non-WAAS GPS is supplementary navigation. So you can file with the destination or the alternate having only a GPS approach, but not both. WAAS-equipped aircraft can file for destinations and alternates having only GPS approaches. In both cases, the weather at the time of arrival must satisfy the LNAV or circling minimums or higher if noted.

AIM 1−1−17. Global Positioning System (GPS)
5. GPS Instrument Approach Procedures
(c) For flight planning purposes, TSO-C129() and TSO-C196()−equipped users (GPS users) whose navigation systems have fault detection and exclusion (FDE) capability, who perform a preflight RAIM prediction for the approach integrity at the airport where the RNAV (GPS) approach will be flown, and have proper knowledge and any required training and/or approval to conduct a GPS-based IAP, may file based on a GPS−based IAP at either the destination or the alternate airport, but not at both locations. At the alternate airport, pilots may plan for:
(1) Lateral navigation (LNAV) or circling minimum descent altitude (MDA);
(2) LNAV/vertical navigation (LNAV/ VNAV) DA, if equipped with and using approved barometric vertical navigation (baro-VNAV) equipment;
(3) RNP 0.3 DA on an RNAV (RNP) IAP, if they are specifically authorized users using approved baro-VNAV equipment and the pilot has verified required navigation performance (RNP) availability through an approved prediction program.

AIM 1−1−18. Wide Area Augmentation System (WAAS)
9 (a) Pilots with WAAS receivers may flight plan to use any instrument approach procedure authorized for use with their WAAS avionics as the planned approach at a required alternate, with the following restrictions. When using WAAS at an alternate airport, flight planning must be based on flying the RNAV (GPS) LNAV or circling minima line, or minima on a GPS approach procedure, or conventional approach procedure with “or GPS” in the title.

For more details, see Notice N 8900.218.

IFR Alternate Required

January 16th, 2019

The 1-2-3 Rule for determining if an alternate is required comes from CFR §91.169 as highlighted below. The way the FAR is written an alternate is always required unless certain conditions are met. Basically, if your destination has an instrument approach then, for 1 hour before to 1 hour after your anticipated arrival at your destination the weather does not have a ceiling of 2,000′ and visibility of 3 miles, an alternate is required. The alternate must have a ceiling of 600′ and visibility of 2 statute miles for precision approaches and 800′ and 2 miles for non-precision approaches unless non-standard alternates are published for the approach. If there is no published approach at the alternate, then the ceiling and visibility minima are those allowing descent from the MEA, approach, and landing under basic VFR.

Scenarios:
Destination Palm Springs (KPSP). The airport has multiple approaches and weather reporting. But unless we are RNP equipped, we can only fly the VOR or GPS-B approach. Apply the 1-2-3 rule to see if an alternate is required. However, note that the minimum ceiling for the approach is 2,300′ so even if you meet the 1-2-3 rule, you may still not be able to land. An alternate would be a good idea in that case.

Destination Bermuda Dunes (KUDD). The airport has multiple approaches but no weather reporting. The nearest weather forecast (TAF) is 8 miles away at Palm Springs (KTRM) and 10 miles away at Palm Springs (KPSP). There is no way to apply the 1-2-3 rule so an alternate is required.

Destination Borrego Valley (L08). There is weather reporting but no forecast and one GPS approach. An alternate is required.

Destination King City (KKIC). There is weather reporting but no forecast and no approaches. An alternate is required.

Destination Harris Ranch (308). There is no weather reporting, no forecast, and no approaches. An alternate is required.

Alternate Bermuda Dunes (KUDD). The airport has multiple approaches but no weather reporting but all three of the approaches show Alternate Minimums NA Symbol in the Pilot Briefing section of the approach plate so they are not available as an alternate. This is true even if, by looking at the weather at KTRM and KPSP you can determine that the ceiling and visibility in the area would allow descent from the MEA, approach, and landing under basic VFR.

Alternate Palm Springs, Jacqueline Cochran International (KRTM). The airport has two GPS approaches and two VOR approaches. All of them show Alternate Minimums Symbol in the briefing section, so we can check to see what the restrictions are and see that it just bumps the ceiling and visibility from standard. If the weather at the time of arrival meets those minimums, then we can file it as an alternate. When we get there we can use any approach that the weather permits.

Alternate Borrego Valley (L08). There is one approach showing Alternate Minimums NA Symbol in the Pilot Briefing section of the approach plate. It is not available as an alternate because when you look up the alternate minimums, it says “NA when local weather not available” and there is no TAF.

The rule for filing with GPS is not spelled out in the FAR. This post goes into detail. Basically, if you have a non-WAAS GPS you can file either the destination or alternate with a GPS approach—but not both. If you have WAAS, you both can have only GPS IAPs. You must to use the LNAV or circling minimums at the destination.

§91.169 IFR flight plan: Information required.
(a) Information required. Unless otherwise authorized by ATC, each person filing an IFR flight plan must include in it the following information:

(1) Information required under §91.153 (a) of this part;

(2) Except as provided in paragraph (b) of this section, an alternate airport.

(b) Paragraph (a)(2) of this section does not apply if :

(1) Part 97 of this chapter prescribes a standard instrument approach procedure to, or a special instrument approach procedure has been issued by the Administrator to the operator for, the first airport of intended landing; and

(2) Appropriate weather reports or weather forecasts, or a combination of them, indicate the following:

(i) For aircraft other than helicopters. For at least 1 hour before and for 1 hour after the estimated time of arrival, the ceiling will be at least 2,000 feet above the airport elevation and the visibility will be at least 3 statute miles.

(ii) For helicopters.…

(c) IFR alternate airport weather minima. Unless otherwise authorized by the Administrator, no person may include an alternate airport in an IFR flight plan unless appropriate weather reports or weather forecasts, or a combination of them, indicate that, at the estimated time of arrival at the alternate airport, the ceiling and visibility at that airport will be at or above the following weather minima:

(1) If an instrument approach procedure has been published in part 97 of this chapter, or a special instrument approach procedure has been issued by the Administrator to the operator, for that airport, the following minima:

(i) For aircraft other than helicopters: The alternate airport minima specified in that procedure, or if none are specified the following standard approach minima:

(A) For a precision approach procedure. Ceiling 600 feet and visibility 2 statute miles.

(B) For a nonprecision approach procedure. Ceiling 800 feet and visibility 2 statute miles.

(ii) For helicopters: …

(2) If no instrument approach procedure has been published in part 97 of this chapter and no special instrument approach procedure has been issued by the Administrator to the operator, for the alternate airport, the ceiling and visibility minima are those allowing descent from the MEA, approach, and landing under basic VFR.

(d) Cancellation. When a flight plan has been activated, the pilot in command, upon canceling or completing the flight under the flight plan, shall notify an FAA Flight Service Station or ATC facility.

Fly-over or fly-by on airways?

November 24th, 2018

Someone asked this on a flying blog and while, I was taught to anticipate turns and my Garmin 430 will anticipate turns I don’t remember any specific guidance on the matter, so I looked it up. I found this in the AIM.

5−3−5. Airway or Route Course Changes

a. Pilots of aircraft are required to adhere to airways or routes being flown. Special attention must be given to this requirement during course changes. Each course change consists of variables that make the technique applicable in each case a matter only the pilot can resolve. Some variables which must be considered are turn radius, wind effect, airspeed, degree of turn, and cockpit instrumentation. An early turn, as illustrated below, is one method of adhering to airways or routes. The use of any available cockpit instrumentation, such as Distance Measuring Equipment, may be used by the pilot to lead the turn when making course changes. This is consistent with the intent of 14 CFR Section 91.181, which requires pilots to operate along the centerline of an airway and along the direct course between navigational aids or fixes.

b. Turns which begin at or after fix passage may exceed airway or route boundaries. FIG 5−3−1 contains an example flight track depicting this, together with an example of an early turn.

c. Without such actions as leading a turn, aircraft operating in excess of 290 knots true air speed (TAS) can exceed the normal airway or route boundaries… Consequently, the FAA expects pilots to lead turns and take other actions they consider necessary during course changes to adhere as closely as possible to the airways or route being flown.

AIM Fig 5-3-1

Knowledge Tests for an AGI/IGI or CFI

November 20th, 2018

Our local flight school had a dedicated AGI/IGI for a while. Lots of CFIs don”t like doing ground instruction and, if they are building hours, every hour on the ground is an hour they are not building time. I like teaching and have been tutoring high school algebra and helping guys who rent the Cherokee with their PPL tests. I thought it might be nice to get paid for teaching, and also to be able to sign people off for tests so I just took and passed all three ground instructor tests. Now I need to fly over to a FSDO to get my certificate.

For the Fundamentals of Instruction Knowledge Test, study the Gleim book. If you have time, and if you want to be a CFI, at some point you will need to read FAA-H-8083-9A Aviation Instructor’s Handbook but you don’t need to read it to pass the exam. A lot of the information in the book is wrong, so I had a hard time memorizing the wrong answer to questions. Most people probably won’t have to worry about that unless you have a background in psychology or education.

For the IGI and AGI it probably helps to read FAA books or whatever you prefer for getting your aviation knowledge. And you definitely need to study FAR 61 and 91 and know the VFR and IFR charts. I used the Gleim books Flight/Ground Instructor and Instrument Pilot and there were only a few questions that were completely new to me. But I also wrote all of these posts and the answers to the test questions in the test portion of this website, so your mileage may vary.

I’d say that about half of the questions are really stupid. e.g.

The force acting opposite of lift is:
A. Apples
B. Gravity
C. Oranges

Another quarter were interpolation/math questions that you will never ever need while flying. The study guides do a good job preparing you for them. The trick it to read the question really carefully. e.g. Are they asking you to look on the chart for landing distance or do you need to do another calculation to get ground roll.

There are a bunch of E6B questions that you just need to be really careful in your arithmetic and reading the question.

There are a few that I really had no idea what the question was, let alone what the right answer was. I got about half of those right. There were also a couple on each test where none of the answers was the correct answer.

I took the Commercial and CFI since they are good for two years and I won’t have to study them again if I decide to get a CFI.

My results: FOI 84%, AGI 88%, IGI 90%, CFI 87%, COM 92%. I took the Instrument last year and got a 96%.

I studied for a month, which was a bit of overkill, but I also wrote up a lot of these posts so that took some time.

Sporty’s and King Schools let you take practice tests for free. I put a lot of my notes up on this website so you can save time when studying by printing and memorizing them.

Commercial Pilot Opportunities

November 5th, 2018

You will often see lists of things that you can do with your commercial pilot certificate either as a job or to build time for the airlines. You can work for a Part 135 operation or in a corporate flight department operating under Part 91 but there are other things you can do as well. You can even fly individuals in their own airplane for hire. You can’t carry them in your airplane, since this would be a Part 135 operation.

§119.1 of the FARs is where those other activities are spelled out—basically listing exceptions to needing to comply with the rules for air carriers and cargo operations.

§119.1 Applicability.

(e) Except for operations when common carriage is not involved conducted with airplanes having a passenger-seat configuration of 20 seats or more, excluding any required crewmember seat, or a payload capacity of 6,000 pounds or more, this part does not apply to

(1) Student instruction;

(2) Nonstop Commercial Air Tours conducted after September 11, 2007, in an airplane or helicopter having a standard airworthiness certificate and passenger-seat configuration of 30 seats or fewer and a maximum payload capacity of 7,500 pounds or less that begin and end at the same airport, and are conducted within a 25-statute mile radius of that airport, in compliance with the Letter of Authorization issued under §91.147 of this chapter. For nonstop Commercial Air Tours conducted in accordance with part 136, subpart B of this chapter, National Parks Air Tour Management, the requirements of part 119 of this chapter apply unless excepted in §136.37(g)(2). For Nonstop Commercial Air Tours conducted in the vicinity of the Grand Canyon National Park, Arizona, the requirements of SFAR 50-2, part 93, subpart U, and part 119 of this chapter, as applicable, apply.

(3) Ferry or training flights;

(4) Aerial work operations, including—

   (i) Crop dusting, seeding, spraying, and bird chasing;
   (ii) Banner towing;
   (iii) Aerial photography or survey;
   (iv) Fire fighting;
   (v) Helicopter operations in construction or repair work (but it does apply to transportation to and from the site of operations); and
   (vi) Powerline or pipeline patrol;

(5) Sightseeing flights conducted in hot air balloons;

(6) Nonstop flights conducted within a 25-statute-mile radius of the airport of takeoff carrying persons or objects for the purpose of conducting intentional parachute operations.

(7) Helicopter flights conducted within a 25 statute mile radius of the airport of takeoff if— [Lots of stuff about limitations here]

(8) Operations conducted under part 133 of this chapter or 375 of this title;

(9) Emergency mail service conducted under 49 U.S.C. 41906;

(10) Operations conducted under the provisions of §91.321 of this chapter; or

(11) Small UAS operations conducted under part 107 of this chapter.

Turn and Slip vs. Turn Coordinator

October 26th, 2018

The difference between the two is something you need to memorize for your instrument knowledge test, but don’t necessarily have to understand. John D. Collins on Oct 28, 2012 in a post on Ask A CFI explains it quite well.

The TC indicator provides both bank rate and turn rate, whereas the TS only provides turn rate. Because the TC is affected simultaneously by turn and yaw, it can be difficult to use it to recover from an upset. The old TS has the advantage that it only indicates turn rate, and if you are partial panel, this can save your life.

Back in the mid 1920’s it was fatal if you flew into a cloud. A pilot named Howard Stark worked out a method using the newly developed Turn and Slip Indicator by Sperry to safely fly in the clouds with this instrument. If an upset occurred, he employed what came to be known as the Stark 1-2-3 method to recover control.

First, stop the turn with the rudder so the turn needle is in the center, second center the ball by using the ailerons to level the wings and third control the dive with use of the airspeed indicator and the stick to control the elevators. This method is foolproof and is still used by many experienced pilots today.

Unfortunately, it doesn’t work with the Turn Coordinator because the indicator shows both rate of turn and rate of bank. There are many old and not so bold pilots who either replace the turn coordinator with a turn and slip indicator or install a spare turn and slip in their aircraft when it is equipped with a turn coordinator that is required for the autopilot operation.

More info at AvWeb as well.

CFI and AGI Tests: Miscellaneous Things I Can’t Remember

October 24th, 2018

There are a bunch of questions where the answer that the test-prep books say is the correct answer, is wrong. There are also a bunch where the question probably makes sense in context, but qualifying words are omitted from the question. There are also lots of them where there are two or more correct answers and you need to guess which one the FAA wants. And then there are a few that I just can’t remember the answer to. This is my list. I’d recommend that you prepare your own.

Lots of weather stuff I can’t remember is covered in this post.

In general, at high angles of attack the center of pressure moves forward, while at low angles of attack, the CP moves aft. The relationship between CP and CG affects both aerodynamic balance and controllability.

Thrust is the force that imparts a change in the velocity of a mass. It may be measured in pounds, but it has no element of time or rate. The term “thrust required” is generally associated with jet engines. A forward force which propels the airplane through the air. Power implies work rate or units of work per unit of time, and as such, it is a function of the speed at which the force is developed. The term “power required” is generally associated with reciprocating engines.

Stalling speed: An aircraft’s stalling speed increases in proportion to the square root of the load factor.

The “design maneuvering speed” (VA), which is the speed below which you can move a single flight control, one time, to its full deflection, for one axis of airplane rotation only (pitch, roll or yaw), in smooth air, without risk of damage to the airplane.

When rolling out of a steep-banked turn, what causes the lowered aileron to create more drag than when rolling into the turn. The wing’s angle of attack is greater as the rollout is started.

Adverse Yaw Since the downward deflected aileron produces more lift as evidenced by the wing raising, it also produces more drag. This added drag causes the wing to slow down slightly. This results in the aircraft yawing toward the wing which had experienced an increase in lift (and drag). From the pilot’s perspective, the yaw is opposite the direction of the bank.

Dutch Roll: If the aircraft has a right wing pushed down, the positive sideslip angle corrects the wing laterally before the nose is realigned with the relative wind. As the wing corrects the position, a lateral directional oscillation can occur resulting in the nose of the aircraft making a figure eight on the horizon as a result of two oscillations (roll and yaw), which, although of about the same magnitude, are out of phase with each other.

Spiral instability. A condition that exists when the static directional stability of the airplane is very strong as compared to the effect of its dihedral in maintaining lateral equilibrium.

The propeller acts as a gyroscope. Precession is the resultant action, or deflection, of a spinning rotor when a deflecting force is applied to the rotor’s rim. Thus, as the airplane yaws arond its vertical axis, it results in a pitching moment about its lateral axis.

Effect of Weight on Flight Performance
The takeoff/climb and landing performance of an aircraft are determined on the basis of its maximum allowable takeoff and landing weights. A heavier gross weight results in a longer takeoff run, higher takeoff speed, and shallower climb. On landing, a faster touchdown speed and longer landing roll.

The aircraft stalls at a higher speed with a forward CG location.

The aircraft cruises faster with an aft CG location because of reduced drag. The drag is reduced because a smaller AOA and less downward deflection of the stabilizer are required to support the aircraft and overcome the nose-down pitching tendency. The aircraft becomes less stable as the CG is moved rearward.

The left engine on most light twins is the critical engine. This is due to multiengine airplanes being subject to P-factor, as are single-engine airplanes. The descending propeller blade of each engine will produce greater thrust than the ascending blade when the airplane is operated under power and at positive angles of attack. The descending propeller blade of the right engine is also a greater distance from the center of gravity, and therefore has a longer moment arm than the descending propeller blade of the left engine.

A common mistake students often make when performing turns is attempting to turn using only instrument references.

When landing with slight ballooning hold a constant angle of attack.

A demarcation bar delineates a runway with a displaced threshold from a blast pad, stopway, or taxiway that precedes the runway.

Separation, measured at the time the preceding aircraft is over the landing threshold, is provided to small aircraft: (a) Small landing behind heavy − 6 miles. (b) Small landing behind large, non−B757 − 4 miles.

Appropriate time or distance intervals are provided to departing aircraft when the departure will be from the same threshold… (a) Three minutes or the appropriate radar separation when takeoff will be behind a super aircraft; (b) Two minutes or the appropriate radar separation when takeoff will be behind a heavy aircraft.

Weather

The ceiling/sky condition, visibility, and obstructions to vision may be omitted from the ATIS broadcast if the ceiling is above 5,000 feet and the visibility is more than 5 miles.

A warm front describes a condition where warm air overrides cooler air. When relatively warm rain or drizzle falls through the cool air, evaporation from the precipitation saturates the cool air and produces fog. Q: Fog associated with a warm front is a result of saturation due to evaporation of precipitation. Q: Which in-flight hazard is mot commonly associated with warm fronts: Precipitation-induced fog.

Temperature and dew point converge at 4.4°F (2.5°C).

A moist warm air mass is being cooled from below is characterized by: smooth air.
A moist cold air mass is being warmed from below is characterized by: showers and thunderstorms.
Cool air moving over a warm surface is characterized by instability and showers.

If a moist air mass moves over a warmer surface, lifting action is induced, which results in instability, turbulence, and cumulus clouds. These are indications of an unstable air mass.

Cool air moving over a warm surface becomes warmed from below and is therefore an unstable air mass characterized by showery precipitation, good visibility, turbulent air, and cumuliform clouds.

Stable lapse rate, stratiform clouds, fog, smooth air, and poor visibility are characteristics of a moist air mass that is warmer than the surface over which it passes.

A cold front occlusion occurs when the air behind the cold front is colder than the air in advance of the warm front. This lifts the warm air aloft.

Frontal waves and cyclones (areas of low pressure) usually form on slow-moving cold fronts of stationary fronts.

Microbursts may have a maximum downdraft of 6,000 fpm, last 15 minutes from the time the burst strikes the ground until dissipation, and have a maximum intensity of 2 to 4 minutes.

Fog should be reported on a METAR when visibility is below 5/8 statute miles and mist should be reported when visibility is greater than 5/8 SM and less than 7 statute miles.

In a convective outlook chart, slight risk is 2-5%, moderate risk is 6-10%, high risk is 10-50%.
Slight risk means that risk of severe thunderstorms is indicated, but they will be small in numbers and/or low coverage of the affected area.

FARs

An applicant requires 3 hours of training in the previous 2 calendar months in preparation for a practical test. If all parts aren’t completed, the remaining must be competed in 60 days.

Solo Cross Country
For each cross-country flight, the authorized instructor who reviews the cross-country planning must make an endorsement in the person’s logbook after reviewing that person’s cross-country planning, as specified in paragraph (d) of this section. The endorsement must—
(i) Specify the make and model of aircraft to be flown;
(ii) State that the student’s preflight planning and preparation is correct and that the student is prepared to make the flight safely under the known conditions; and
(iii) State that any limitations required by the student’s authorized instructor are met.

A flight instructor must have 5 hours PIC time in the make and model of multiengine airplane, a helicopter, or a powered-lift.

Instruction by an ATP is limited to 36 hours in any 7-day period.
In any 24-consecutive-hour period, a flight instructor may not conduct more than 8 hours of flight training.

Prep for the Knowledge Test and Checkride: FAA-H-8083-25B Pilots Handbook of Aeronautical Knowledge

October 24th, 2018

I’m not very good at remembering things out of context or when they don’t make any sense to me. For example, some of the questions deal with excess power and thrust. Before I read the book, power and thrust were synonyms, so I had a difficult time answering the questions. But now that I understand the terms and how they relate to each other, remembering the answers is much easier. This post was intended to be a bunch of unrelated things that I can’t remember, but appear on the CFI and AGI knowledge tests, but the explanations all seemed to come from the Pilots Handbook of Aeronautical Knowledge so it turned into a post summarizing and expanding on things in that book instead.

Atmosphere
The standard atmosphere at sea level is a surface temperature of 59° F or 15° C and a surface pressure of 29.92 inches of mercury (“Hg) or 1,013.2 mb.

A standard temperature lapse rate is when the temperature decreases at the rate of approximately 3.5° F or 2° C (1.9833) per thousand feet up to 36,000 feet, which is approximately –65° F or –55° C. (You can see this in the winds aloft forecast where it is around this value across the country,) Above this point, the temperature is considered constant up to 80,000 feet. A standard pressure lapse rate is when pressure decreases at a rate of approximately 1 “Hg per 1,000 feet of altitude gain to 10,000 feet.

The dry adiabatic lapse rate (unsaturated air) is 3 °C (5.4 °F) per 1,000 feet. The moist adiabatic lapse rate varies from 1.1 °C to 2.8 °C (2 °F to 5 °F) per 1,000 feet. When lifted, unsaturated air cools at a rate of 5.4 °F per 1,000 feet and the dew point temperature decreases at a rate of 1 °F per 1,000 feet. This results in a convergence of temperature and dew point at a rate of 4.4 °F (1° C).

When the temperature of the air is reduced to the dew point, the air is completely saturated and moisture begins to condense out of the air in the form of fog, dew, frost, clouds, rain, or snow.

High density altitude refers to thin air, while low density altitude refers to dense air. The conditions that result in a high density altitude are high elevations, low atmospheric pressures, high temperatures, high humidity, or some combination of these factors. Lower elevations, high atmospheric pressure, low temperatures, and low humidity are more indicative of low density altitude.

Instruments
FAA-H-8083-25B Pilots Handbook of Aeronautical Knowledge Chapter 3
The control instruments display immediate attitude and power changes and are calibrated to permit adjustments in precise increments. They are the manifold pressure, tachometer (RPM), and attitude indicator.

The performance instruments indicate the aircraft’s actual performance. Performance is determined by reference to the altimeter, airspeed, vertical speed indicator (VSI), heading indicator, turn coordinator (or turn and bank indicator), and slip/skid indicator (ball).

Climb Performance
FAA-H-8083-25B Pilots Handbook of Aeronautical Knowledge Chapter 11

If an aircraft is to move, fly, and perform, work must act upon it. Work involves force moving the aircraft. The aircraft acquires mechanical energy [as opposed to chemical or atomic energy] when it moves. Mechanical energy comes in two forms: (1) Kinetic Energy (KE), the energy of speed; (2) Potential Energy (PE), the stored energy of position.

Aircraft motion (KE) is described by its velocity (airspeed). Aircraft position (PE) is described by its height (altitude). Both KE and PE are directly proportional to the object’s mass. KE is directly proportional to the square of the object’s velocity (airspeed). PE is directly proportional to the object’s height (altitude). The formulas below summarize these energy relationships:

m = object mass
v = object velocity

g = gravity field strength
h = object height

KE = ½ × m × v2
PE = m × g × h

We sometimes use the terms “power” and “thrust” interchangeably when discussing climb performance. This erroneously implies the terms are synonymous. It is important to distinguish between these terms. Thrust is a force or pressure exerted on an object. Thrust is measured in pounds (lb) or newtons (N). Power, however, is a measurement of the rate of performing work or transferring energy (KE and PE). Power is typically measured in horsepower (hp) or kilowatts (kw). We can think of power as the motion (KE and PE) a force (thrust) creates when exerted on an object over a period of time.

Positive climb performance occurs when an aircraft gains PE by increasing altitude. Two basic factors, or a combination of the two factors, contribute to positive climb performance in most aircraft:

1. The aircraft climbs (gains PE) using excess power above that required to maintain level flight, or
2. The aircraft climbs by converting airspeed (KE) to altitude (PE).

As an example of factor 1 above, an aircraft with an engine capable of producing 200 horsepower (at a given altitude) is using only 130 horsepower to maintain level flight at that altitude. This leaves 70 horsepower available to climb. The pilot holds airspeed constant and increases power to perform the climb.

As an example of factor 2, an aircraft is flying level at 120 knots. The pilot leaves the engine power setting constant but applies other control inputs to perform a climb. The climb, sometimes called a zoom climb, converts the airspeed (KE) to altitude (PE); the airspeed decreases to something less than 120 knots as the altitude increases.

Test Question
During a steady climb, the rate of climb depends on excess power. [During a steady climb (rate of climb constant) thrust equals drag. The rate then is determined by the power. As in Case 1 above, if more power than is needed for the airspeed is provided, the aircraft climbs. If less than the airspeed decreases.]

Angle of Climb (AOC)
FAA-H-8083-25 (2003) Pilots Handbook of Aeronautical Knowledge Chapter 3
Before the airplane begins to move, thrust must be exerted. It continues to move and gain speed until thrust and drag are equal. In order to maintain a constant airspeed, thrust and drag must remain equal, just as lift and weight must be equal to maintain a constant altitude. If in level flight, the engine power is reduced, the thrust is lessened, and the airplane slows down. As long as the thrust is less than the drag, the airplane continues to decelerate until its airspeed is insufficient to support it in the air.

Likewise, if the engine power is increased, thrust becomes greater than drag and the airspeed increases. As long as the thrust continues to be greater than the drag, the airplane continues to accelerate. When drag equals thrust, the airplane flies at a constant airspeed.

During straight-and level-flight when thrust is increased and the airspeed increases, the angle of attack must be decreased. That is, if changes have been coordinated, the airplane will still remain in level flight but at a higher speed when the proper relationship between thrust and angle of attack is established. If the angle of attack were not coordinated (decreased) with this increase of thrust, the airplane would climb.

FAA-H-8083-25B Pilots Handbook of Aeronautical Knowledge Chapter 5
One method to climb (have positive AOC performance) is to have excess thrust available. Essentially, the greater the force that pushes the aircraft upward, the steeper it can climb. Maximum AOC occurs at the airspeed and angle of attack (AOA) combination which allows the maximum excess thrust. The airspeed and AOA combination where excess thrust exists varies amongst aircraft types.

Test Question
During a steady climb, the angle of climb depends on excess thrust. [Here they are not referring to thrust greater than drag, rather they are referring to thrust required for steady level flight. Given the above text, I think they are really asking about the angle of attack—not angle of climb.]

Lift
Lift is the upward force on the wing acting perpendicular to the relative wind and perpendicular to the aircraft’s lateral axis. Lift is required to counteract the aircraft’s weight. In stabilized level flight, when the lift force is equal to the weight force, the aircraft is in a state of equilibrium and neither accelerates upward or downward. If lift becomes less than weight, the vertical speed will decrease. When lift is greater than weight, the vertical speed will increase. [The angle of attack will need to remain constant and airspeed will increase or decrease.]

Ground Effect
Ground effect also alters the thrust required versus velocity. Since induced drag predominates at low speeds, the reduction of induced drag due to ground effect will cause a significant reduction of thrust required (parasite plus induced drag) at low speeds. Due to the change in upwash, downwash, and wingtip vortices, there may be a change in position (installation) error of the airspeed system associated with ground effect. In the majority of cases, ground effect causes an increase in the local pressure at the static source and produces a lower indication of airspeed and altitude. Thus, an aircraft may be airborne at an indicated airspeed less than that normally required.

An aircraft leaving ground effect after takeoff encounters just the reverse of an aircraft entering ground effect during landing. The aircraft leaving ground effect will:
• Require an increase in AOA to maintain the same CL
• Experience an increase in induced drag and thrust required
• Experience a decrease in stability and a nose-up change in moment
• Experience a reduction in static source pressure and increase in indicated airspeed

Vortices
When the vortices of larger aircraft sink close to the ground (within 100 to 200 feet), they tend to move laterally over the
ground at a speed of 2 or 3 knots. A crosswind will decrease the lateral movement of the upwind vortex and increase the movement of the downwind vortex. Thus a light wind with a cross runway component of 1 to 5 knots could result in the upwind vortex remaining in the touchdown zone for a period of time and hasten the drift of the downwind vortex toward another runway.

Moment and Moment Arm
A study of physics shows that a body that is free to rotate will always turn about its CG. In aerodynamic terms, the mathematical measure of an aircraft’s tendency to rotate about its CG is called a “moment.” A moment is said to be equal to the product of the force applied and the distance at which the force is applied. (A moment arm is the distance from a datum [reference point or line] to the applied force.) For aircraft weight and balance computations, “moments” are expressed in terms of the distance of the arm times the aircraft’s weight, or simply, inch-pounds.

Stability is the inherent quality of an aircraft to correct for conditions that may disturb its equilibrium and to return to or to continue on the original flight path.

Stability in an aircraft affects two areas significantly:
Maneuverability—the quality of an aircraft that permits it to be maneuvered easily and to withstand the stresses imposed by maneuvers. It is governed by the aircraft’s weight, inertia, size and location of flight controls, structural strength, and powerplant. It too is an aircraft design characteristic.
Controllability—the capability of an aircraft to respond to the pilot’s control, especially with regard to flight path and attitude. It is the quality of the aircraft’s response to the pilot’s control application when maneuvering the aircraft, regardless of its stability characteristics.

Adverse Yaw
In a turn, the downward deflected aileron produces more lift as evidenced by the wing raising, it also produces more drag. This added drag causes the wing to slow down slightly. This results in the aircraft yawing toward the wing which had experienced an increase in lift (and drag). From the pilot’s perspective, the yaw is opposite the direction of the bank. The adverse yaw is a result of differential drag and the slight difference in the velocity of the left and right wings. Application of rudder in the direction of turn counters the adverse yaw.

Propellers
FAA-H-8083-25B Pilots Handbook of Aeronautical Knowledge Chapter 5
A propeller is a rotating airfoil that produces thrust through aerodynamic action. A high-pressure area is formed at the back of the propeller’s airfoil, and low pressure is produced at the face of the propeller, similar to the way lift is generated by an airfoil used as a lifting surface or wing. This pressure differential develops thrust from the propeller, which in turn pulls the airplane forward. Engines may be turned around to be pushers with the propeller at the rear.

There are two significant factors involved in the design of a propeller that impact its effectiveness. The angle of a propeller blade, as measured against the hub of the propeller, keeps the angle of attack (AOA) relatively constant along the span of the propeller blade, reducing or eliminating the possibility of a stall. The amount of lift being produced by the propeller is directly related to the AOA, which is the angle at which the relative wind meets the blade. The AOA continuously changes during the flight depending upon the direction of the aircraft.

The pitch is defined as the distance a propeller would travel in one revolution if it were turning in a solid. These two factors combine to allow a measurement of the propeller’s efficiency.

Blade angle, usually measured in degrees, is the angle between the chord of the blade and the plane of rotation and is measured at a specific point along the length of the blade. Because most propellers have a flat blade “face,” the chord line is often drawn along the face of the propeller blade. Pitch is not blade angle, but because pitch is largely determined by blade angle, the two terms are often used interchangeably. An increase or decrease in one is usually associated with an increase or decrease in the other. The pitch of a propeller may be designated in inches. A propeller designated as a “74–48” would be 74 inches in length and have an effective pitch of 48 inches. The pitch is the distance in inches, which the propeller would screw through the air in one revolution if there were no slippage.

The angle at which this air (relative wind) strikes the propeller blade is its AOA. The air deflection produced by this angle causes the dynamic pressure at the engine side of the propeller blade to be greater than atmospheric pressure, thus creating thrust. The shape of the blade also creates thrust because it is cambered like the airfoil shape of a wing. As the air flows past the propeller, the pressure on one side is less than that on the other. As in a wing, a reaction force is produced in the direction of the lesser pressure. The airflow over the wing has less pressure, and the force (lift) is upward. In the case of the propeller, which is mounted in a vertical instead of a horizontal plane, the area of decreased pressure is in front of the propeller, and the force (thrust) is in a forward direction. Aerodynamically, thrust is the result of the propeller shape and the AOA of the blade.

The blade angle is also an excellent method of adjusting the AOA of the propeller. On constant-speed propellers, the blade angle must be adjusted to provide the most efficient AOA at all engine and aircraft speeds. Lift versus drag curves, which are drawn for propellers as well as wings, indicate that the most efficient AOA is small, varying from +2° to +4°. The actual blade angle necessary to maintain this small AOA varies with the forward speed of the aircraft.

Since the efficiency of any machine is the ratio of the useful power output to the actual power input, propeller efficiency is the ratio of thrust horsepower to brake horsepower. Propeller efficiency varies from 50 to 87 percent, depending on how much the propeller “slips.” Propeller slip is the difference between the geometric pitch of the propeller and its effective pitch. Geometric pitch is the theoretical distance a propeller should advance in one revolution; effective pitch is the distance it actually advances. Thus, geometric or theoretical pitch is based on no slippage, but actual or effective pitch includes propeller slippage in the air.

Test Questions
Blade angle is defined as the angle between the chord line and the plane of rotation,
Propeller efficiency is the ratio of thrust horsepower to brake horsepower.
The distance a propeller advances in one rotation is effective pitch.
Propeller slip is the difference between the geometric pitch and effective pitch of the blade.

The reason a propeller is “twisted” is that the outer parts of the propeller blades, like all things that turn about a central point, travel faster than the portions near the hub. If the blades had the same geometric pitch throughout their lengths, portions near the hub could have negative AOAs while the propeller tips would be stalled at cruise speed. Twisting or variations in the geometric pitch of the blades permits the propeller to operate with a relatively constant AOA along its length when in cruising flight. Propeller blades are twisted to change the blade angle in proportion to the differences in speed of rotation along the length of the propeller, keeping thrust more nearly equalized along this length.

Usually 1° to 4° provides the most efficient lift/drag ratio, but in flight the propeller AOA of a fixed-pitch propeller varies—normally from 0° to 15°. This variation is caused by changes in the relative airstream, which in turn results from changes in aircraft speed. Thus, propeller AOA is the product of two motions: propeller rotation about its axis and its forward motion.

A constant-speed propeller automatically keeps the blade angle adjusted for maximum efficiency for most conditions encountered in flight. During takeoff, when maximum power and thrust are required, the constant-speed propeller is at a low propeller blade angle or pitch. The low blade angle keeps the AOA small and efficient with respect to the relative wind. At the same time, it allows the propeller to handle a smaller mass of air per revolution. This light load allows the engine to turn at high rpm and to convert the maximum amount of fuel into heat energy in a given time. The high rpm also creates maximum thrust because, although the mass of air handled per revolution is small, the rpm and slipstream velocity are high, and with the low aircraft speed, there is maximum thrust. After liftoff, as the speed of the aircraft increases, the constant speed propeller automatically changes to a higher angle (or pitch). Again, the higher blade angle keeps the AOA small and efficient with respect to the relative wind. The higher blade angle increases the mass of air handled per revolution. This decreases the engine rpm, reducing fuel consumption and engine wear, and keeps thrust at a maximum.

After the takeoff climb is established in an aircraft having a controllable-pitch propeller, the pilot reduces the power output of the engine to climb power by first decreasing the manifold pressure and then increasing the blade angle to lower the rpm.

At cruising altitude, when the aircraft is in level flight and less power is required than is used in takeoff or climb, the pilot again reduces engine power by reducing the manifold pressure and then increasing the blade angle to decrease the rpm. Again, this provides a torque requirement to match the reduced engine power. Although the mass of air handled per revolution is greater, it is more than offset by a decrease in slipstream velocity and an increase in airspeed. The AOA is still small because the blade angle has been increased with an increase in airspeed.

Power adjustments in the proper order:
• When power settings are being decreased, reduce manifold pressure before reducing rpm. If rpm is reduced before manifold pressure, manifold pressure automatically increases, possibly exceeding the manufacturer’s tolerances.
• When power settings are being increased, reverse the order—increase rpm first, then manifold pressure.

NTSB 830: Prep for the Knowledge Test and Checkride

October 10th, 2018

This regulation isn’t too long, so you should probably read it on the ecfr.gov website, but there are parts that show up on most of the exams, so they are summarized here.

Definitions
Aircraft accident means an occurrence associated with the operation of an aircraft which takes place between the time any person boards the aircraft with the intention of flight and all such persons have disembarked, and in which any person suffers death or serious injury, or in which the aircraft receives substantial damage. For purposes of this part, the definition of “aircraft accident” includes “unmanned aircraft accident”.

Civil aircraft means any aircraft other than a public aircraft. Public aircraft means an aircraft used only for the United States Government.

Fatal injury means any injury which results in death within 30 days of the accident.

Incident means an occurrence other than an accident, associated with the operation of an aircraft, which affects or could affect the safety of operations.

Operator means any person who causes or authorizes the operation of an aircraft, such as the owner, lessee, or bailee of an aircraft.

Serious injury means any injury which: (1) Requires hospitalization for more than 48 hours, commencing within 7 days from the date of the injury was received; (2) results in a fracture of any bone (except simple fractures of fingers, toes, or nose); (3) causes severe hemorrhages, nerve, muscle, or tendon damage; (4) involves any internal organ; or (5) involves second- or third-degree burns, or any burns affecting more than 5 percent of the body surface.

Substantial damage means damage or failure which adversely affects the structural strength, performance, or flight characteristics of the aircraft, and which would normally require major repair or replacement of the affected component.

Not considered “substantial damage” for the purpose of this part:
Engine failure or damage limited to an engine if only one engine fails or is damaged, bent fairings or cowling, dented skin, small punctured holes in the skin or fabric, ground damage to rotor or propeller blades, and damage to landing gear, wheels, tires, flaps, engine accessories, brakes, or wingtips.

Unmanned aircraft accident when the system is activated with the purpose of flight and the time that the system is deactivated at the conclusion of its mission, in which any person suffers death or serious injury or the aircraft has a maximum gross takeoff weight of 300 pounds or greater and sustains substantial damage.

Immediate Notification
The operator of any civil aircraft, or any public aircraft not operated by the Armed Forces or an intelligence agency of the United States, or any foreign aircraft shall immediately, and by the most expeditious means available, notify the nearest National Transportation Safety Board (NTSB) office,1 when:

An aircraft accident or any of the following listed serious incidents occur:
Flight control system malfunction or failure;

Inability of any required flight crewmember to perform normal flight duties as a result of injury or illness;

Failure of any internal turbine engine component that results in the escape of debris other than out the exhaust path;

In-flight fire;

Aircraft collision in flight;

Damage to property, other than the aircraft, estimated to exceed $25,000 for repair (including materials and labor) or fair market value in the event of total loss, whichever is less.

For large multiengine aircraft
• In-flight failure of electrical systems which requires the sustained use of an emergency bus powered by a back-up source such as a battery, auxiliary power unit, or air-driven generator to retain flight control or essential instruments;
• In-flight failure of hydraulic systems that results in sustained reliance on the sole remaining hydraulic or mechanical system for movement of flight control surfaces;
• Sustained loss of the power or thrust produced by two or more engines; and
• An evacuation of an aircraft in which an emergency egress system is utilized.

Release of all or a portion of a propeller blade from an aircraft, excluding release caused solely by ground contact;
A complete loss of information, excluding flickering, from more than 50 percent of an aircraft’s cockpit displays known as:
• Electronic Flight Instrument System (EFIS) displays;
• Engine Indication and Crew Alerting System (EICAS) displays;
• Electronic Centralized Aircraft Monitor (ECAM) displays; or
• Other displays of this type, which generally include a primary flight display (PFD), primary navigation display (PND), and other integrated displays;

Airborne Collision and Avoidance System (ACAS) resolution advisories issued when an aircraft is being operated on an instrument flight rules flight plan and compliance with the advisory is necessary to avert a substantial risk of collision between two or more aircraft.

Damage to helicopter tail or main rotor blades, including ground damage, that requires major repair or replacement of the blade(s);

Any event in which an operator, when operating an airplane as an air carrier at a public-use airport on land:
• Lands or departs on a taxiway, incorrect runway, or other area not designed as a runway; or
• Experiences a runway incursion that requires the operator or the crew of another aircraft or vehicle to take immediate corrective action to avoid a collision.

An aircraft is overdue and is believed to have been involved in an accident.

Preservation of Aircraft Wreckage, Mail, Cargo, and Records
The operator of an aircraft involved in an accident or incident is responsible for preserving to the extent possible any aircraft wreckage, cargo, and mail aboard the aircraft, and all records, including all recording mediums of flight, maintenance, and voice recorders, pertaining to the operation and maintenance of the aircraft and to the airmen until the Board takes custody thereof or a release is granted.

Such wreckage, mail, or cargo may not be disturbed or moved except to the extent necessary:
• To remove persons injured or trapped;
• To protect the wreckage from further damage; or
• To protect the public from injury.

Where it is necessary to move aircraft wreckage, mail or cargo, sketches, descriptive notes, and photographs shall be made, if possible, of the original positions and condition of the wreckage and any significant impact marks.

Reports and Statements To Be Filed
(a) Reports. The operator shall file a report on Board Form 6120.5 within 10 days after an accident, or after 7 days if an overdue aircraft is still missing. A report on an incident for which immediate notification is required shall be filed only as requested by an authorized representative of the Board.

Part 91 Maintenance: Prep for the Knowledge Test and Checkride

October 10th, 2018

The maintenance regulations in Part 91 are something that you will be tested on for all certificates. They are especially important if you are the owner/operator of an aircraft. Part 43 governs maintenance and preventive maintenance.

General
The owner or operator of an aircraft is primarily responsible for maintaining that aircraft in an airworthy condition, including compliance with Airworthiness Directives.

No person may perform maintenance, preventive maintenance, or alterations on an aircraft other than as prescribed in this subpart and other applicable regulations, including part 43 of this chapter.

Maintenance Required
Each owner or operator of an aircraft shall have that aircraft inspected; have discrepancies repaired; shall ensure that maintenance personnel make appropriate entries in the aircraft maintenance records indicating the aircraft has been approved for return to service; and shall have any inoperative instrument or item of equipment, permitted to be inoperative repaired, replaced, removed, or inspected at the next required inspection.

Operation After Maintenance
No person may carry any person (other than crewmembers) in an aircraft that has been maintained, rebuilt, or altered in a manner that may have appreciably changed its flight characteristics or substantially affected its operation in flight until an appropriately rated pilot with at least a private pilot certificate flies the aircraft, makes an operational check of the maintenance performed or alteration made, and logs the flight in the aircraft records.

The aircraft does not have to be flown as required above if, prior to flight, ground tests, inspection, or both show conclusively that the maintenance, preventive maintenance, rebuilding, or alteration has not appreciably changed the flight characteristics or substantially affected the flight operation of the aircraft.

Annual Inspections
No person may operate an aircraft unless, within the preceding 12 calendar months, it has had an annual inspection in accordance with part 43 and approved for return to service. An annual inspection must be conducted by an IA.

100 Hour Inspections
No person may operate an aircraft carrying any person (other than a crewmember) for hire, and no person may give flight instruction for hire in an aircraft which that person provides, unless within the preceding 100 hours of time in service the aircraft has received an annual or 100-hour inspection and been approved for return to service in accordance with part 43. A 100 hour inspection may be conducted by an A&P or IA.

The 100-hour limitation may be exceeded by not more than 10 hours while en route to reach a place where the inspection can be done. The excess time used to reach a place where the inspection can be done must be included in computing the next 100 hours of time in service.

There are lots of exceptions that do not generally apply to light aircraft. Experimental certificate and light-sport require an annual condition inspection performed once every 12 calendar months.

Light sport can be conducted by a certificated repairman (light-sport aircraft) with a maintenance rating, an appropriately rated mechanic, or an appropriately rated repair station.

The operating limitations on your homebuilt will include the following (or something similar):
No person shall operate this aircraft unless within the preceding 12 calendar months it has had a condition inspection performed in accordance with the scope and detail of appendix D to part 43, or other FAA-approved programs, and found to be in a condition for safe operation. The inspection can be performed by any licensed A&P mechanic, an FAA Approved Repair Station, or by the builder of the airplane provided the builder obtains a “Repairman’s Certificate” from the FAA.

Altimeter
No person may operate an airplane, or helicopter, in controlled airspace under IFR unless within the preceding 24 calendar months, each static pressure system, each altimeter instrument, and each automatic pressure altitude reporting system has been tested and inspected.

ATC Transponder
No persons may use an ATC transponder unless, within the preceding 24 calendar months, the ATC transponder has been tested and inspected and found to comply with regulations.

Maintenance Records
Except for work performed in accordance with the regulations governing Altimeters and Transponders, each registered owner or operator shall keep the following records until the work is repeated or superseded by other work or for 1 year after the work is performed.

Records of the maintenance, preventive maintenance, and alteration and records of the 100-hour, annual, progressive, and other required or approved inspections, as appropriate, for each aircraft (including the airframe) and each engine, propeller, rotor, and appliance of an aircraft must include—
• A description (or reference to data acceptable to the Administrator) of the work performed; and
• The date of completion of the work performed; and
• The signature, and certificate number of the person approving the aircraft for return to service.
• The total time in service of the airframe, each engine, each propeller, and each rotor.
• The current status of life-limited parts of each airframe, engine, propeller, rotor, and appliance.
• The time since last overhaul of all items installed on the aircraft which are required to be overhauled on a specified time basis.
• The current inspection status of the aircraft, including the time since the last inspection required by the inspection program under which the aircraft and its appliances are maintained.
• The current status of applicable airworthiness directives (AD) and safety directives including, for each, the method of compliance, the AD or safety directive number and revision date. If the AD or safety directive involves recurring action, the time and date when the next action is required.
• Copies of the forms prescribed by §43.9(d) of this chapter for each major alteration to the airframe and currently installed engines, rotors, propellers, and appliances.

The records specified in of this section shall be retained and transferred with the aircraft at the time the aircraft is sold.

Part 91: Prep for the CFI or AGI Knowledge Test and Checkride

October 10th, 2018

There are lots of regulations that you should memorize in order to be a safe and legal pilot. There are also lots of things that you can easily look up when you need the info. I wish I needed to know the maximum airspeeds in different airspaces, but the planes I fly don’t come near that limit. Unfortunately, many of those make easy to write test questions so they show up on lots of the FAA Knowledge Tests. This post contains things in Part 91 that will probably show up on the CFI or AGI test.

Responsibility and authority of the pilot in command.
The pilot in command of an aircraft is directly responsible for, and is the final authority as to, the operation of that aircraft.

In an in-flight emergency requiring immediate action, the pilot in command may deviate from any rule of this part to the extent required to meet that emergency.

Each pilot in command who deviates from a rule under paragraph (b) of this section shall, upon the request of the Administrator, send a written report of that deviation to the Administrator.

Airworthiness
The pilot in command of a civil aircraft is responsible for determining whether that aircraft is in condition for safe flight. The pilot in command shall discontinue the flight when unairworthy mechanical, electrical, or structural conditions occur. Note that: The owner or operator of an aircraft is primarily responsible for maintaining that aircraft in an airworthy condition, including compliance with Airworthiness Directives.

Alcohol or drugs
No person may act or attempt to act as a crewmember within 8 hours after the consumption of any alcoholic beverage;
While under the influence of alcohol;
While using any drug that affects the person’s faculties in any way contrary to safety; or
While having an alcohol concentration of 0.04 or greater in a blood or breath specimen.

Except in an emergency, no pilot of a civil aircraft may allow a person who appears to be intoxicated or who demonstrates by manner or physical indications that the individual is under the influence of drugs (except a medical patient under proper care) to be carried in that aircraft.

Preflight Action
Each pilot in command shall, before beginning a flight, become familiar with all available information concerning that flight.
For a flight under IFR or a flight not in the vicinity of an airport, weather reports and forecasts, fuel requirements, alternatives available if the planned flight cannot be completed, and any known traffic delays of which the pilot in command has been advised by ATC;
For any flight, runway lengths at airports of intended use, and takeoff and landing distance information.

Safety Belts
Crewmembers—During takeoff and landing, and while en route, each required flight crewmember shall—be at the crewmember station and keep the safety belt fastened. During takeoff and landing, keep their shoulder harness fastened unless there is no shoulder harness or the crewmember would be unable to perform required duties with the shoulder harness fastened.

The pilot in command of that aircraft ensures that each person on board is briefed on how to fasten and unfasten that person’s safety belt and, if installed, shoulder harness.

When moving on the surface, taking off, or landing the pilot in command of the aircraft ensures that each person on board has been notified to fasten their safety belt and, if installed, their shoulder harness.

Safety Pilot
Safety pilot must possess at least a private pilot certificate with category and class ratings appropriate to the aircraft being flown. [The safety pilot is a required crewmember so must possess an appropriate medical certificate. Basic Med would apply if the safety pilot acts as PIC for the entire flight and the flight conforms to the Basic Med rules. AOPA]

Operating Near Other Aircraft
No person may operate an aircraft so close to another aircraft as to create a collision hazard.
No person may operate an aircraft in formation flight except by arrangement with the pilot in command of each aircraft in the formation.
No person may operate an aircraft, carrying passengers for hire, in formation flight.

Right-of-way Rules
Vigilance shall be maintained by each person operating an aircraft so as to see and avoid other aircraft. When a rule of this section gives another aircraft the right-of-way, the pilot shall give way to that aircraft and may not pass over, under, or ahead of it unless well clear.

An aircraft in distress has the right-of-way over all other air traffic.
When aircraft are approaching each other head-on, or nearly so, each pilot of each aircraft shall alter course to the right.
An aircraft towing or refueling other aircraft has the right-of-way over all other engine-driven aircraft.

When aircraft of the same category are converging at approximately the same altitude (except head-on, or nearly so), the aircraft to the other’s right has the right-of-way. If the aircraft are of different categories:
Balloon –> Glider –> Airship –> a powered parachute, weight-shift-control aircraft, airplane, or rotorcraft.

Each aircraft that is being overtaken has the right-of-way and each pilot of an overtaking aircraft shall alter course to the right to pass well clear.

Aircraft, while on final approach to land or while landing, have the right-of-way over other aircraft in flight or operating on the surface, except that they shall not take advantage of this rule to force an aircraft off the runway surface which has already landed and is attempting to make way for an aircraft on final approach. When two or more aircraft are approaching an airport for the purpose of landing, the aircraft at the lower altitude has the right-of-way, but it shall not take advantage of this rule to cut in front of another which is on final approach to land or to overtake that aircraft.

Aircraft Speed
Below 10,000 feet MSL at an indicated airspeed of no more than 250 knots.
At or below 2,500 feet above the surface within 4 nautical miles of the primary airport of a Class C or Class D airspace area at an indicated airspeed of no more than 200 knots.
Underlying a Class B airspace area designated for an airport or in a VFR corridor designated through such a Class B airspace area, at an indicated airspeed of no more than 200 knots.
If the minimum safe airspeed for any particular operation is greater than the maximum speed prescribed in this section, the aircraft may be operated at that minimum speed.

Minimum Safe Altitudes
Except when necessary for takeoff or landing, no person may operate an aircraft below an altitude allowing, if a power unit fails, an emergency landing without undue hazard to persons or property on the surface.

Over congested areas. Over any congested area of a city, town, or settlement, or over any open air assembly of persons, an altitude of 1,000 feet above the highest obstacle within a horizontal radius of 2,000 feet of the aircraft.

Over other than congested areas. An altitude of 500 feet above the surface, except over open water or sparsely populated areas. In those cases, the aircraft may not be operated closer than 500 feet to any person, vessel, vehicle, or structure.

A powered parachute or weight-shift-control aircraft may be operated at less than the minimums over other than congested areas if the operation is conducted without hazard to persons or property on the surface.

Compliance with ATC Clearances and Instructions.
When an ATC clearance has been obtained, no pilot in command may deviate from that clearance unless an amended clearance is obtained, an emergency exists, or the deviation is in response to a traffic alert and collision avoidance system resolution advisory. However, except in Class A airspace, a pilot may cancel an IFR flight plan if the operation is being conducted in VFR weather conditions. When a pilot is uncertain of an ATC clearance, that pilot shall immediately request clarification from ATC.

Except in an emergency, no person may operate an aircraft contrary to an ATC instruction in an area in which air traffic control is exercised.

Each pilot in command who (though not deviating from a rule of this subpart) is given priority by ATC in an emergency, shall submit a detailed report of that emergency within 48 hours to the manager of that ATC facility, if requested by ATC.

Light Signals

Color and type of signal Meaning with respect to aircraft on the surface Meaning with respect to aircraft in flight
Steady green Cleared for takeoff Cleared to land.
Flashing green Cleared to taxi Return for landing (to be followed by steady green at proper time).
Steady red Stop Give way to other aircraft and continue circling.
Flashing red Taxi clear of runway in use Airport unsafe—do not land.
Flashing white Return to starting point on airport Not applicable.
Alternating red and green Exercise extreme caution Exercise extreme caution.

Communications with Control Towers in Class G and E
Unless otherwise authorized or required by ATC, no person may operate an aircraft to, from, through, or on an airport having an operational control tower unless two-way radio communications are maintained between that aircraft and the control tower. Communications must be established prior to 4 nautical miles from the airport, up to and including 2,500 feet AGL.

Each pilot of an airplane must make all turns of that airplane to the left unless the airport displays approved light signals or visual markings indicating that turns should be made to the right, in which case the pilot must make all turns to the right.

Departures from Class E
Each pilot of an aircraft must comply with any traffic patterns established for that airport.

Operations in Class D
Each person must establish two-way radio communications with the ATC facility (including foreign ATC in the case of foreign airspace designated in the United States) providing air traffic services prior to entering that airspace and thereafter maintain those communications while within that airspace. This includes departing from the airport.

From a satellite airport without an operating control tower, must establish and maintain two-way radio communications with the ATC facility having jurisdiction over the Class D airspace area as soon as practicable after departing.

Each pilot operating a large or turbine-powered airplane must enter the traffic pattern at an altitude of at least 1,500 feet above the elevation of the airport and maintain at least 1,500 feet until further descent is required for a safe landing.

Each pilot operating an airplane approaching to land on a runway served by a visual approach slope indicator must maintain an altitude at or above the glide path until a lower altitude is necessary for a safe landing.

Each pilot must comply with any departure procedures established for that airport by the FAA.

No person may, at any airport with an operating control tower, operate an aircraft on a runway or taxiway, or take off or land an aircraft, unless an appropriate clearance is received from ATC.

Operations in Class C
Each person must establish two-way radio communications with the ATC facility (including foreign ATC in the case of foreign airspace designated in the United States) providing air traffic services prior to entering that airspace and thereafter maintain those communications while within that airspace. Same with departing.

From a satellite airport without an operating control tower, must establish and maintain two-way radio communications with the ATC facility having jurisdiction over the Class C airspace area as soon as practicable after departing.

Transponder is required and after 2020-01-01, ADSB.

Operations in Class B
The operator must receive an ATC clearance from the ATC facility having jurisdiction for that area before operating an aircraft in that area.

Large turbine engine-powered airplane to or from a primary airport for which a Class B airspace area is designated must operate at or above the designated floors of the Class B airspace area while within the lateral limits of that area.

The pilot in command holds at least a private pilot certificate. Student, sport, and recreational pilots must have training and endorsement. Only private pilots can takeoff and land at Class B airports.

Communications and navigation equipment requirements.
For IFR operation. An operable VOR or TACAN receiver or an operable and suitable RNAV system; and
For all operations. An operable two-way radio capable of communications with ATC on appropriate frequencies for that Class B airspace area.
Transponder is required and after 2020-01-01, ADSB.

Operations in Class A
Instrument flight rules (IFR) and operations may be conducted only under an ATC clearance received prior to entering the airspace.
Each pilot must maintain two-way radio communications with ATC while operating in Class A airspace.
Transponder is required and after 2020-01-01, ADSB.

Fuel requirements for flight in VFR conditions
No person may begin a flight in an airplane under VFR conditions unless (considering wind and forecast weather conditions) there is enough fuel to fly to the first point of intended landing and, assuming normal cruising speed—
  During the day, to fly after that for at least 30 minutes; or
  At night, to fly after that for at least 45 minutes.

Basic VFR Weather Minimums
Refer to this post.

No person may take off or land an aircraft, or enter the traffic pattern of an airport, under VFR, within the lateral boundaries of the surface areas of Class B, Class C, Class D, or Class E airspace designated for an airport unless ground or flight visibility at that airport is at least 3 statute miles.

Except as provided in Special VFR, no person may operate an aircraft beneath the ceiling under VFR within the lateral boundaries of controlled airspace designated to the surface for an airport when the ceiling is less than 1,000 feet.

Special VFR operations may only be conducted with an ATC clearance and clear of clouds when flight visibility is at least 1 statute mile. Between sunrise and sunset the person being granted the ATC clearance meets the applicable requirements for instrument flight and the aircraft is equipped as required.

Helicopters have no visibility limit. At night pilot does not have to be IFR rated and the helicopter is not required to be IFR equipped.

Required Equipment
Refer to this post.

Emergency Locator Transmitters
Batteries used in the emergency locator transmitters required by paragraphs (a) and (b) of this section must be replaced (or recharged, if the batteries are rechargeable) when the transmitter has been in use for more than 1 cumulative hour; or when 50 percent of their useful life (or, for rechargeable batteries, 50 percent of their useful life of charge) has expired, as established by the transmitter manufacturer under its approval.

Each emergency locator transmitter must be inspected within 12 calendar months after the last inspection.

You may ferry an airplane with an inoperative emergency locator transmitter.
An aircraft while engaged in training operations conducted entirely within a 50-nautical mile radius of the airport from which such local flight operations began does not need an ELT.

An aircraft during any period for which the transmitter has been temporarily removed for inspection, repair, modification, or replacement, may be operated for no more than 90 days after the ELT is initially removed from the aircraft.

Minimum Equipment List
No person may take off an aircraft with inoperative instruments or equipment installed unless the aircraft has within it a letter of authorization, issued by the responsible Flight Standards office, authorizing operation of the aircraft under the Minimum Equipment List.

The approved Minimum Equipment List must provide for the operation of the aircraft with the instruments and equipment in an inoperable condition.

The following instruments and equipment may not be included in a Minimum Equipment List:
(1) Instruments and equipment that are either specifically or otherwise required by the airworthiness requirements under which the aircraft is type certificated and which are essential for safe operations under all operating conditions.
(2) Instruments and equipment required by an airworthiness directive to be in operable condition unless the airworthiness directive provides otherwise.

[The primary purpose of a minimum equipment list is to list the equipment that can be inoperative and still not affect the airworthiness of an aircraft.] [An MEL is a precise listing of instruments, equipment, and procedures that allows an aircraft to be operated under specific conditibns with inoperative equipment. AC 91-67]

A person may takeoff an aircraft in operations conducted under this part with inoperative instruments and equipment without an approved Minimum Equipment List provided the inoperative instruments and equipment are:
Removed from the aircraft, the cockpit control placarded, and the maintenance recorded; or deactivated and placarded “Inoperative.” If deactivation of the inoperative instrument or equipment involves maintenance, it must be accomplished and recorded in accordance with part 43 of this chapter; and
A determination is made by a pilot, who is certificated and appropriately rated under part 61 of this chapter, or by a person, who is certificated and appropriately rated to perform maintenance on the aircraft, that the inoperative instrument or equipment does not constitute a hazard to the aircraft.

Transponder
If a transponder is installed, it must be turned on while in controlled airspace. It must be inspected every 24 months. Exceptions exist for aircraft not originally certified with an electrical system, balloons, and gliders.

ATC authorized deviations
Requests for ATC authorized deviations at any time for: without operating automatic pressure altitude reporting equipment; inoperative transponder to the airport of ultimate destination, including any intermediate stops, or to proceed to a place where suitable repairs can be made or both.

At least one hour before the proposed operation if not equipped with a transponder.

Transponder Requirements
Airspace Altitude
Class A, B, C All
Above the ceiling of Class B or C and within lateral boundaries Below 10,000′ MSL
Within 30 nm of at least one airport in Class B Below 10,000′ MSL
Within Contiguous US Above 10,000′ MSL and above 2,500′ AGL
Controlled Airspace–if equipped and maintained. All
ADIZ All
DC Special Flight Rules Area All

ADSB
ADSB is required in the same airspace as transponders plus:
Class E airspace at and above 3,000 feet MSL over the Gulf of Mexico from the coastline of the United States out to 12 nautical miles.

Aerobatic Flight
No person may operate an aircraft in aerobatic flight over any congested area of a city, town, or settlement; over an open air assembly of persons; within the lateral boundaries of the surface areas of Class B, Class C, Class D, or Class E airspace designated for an airport; within 4 nautical miles of the center line of any Federal airway; below an altitude of 1,500 feet above the surface; or when flight visibility is less than 3 statute miles.

For the purposes of this section, aerobatic flight means an intentional maneuver involving an abrupt change in an aircraft’s attitude, an abnormal attitude, or abnormal acceleration, not necessary for normal flight.

Flight test areas.
No person may flight test an aircraft except over open water, or sparsely populated areas, having light air traffic.

Parachutes
Packed within the preceding 180 days, if its canopy, shrouds, and harness are composed exclusively of nylon, rayon, or other similar synthetic fiber or materials that are substantially resistant to damage from mold, mildew, or other fungi and other rotting agents propagated in a moist environment; or

Packed within the preceding 60 days, if any part of the parachute is composed of silk, pongee, or other natural fiber or materials not specified above.

Unless wearing a parachute, may not execute any intentional maneuver that exceeds a bank of 60 degrees relative to the horizon; or a nose-up or nose-down attitude of 30 degrees relative to the horizon.

Part 61 Subpart A: Prep for the CFI or AGI Knowledge Test and Checkride

October 9th, 2018

The Knowledge Test covers regulations and you will be asked about them in the oral portion of the checkride. Some of the stuff is common knowledge if you have passed a bunch of checkrides, but some of it you won’t remember after the test. This is my cheat sheet for the exams.

Definitions
Complex airplane means an airplane that has a retractable landing gear, flaps, and a controllable pitch propeller, including airplanes equipped with an engine control system consisting of a digital computer and associated accessories for controlling the engine and propeller, such as a full authority digital engine control.

Cross-country time includes a point of landing that was at least a straight-line distance of more than 50 nautical miles from the original point of departure; sport pilot certificate—more than 25nm; powered parachute privileges or a private pilot certificate with a powered parachute category rating—more than 15nm; rotorcraft 25nm.

Pilot time means that time in which a person—Serves as a required pilot flight crewmember, so this includes safety pilot; Receives training from an authorized instructor or gives training as an authorized instructor in an aircraft, full flight simulator, flight training device, or aviation training device.

Required Documents
Photo Identification
Pilot Certificate
Medical certificate—Except not required for glider category rating, a balloon class rating, or glider or balloon privileges;
U.S. driver’s license for sport pilot certificate, weight-shift-control aircraft category rating, or a powered parachute category.
May not use a driver’s license for sport pilot if a medical has been denied.

CFI Required Documents
Photo Identification
Pilot Certificate
Flight Instructor Certificate if conducting training
Medical Certificate if a required crewmember (e.g Private Pilot training, IFR training, BFR with pilot whose BFR is expired)

Flight Instructor may—
Give training required to qualify a person for solo flight and solo cross-country flight;
Endorse an applicant for a—
  Pilot certificate or rating issued under this part
  Flight instructor certificate or rating issued under this part
  Ground instructor certificate or rating issued under this part
    Although Ground Instructor doesn’t require endorsements unless you fail a knowledge test.
Endorse a pilot logbook to show training given
Endorse a logbook for solo operating privileges
Ground instructor may do all of these except endorse for solo. [However, the student’s authorized instructor must administer the pre-solo test.]

Instrument Rating
No person may act as pilot in command of a civil aircraft under IFR or in weather conditions less than the minimums prescribed for VFR flight unless that person holds the appropriate aircraft category, class, type (if required), and instrument rating on that person’s pilot certificate for any airplane, helicopter, or powered-lift being flown (or an ATP certificate for the category, class, and type).

For a glider, a pilot certificate with a glider category rating and an airplane instrument rating; or
For an airship, a commercial pilot certificate with a lighter-than-air category rating and airship class rating.

Certificates
Student pilot, Sport pilot, Recreational pilot, Private pilot, Commercial pilot, Airline transport pilot, Flight instructor, Ground instructor

Aircraft category ratings
Airplane, Rotorcraft, Glider, Lighter-than-air, Powered-lift, Powered parachute, Weight-shift-control.

Class Ratings
Airplane—Single-engine land, Multiengine land, Single-engine sea, Multiengine sea.
Rotorcraft—Helicopter, Gyroplane.
Lighter-than-air—Airship, Balloon.
Weight-shift-control aircraft—Weight-shift-control aircraft land, Weight-shift-control aircraft sea.
Powered parachute— Powered parachute land, Powered parachute sea.
Aircraft type ratings— Large aircraft other than lighter-than-air, Turbojet-powered airplanes, Other aircraft type ratings specified by the Administrator through the aircraft type certification procedures.

Instrument ratings apply to private and commercial pilot certificates only. ATP has privileges but not rating.
Instrument—Airplane, Instrument—Helicopter, Instrument—Powered-lift.

Flight Instructor Certificate Ratings
Aircraft category—Airplane, Rotorcraft, Glider, Powered-lift.
Airplane class— Single-engine, Multiengine.
Rotorcraft class— Helicopter, Gyroplane.
Instrument— Instrument—Airplane, Instrument—Helicopter, Instrument—Powered-lift.
Sport pilot.

Ground Instructor Certificate Ratings
Basic, Advanced, Instrument

Suspension or revocation.
Suspension—may not apply for any certificate, rating, or authorization during the period of suspension.
Revoked—may not apply for any certificate, rating, or authorization for 1 year after the date of revocation.

Drugs and Alcohol
A conviction for the violation of any Federal or State statute relating to the growing, processing, manufacture, sale, disposition, possession, transportation, or importation of narcotic drugs, marijuana, or depressant or stimulant drugs or substances is grounds for denial or suspension.

Provide a written report of each motor vehicle action to the FAA not later than 60 days after the motor vehicle action.

Light Sport Medical
U.S. driver’s license and comply with each restriction and limitation.
Have been found eligible for the issuance of at least a third-class airman medical certificate at the time of his or her most recent application (if the person has applied for a medical certificate and not have had his or her most recently issued medical certificate (if the person has held a medical certificate) suspended or revoked or most recent Authorization for a Special Issuance of a Medical Certificate withdrawn.

Not know or have reason to know of any medical condition that would make that person unable to operate a light-sport aircraft in a safe manner.

Basic Med
At any point after July 14, 2006, have held a medical certificate.
Complete the medical education course during the previous 24 months.
Receive a comprehensive medical examination from a State-licensed physician during the previous 48 months.
Most recently issued medical certificate— May include an authorization for special issuance; may be expired; and cannot have been suspended or revoked.

Acting as PIC
To serve as the pilot in command of an aircraft, a person must hold the appropriate category, class, and type rating (if a class or type rating is required) for the aircraft to be flown; or received training and an endorsement for solo flight in that aircraft from an authorized instructor.

Endorsements require that the pilot has received and logged ground and flight training and received a one-time endorsement in the pilot’s logbook.
Complex airplane, high performance airplane, pressurized aircraft (an aircraft that has a service ceiling or maximum operating altitude, whichever is lower, above 25,000 feet MSL), tailwheel airplanes.
Glider: ground-tow procedures, aerotow procedures, self-launch procedures.
Night vision goggle operations

Prerequisites for practical tests.
Knowledge test in past 24 months.
Received and logged 3 hours of training time within 2 calendar months preceding the month of application in preparation for the practical test.
Hold at least a third-class medical certificate (or Basic Med), if a medical certificate is required.
Age
  Student Pilots – 16 years of age for other than the operation of a glider or balloon.
  Student Pilots – 14 years of age for the operation of a glider or balloon.
  Private Pilot — 17
  Recreational Pilots – 17
  Sport Pilot – 17 years old (or 16 years old if you are applying to operate a glider or balloon).
  Commercial Pilot – 18
  Flight Instructor – 18
  Ground Instructor – 18
  ATP – 23 (21 if bachelors degree from 141 school)

Practical tests: Required aircraft and equipment.
Is of the category, class, and type, if applicable, for which the applicant is applying for a certificate or rating and has a standard airworthiness certificate or special airworthiness certificate in the limited, primary, or light-sport category.

An aircraft used for a practical test must have the equipment for each area of operation required for the practical test. No prescribed operating limitations that prohibit its use in any of the areas of operation required for the practical test. At least two pilot stations with adequate visibility for each person to operate the aircraft safely.

An aircraft used for a practical test must have engine power controls and flight controls that are easily reached and operable in a conventional manner by both pilots.

An examiner may waive some of these requirements see §61.45 for details.

Cheating on Knowledge Test
An applicant who the Administrator finds has committed an act prohibited by paragraph (a) of this section is prohibited, for 1 year after the date of committing that act, from applying for any certificate, rating, or authorization issued under this chapter; and applying for and taking any test under this chapter.

Retesting after failure.
An applicant for a knowledge or practical test who fails that test may reapply for the test only after the applicant has received the necessary training from an authorized instructor who has determined that the applicant is proficient to pass the test and an endorsement from an authorized instructor who gave the applicant the additional training.

An applicant for a flight instructor certificate with an airplane category rating or a glider category rating, who has failed the practical test due to deficiencies in instructional proficiency on stall awareness, spin entry, spins, or spin recovery must receive additional training and bring an aircraft to the retest that is certificated for spins and demonstrate satisfactory instructional proficiency on stall awareness, spin entry, spins, and spin recovery to an examiner during the retest.

Logging PIC Time—CFI
A certificated flight instructor may log pilot in command flight time for all flight time while serving as the authorized instructor in an operation if the instructor is rated to act as pilot in command of that aircraft. Or gives training as an authorized instructor in an aircraft, full flight simulator, flight training device, or aviation training device.

An authorized instructor may log instrument time when conducting instrument flight instruction in actual instrument flight conditions.

Logging training time
A person may log training time when that person receives training from an authorized instructor in an aircraft, full flight simulator, flight training device, or aviation training device.

Operations that require a medical certificate
No person who holds a medical certificate issued under part 67 of this chapter may act as pilot in command, or in any other capacity as a required pilot flight crewmember, while that person knows or has reason to know of any medical condition that would make the person unable to meet the requirements for the medical certificate necessary for the pilot operation or is taking medication or receiving other treatment for a medical condition that results in the person being unable to meet the requirements for the medical certificate necessary for the pilot operation.

Operations that do not require a medical certificate.
A person shall not act as pilot in command, or in any other capacity as a required pilot flight crewmember, while that person knows or has reason to know of any medical condition that would make the person unable to operate the aircraft in a safe manner.

Flight review.
Required every 24 months unless passed a pilot proficiency check or practical test for a pilot certificate, rating, or operating privilege. Or passed a practical test conducted by an examiner for the issuance of a flight instructor certificate, an additional rating on a flight instructor certificate, renewal of a flight instructor certificate, or reinstatement of a flight instructor certificate (and a few other cases).

A flight review consists of a minimum of 1 hour of flight training and 1 hour of ground training. The review must include: a review of the current general operating and flight rules of part 91 of this chapter; and a review of those maneuvers and procedures that, at the discretion of the person giving the review, are necessary for the pilot to demonstrate the safe exercise of the privileges of the pilot certificate.

Recent flight experience: Pilot in command.
No person may act as a pilot in command of an aircraft carrying passengers or of an aircraft certificated for more than one pilot flight crewmember unless that person has made at least three takeoffs and three landings within the preceding 90 days, and— (i) The person acted as the sole manipulator of the flight controls; and (ii) The required takeoffs and landings were performed in an aircraft of the same category, class, and type (if a type rating is required), and, if the aircraft to be flown is an airplane with a tailwheel, the takeoffs and landings must have been made to a full stop in an airplane with a tailwheel.

Night currency is the same except that the landings must be to a full stop.

IFR
Within the 6 calendar months preceding the month of the flight, that person performed and logged at least the following tasks and iterations in an airplane, powered-lift, helicopter, or airship, as appropriate, for the instrument rating privileges to be maintained in actual weather conditions, or under simulated conditions using a view-limiting device that involves having performed the following— (i) Six instrument approaches. (ii) Holding procedures and tasks. (iii) Intercepting and tracking courses through the use of navigational electronic systems.

A person may use time in a full flight simulator, flight training device, or aviation training device for satisfying instrument recency experience requirements provided a logbook or training record is maintained to specify the training device, time, and the content.

A person who has failed to meet the instrument experience requirements of this section for more than six calendar months may reestablish instrument currency only by completing an instrument proficiency check.

Change of address
The holder of a pilot, flight instructor, or ground instructor certificate who has made a change in permanent mailing address may not, after 30 days from that date, exercise the privileges of the certificate unless the holder has notified in writing the FAA.

Logging Time
A pilot is only required to log time required to establish recent flight experience and to qualify for a certificate, rating, or endorsement.

Proposal for the FAA Fundamentals of Instruction Knowledge Test

October 4th, 2018

The Fundamentals of Instruction Knowledge Test is based (according to the FAA) entirely on one document, Aviation Instructor’s Handbook FAA-H-8083-9A. I read it twice and took notes then tried answering questions from the test at King Schools. I scored really poorly on the practice tests. Most of the questions I got wrong—and lots of the ones I got right, had two answers that as far as I could tell, were equally correct. Here’s an example:


In developing a lesson, the instructor should organize explanations and demonstrations to help the student
 A.  achieve the desired learning outcome.
 B.  acquire a thorough understanding of the material presented.
 C.  acquire new concepts, generally progressing from the known to the unknown.

All three of these are correct, but you are supposed to pick the one the is, in their words, more correct.

Here’s another one where you need to guess which answer the test writer was thinking of:


Which is one of the major difficulties encountered in the construction of multiple-choice test items?
 B.  Keeping all responses approximately equal in length.
 C.  Inventing distractors which will be attractive to students lacking knowledge or understanding.

What’s ironic about this one is that one of the things they mention in discussing the difficulties of writing test questions is that often the correct answer is longer than the others. And in this case both ASA and Gleim agree that C is the correct answer.

Here’s one where the “correct” answer is wrong, but you need to memorize it anyway.


An advantage of e-learning includes
A. higher levels of mastery and retention. 
B. The instructor need not be actively involved with the student when using a form of e-learning.

I purchased the Gleim book and after studying the questions and answers I can consistently get 95-100% on the tests.

I wonder if anyone at the big schools would be interested in doing an experiment to test whether the Knowledge Tests are a test of knowledge or of test taking ability. There should be three groups. One group of aspiring CFIs who have their Instrument and Commercial ratings, but have not studied for the FOI test. One group who have studied the book and/or taken a course in the material, and one group that has exclusively studied one of the test prep books/courses.

I’m guessing that the test prep folks score in the 90s, the ones who read the material and the ones who haven’t read the book score about the same somewhere in the 70s. I have been taking the practice tests and scoring 90% or 100%. I just took the test and scored 84% (I usually score 94-98% on FAA tests). There were a couple of questions that I had never seen before but they were close to ones that I had studied. There were two questions that I had no idea what they were asking but I was confident that I had the correct answer on all the rest. When I reviewed the questions I got wrong, I couldn’t tell why my answers were incorrect.

Aviation Instructor’s Handbook FAA-H-8083-9A: Knowledge Test

October 1st, 2018

A major problem that I have with the FAA knowledge tests is that they often have multiple correct answers (or in the case of questions related to flight planning—no correct answer). Knowing which answer is correct often hinges on deciphering what was in the mind of the person who wrote the question or remembering which word was used in a list of items when the answers are all synonyms. These are some of the questions that I have to just memorize the answers to.

You can practice taking the test at:
King Schools
Exams4Pilots

The learning process may include verbal elements, conceptual elements, perceptual elements, emotional elements, and problem-solving elements all taking place at once. p. 2-17

Which factor affecting perceptions is based on the effectiveness of a properly planned training syllabus?
Time and opportunity.
A perception factor in which learning something is dependent on the student having the time to sense and relate current experiences in context with previous events. p. G-7

Factors That Affect Perception
• Physical organism
• Goals and values
• Self-concept
• Time and opportunity
• Element of threat

Which principle of learning creates a strong impression? You would think it would be intensity, since the definition literally matches the question—the quality of being extreme in degree; excessive. But it is primacy. They claim, without providing any evidence, that Primacy, the state of being first, often creates a strong, almost unshakable impression and underlies the reason an instructor must teach correctly the first time and the student must learn correctly the first time. p. 2-11

Integrated flight instruction is flight instruction during which students are taught to perform flight maneuvers both by outside visual references and by reference to flight instruments. p. 8-10 However, the answer to a question of integrated flight instruction is the student develops the habit of looking for other traffic, which is true but not mentioned in relation to this topic.

In the lecture method, the instructor delivers his knowledge via lectures to students who are more or less silent participants. Lectures are best used when an instructor wishes to convey a general understanding of a subject that students lack. Lectures are used for introduction of new subjects, summarizing ideas, showing relationships between theory and practice, and reemphasizing main points. p. 4-10
One advantage of a lecture is? Excellent when additional research is required. Why Gleim thinks this is the correct answer rather than Allows for maximum attainment of certain types of learning outcomes. is a mystery to me. Exams4Pilots agrees with me that the answer is Uses time economically.

The more effective way for an instructor to properly motivate students: Positive motivation is provided by the promise or achievement of rewards.

Responses that create a pleasurable return are called praise. I have no idea what this even means, but it is apparently the answer.

This question hinges on their definition of prepare.
The best way to prepare a student to perform a task is to: explain the purpose of the task, provide a clear step-by-step example. In my mind, you prepare someone before you start the task. Providing an example comes after preparing them.

When students are unable to see the benefits or purpose of a lesson, they will be less motivated. No idea why this is correct and not learn as quickly isn’t. I get this wrong every time it comes up.

Drops in motivation appear in several different ways. During these times, it is often helpful to remind students of their own stated goals for seeking aviation training.

Success in reducing stress associated with a crisis on the flight deck begins with assessing stress areas in one’s personal life. This isn’t mentioned anywhere in the book and the other answers make at least as much sense as this one.

I hate most of the acronyms that you need to memorize, this one in particular since it doesn’t show up in the FOI book. It is in AC60-22 Aeronautical Decision Making and FAA-G-8082-22 Remote Pilot Study Guide, and FAA-H-8083-25B Pilots Handbook of Aeronautical Knowledge.

DECIDE
1. Detect The decision maker detects the fact that change has occurred.
2. Estimate The decision maker estimates the need to counter or react to the change.
3. Choose The decision maker chooses a desirable outcome (in terms of success for the flight).
4. Identify The decision maker identifies actions which could successfully control the change.
5. Do The decision maker takes the necessary action.
6. Evaluate The decision maker evaluate the effect(s) of his action countering the change.

In evaluating student demonstrations of piloting ability, it is important for the flight instructor to keep the student informed of progress. The key word is evaluating. While the student is performing the instructor should remain silent and observe.

There is a question on five responsibilities. All of them are correct but they want:
Helping students learn, providing adequate instruction, demanding adequate standards of performance, emphasizing the positive, and ensuring aviation safety.

Aviation Instructor Responsibilities

Helping Students Learn
Providing Adequate Instruction
Training to Standards of Performance
Emphasizing the Positive
Minimizing Student Frustrations
Motivate students
Keep students informed
Approach students as individuals
Give credit when due
Criticize constructively
Be consistent
Admit errors

Here’s another one where they use synonyms and you need to memorize the word they use.

The three types of problem-based learning instruction are: Scenario-based training, the collaborative problem-solving method, and the case study method.

An effective scenario should not promote errors.

The e-learning answer is wrong, but memorize it anyway. The whole point of e-learning is that it does not require active involvement of the instructor and can be done on the student’s timeframe and schedule. Whether it results in higher levels of mastery is an open question—but I doubt it.
An advantage of e-learning includes higher levels of mastery and retention. The instructor need not be actively involved with the student when using a form of e-learning.

Asking students about problems or decisions that test the limits of their knowledge is an effective method to help students acquire knowledge.

The main concern in developing a lesson plan is the student.

A primary consideration in planning for student performance is the length of the practice session. This is one of the questions where none of the answers is correct, but this is what they want.

Instructional aids should be designed to cover the key points in a lesson.

A fact question would be answered based on memory or recall. It is characterized by rote learning but that is not a type of question.

There are only two types of learning transfer: Positive and Negative.

In the communication process, the communicator will be more successful in gaining and retaining the receiver’s attention by
being friendly and informative. using a varied communicative approach.

When students display the defense mechanism called aggression, they become visibly angry, upset, and childish. may refuse to participate in class activities. This isn’t one of the defense mechanisms, so I have no idea why this is a question, let alone why this is the answer.

Simulated complete loss of engine power by closing the throttle and announcing “simulated engine failure” is Correlation.

The proper sequence for the subparts of an introduction is attention, motivation, and overview.

Which method of presentation is desirable for teaching a skill such as ground school lesson on the flight computer?
Demonstration/performance.

Which is one of the major difficulties encountered in the construction of multiple-choice test items? Keeping all responses approximately equal in length. Inventing distractors which will be attractive to students lacking knowledge or understanding. The book clearly states that “Research of instructor-made tests reveals that, in general, correct alternatives are longer than incorrect ones.” p. B4 so I don’t know why that isn’t the answer.

The educational objective level of the psychomotor domain at which a student’s skill demonstrates new movement patterns and creativity is origination.

When teaching new material, the teaching process can be divided into which steps? Preparation, presentation, application, and review and evaluation.

Weather Conditions for Takeoff

September 26th, 2018

There are two FARs that govern takeoff. The first is §91.155 and quite clearly states that no one may take off under VFR (except at Class G airports) if the ceiling is less than 1,000′. You could ask for a Special VFR, but you might have to wait for IFR traffic to land and take off since they have priority. You won’t get it at Class B—in fact many (most?) have a notation on the chart saying that it is not allowed.

The second FAR is §91.175 (f) governs visibility in IFR operations and does not apply to Part 91 operations (which I assume most readers of this post are).

So assuming you want to depart VFR, you would need 1,000′ ceiling to depart, except for an airport in Class G airspace where the takeoff and landing require the same weather conditions as the airspace—1 mile visibility and clear of clouds. If you want to depart IFR, you can do so with 0 ceiling and 0 visibility. (Subject to any conditions in the Obstacle Departure Procedure §91.175 (f)(3)).

§91.155 Basic VFR weather minimums.

(c) Except as provided in §91.157, no person may operate an aircraft beneath the ceiling under VFR within the lateral boundaries of controlled airspace designated to the surface for an airport when the ceiling is less than 1,000 feet.

(d) Except as provided in §91.157 of this part [Special VFR—which you won’t get at a Class B airport], no person may take off or land an aircraft, or enter the traffic pattern of an airport, under VFR, within the lateral boundaries of the surface areas of Class B, Class C, Class D, or Class E airspace designated for an airport—

(1) Unless ground visibility at that airport is at least 3 statute miles; or

(2) If ground visibility is not reported at that airport, unless flight visibility during landing or takeoff, or while operating in the traffic pattern is at least 3 statute miles.

(e) For the purpose of this section, an aircraft operating at the base altitude of a Class E airspace area is considered to be within the airspace directly below that area.

§91.175 Takeoff and landing under IFR.

(f) Civil airport takeoff minimums. This paragraph applies to persons operating an aircraft under part 121, 125, 129, or 135 of this chapter.

(1) Unless otherwise authorized by the FAA, no pilot may takeoff from a civil airport under IFR unless the weather conditions at time of takeoff are at or above the weather minimums for IFR takeoff prescribed for that airport under part 97 of this chapter.

(2) If takeoff weather minimums are not prescribed under part 97 of this chapter for a particular airport, the following weather minimums apply to takeoffs under IFR:

(i) For aircraft, other than helicopters, having two engines or less—1 statute mile visibility.

(ii) For aircraft having more than two engines— 1⁄2 statute mile visibility.

(iii) For helicopters— 1⁄2 statute mile visibility.

(3) Except as provided in paragraph (f)(4) of this section, no pilot may takeoff under IFR from a civil airport having published obstacle departure procedures (ODPs) under part 97 of this chapter for the takeoff runway to be used, unless the pilot uses such ODPs or an alternative procedure or route assigned by air traffic control.

Aviation Instructor’s Handbook FAA-H-8083-9A: Lists

September 20th, 2018

The Fundamentals of Instruction test assumes that you have memorized a bunch of lists. Here are some of them.

Maslow’s Hierarchy

Physiological
These are biological needs. They consist of the need for air, food, water, and maintenance of the human body.

Security
Security needs are about keeping oneself from harm.

Belonging
Maslow states that people seek to overcome feelings of loneliness and alienation. This involves both giving and receiving love, affection, and the sense of belonging.

Esteem
Humans get esteem in two ways: internally or externally. Internally, a person judges himself or herself worthy by personally defined standards. High self-esteem results in self-confidence, independence, achievement, competence, and knowledge.

Cognitive and Aesthetic
In later years, Maslow added cognitive (need to know and understand) and aesthetic (the emotional need of the artist) needs to the pyramid.

Self-Actualization
When all of the foregoing needs are satisfied, then and only then are the needs for self-actualization activated. Maslow describes self-actualization as a person’s need to be and do that which the person was “born to do.”

Human Nature and Motivation

Douglas McGregor set out two opposing assumptions about human nature and motivation in 1960.
“Theory X” assumes that management’s role is to coerce and control employees because people need control and direction.

“Theory Y” and holds that work is as natural as play and rest. The average person does not inherently dislike work.

Defense Mechanisms

Repression is the defense mechanism whereby a person places uncomfortable thoughts into inaccessible areas of the unconscious mind.

Denial is a refusal to accept external reality because it is too threatening.

Compensation is a process of psychologically counterbalancing perceived weaknesses by emphasizing strength in other areas.

Through projection, an individual places his or her own unacceptable impulses onto someone else.

Rationalization is a subconscious technique for justifying actions that otherwise would be unacceptable.

In reaction formation a person fakes a belief opposite to the true belief because the true belief causes anxiety.

Fantasy occurs when a student engages in daydreams about how things should be rather than doing anything about how things are.

Displacement results in an unconscious shift of emotion, affect, or desire from the original object to a more acceptable, less threatening substitute.

Thorndike and the Laws Principles of Learning

Readiness
The basic needs of the learner must be satisfied before they are ready or capable of learning.

Effect
All learning involves the formation of connections and connections are strengthened or weakened according to the law of effect. Responses to a situation that are followed by satisfaction are strengthened; responses followed by discomfort are weakened, either strengthening or weakening the connection of learning.

Exercise
Practice strengthens the learning connection; disuse weakens it. Exercise is most meaningful and effective when a skill is learned within the context of a real world application.

Primacy
The state of being first, often creates a strong, almost unshakable impression and underlies the reason an instructor must teach correctly the first time and the student must learn correctly the first time.

Intensity
Real world applications (scenarios) that integrate procedures and tasks the learner is capable of learning make a vivid impression and he or she is least likely to forget the experience.

Recency
The principle of recency states that things most recently learned are best remembered.

Freedom
The principle of freedom states that things freely learned are best learned.

Requirement
The law of requirement states that “we must have something to obtain or do something.” It can be an ability, skill, instrument or anything that may help us to learn or gain something.

Domains of Learning

Cognitive Domain
The four practical learning levels are rote, understanding, application, and correlation.

Affective Domain
The affective domain addresses a learner’s emotions toward the learning experience. It includes feelings, values, enthusiasms, motivations, and attitudes.
Five levels: awareness, response, value, organizing, and integration.

Psychomotor Domain
The psychomotor domain is skill based and includes physical movement, coordination, and use of the motor-skill areas.

Characteristics of Learning

Purposeful
A Result of Experience
Multifaceted
Active Process

Learning Is Multifaceted

The learning process may include verbal elements, conceptual elements, perceptual elements, emotional elements, and problem-solving elements all taking place at once.

Stages of Skill Acquisition

Cognitive Stage
The best way to prepare the student to perform a task is to provide a clear, step-by-step example.

Associative Stage
As the storage of a skill via practice continues, the student learns to associate individual steps in performance with likely outcomes.

Automatic Response Stage
As procedures become automatic, less attention is required to carry them out, so it is possible to do other things simultaneously, or at least do other things more comfortably.

Types of Practice

Deliberate Practice
Practices specific areas for improvement and receives specific feedback after practice.

Blocked Practice
Practicing the same drill until the movement becomes automatic. While blocked practice enhances current performance, it does not improve either concept learning or retrieval from long-term memory.

Random Practice
Random practice mixes up the skills to be acquired throughout the practice session.

Summary of Instructor Actions

To help students acquire skills, the instructor should:
• Explain that the key to acquiring and improving any skill is continued practice.
• Monitor student practice of skills and provide immediate feedback.
• Avoid conversation and other distractions when students are practicing individual skills.
• Explain that learning plateaus are common and that continued practice leads to continued improvement.

Reducing Error

Learning and Practicing
Taking Time
Checking for Errors
Using Reminders
Developing Routines
Raising Awareness

Basic Elements of Communication

Source (sender, speaker, writer, encoder, transmitter, or instructor)
Symbols used in composing and transmitting the message (words or signs (model prop))
Receiver (listener, reader, decoder, or student)

Barriers to Effective Communication

Lack of Common Experience
Confusion Between the Symbol and the Symbolized Object
Overuse of Abstractions
Interference

Essence of Good Teaching

Good teachers:
select and organize worthwhile course material,
lead students to encode and integrate this material in memorable form,
ensure competence in the procedures and methods of a discipline,
sustain intellectual curiosity,
promote how to learn independently.

Instructor’s Code of Conduct

• Make safety the number one priority,
• Develop and exercise good judgment in making decisions,
• Recognize and manage risk effectively,
• Be accountable for his or her actions,
• Act with responsibility and courtesy,
• Adhere to prudent operating practices and personal operating parameters, and
• Adhere to applicable laws and regulations.

The Certificated Flight Instructor (CFI) needs to remember he or she is teaching a pilot who should:
• Seek proficiency in control of the aircraft,
• Use flight deck technology in a safe and appropriate way,
• Be confident in a wide variety of flight situations, and
• Be respectful of the privilege of flight.

Teaching Process

Preparation
Presentation
Application
Assessment.

Preparation of a Lesson

Performance-based objectives
Standards
Description of the Skill or Behavior
Conditions
Criteria

Presentation of a Lesson

Introduction
• attention
• motivation
• an overview of what is to be covered.
Development
past to present
simple to complex
known to unknown
most frequently used to least used.
Conclusion

Training Delivery Methods

Lecture Method
Teaching Lecture
Formal Versus Informal Lectures
Discussion Method
Guided Discussion Method

Problem-Based Learning

• Relate to the real world so students want to solve them.
• Require students to make decisions.
• Are open ended and not limited to one correct answer.
• Are connected to previously learned knowledge as well as new knowledge.
• Reflect lesson objective(s).
• Challenge students to think critically.

Teaching Higher Order Thinking Skills (HOTS)
Risk management, ADM, automation management, situational awareness, and Controlled Flight into Terrain (CFIT) awareness are the skills encompassed by HOTS.

Types of Problem-Based Instruction

Scenario-Based Training Method (SBT)
Collaborative Problem-Solving Method
Case Study Method

Demonstration-Performance Method

• Explanation
• Demonstration
• Student Performance
• Instructor Supervision
• Evaluation

Drill and Practice Method

Guidelines for Use of Instructional Aids

• Clearly establish the lesson objective.
• Gather the necessary data by researching for support material.
• Organize the material into an outline or a lesson plan.
• Select the ideas to be supported with instructional aids.

Assessment Terminology

Traditional assessment
Authentic assessment
Diagnostic assessments
Formative assessments
Summative assessments

General Characteristics of Effective Assessment

Objective
Flexible
Acceptable
Comprehensive
Constructive
Organized
Thoughtful
Specific

Traditional Assessment

Reliability
Validity
Usability
Objectivity
Comprehensiveness
Discrimination

Collaborative Assessment

Replay
Reconstruct
Reflect
Redirect

Maneuver or Procedure “Grades”

Describe—the student is able to describe the physical characteristics and cognitive elements of the scenario activities, but needs assistance to execute the maneuver or procedure successfully.
Explain—the student is able to describe the scenario activity and understand the underlying concepts, principles, and procedures that comprise the activity, but needs assistance to execute the maneuver or procedure successfully.
Practice—the student is able to plan and execute the scenario. Coaching, instruction, and/or assistance will correct deviations and errors identified by the instructor.
Perform—the student is able to perform the activity without instructor assistance. The student will identify and correct errors and deviations in an expeditious manner. At no time will the successful completion of the activity be in doubt.
Not observed—any event not accomplished or required.

Single-Pilot Resource Management (SRM) “Grades”

Explain—the student can verbally identify, describe, and understand the risks inherent in the flight scenario, but needs to be prompted to identify risks and make decisions.
Practice—the student is able to identify, understand, and apply SRM principles to the actual flight situation. Coaching, instruction, and/or assistance quickly corrects minor deviations and errors identified by the instructor. The student is an active decision maker.
Manage-Decide—the student can correctly gather the most important data available both inside and outside the flight deck, identify possible courses of action, evaluate the risk inherent in each course of action, and make the appropriate decision. Instructor intervention is not required for the safe completion of the flight.

Choosing an Effective Assessment Method

• Determine level-of-learning objectives.
• List indicators of desired behaviors.
• Establish criterion objectives.
• Develop criterion-referenced test items.

Critiques and Oral Assessments

An effective critique considers good as well as bad performance, the individual parts, relationships of the individual parts, and the overall performance.

Instructor/Student Critique
Student-Led Critique
Small Group Critique
Individual Student Critique by Another Student
Self-Critique
Written Critique
Oral Assessment

Characteristics of Effective Questions

• Apply to the subject of instruction.
• Be brief and concise, but also clear and definite.
• Be adapted to the ability, experience, and stage of training of the students.
• Center on only one idea (limited to who, what, when, where, how, or why, not a combination).
• Present a challenge to the students.

Answering Student Questions

• Be sure that you clearly understand the question before attempting to answer.
• Display interest in the student’s question and frame an answer that is as direct and accurate as possible.
• After responding, determine whether or not the student is satisfied with the answer.

Characteristics of a Well-Planned Lesson

After the objective is determined, the instructor must research the subject as it is defined by the objective. Once the research is complete, the instructor determines the method of instruction and identifies a useful lesson planning format. The decision of how to organize the lesson and the selection of suitable support material come next. The final steps include assembling training aids and writing the lesson plan outline. [Exam questions.]

Unity
Content
Scope
Practicality
Flexibility
Relation to course of training
Instructional steps-every lesson, when adequately developed, falls logically into the four steps of the teaching process: preparation, presentation, application, and review and evaluation.

Duties, Responsibilities, and Authority of the Aviation Instructor

1. Orient new learners to the SBT approach.
2. Help the learner become a confident planner and a critical evaluator of his or her own performance.
3. Help the learner understand the knowledge requirements present in real world applications.
4. Diagnose learning difficulties and help the individual overcome them.
5. Evaluate student progress and maintain appropriate records.
6. Provide continuous review of student learning.

Aviation Instructor Responsibilities

Helping Students Learn
Providing Adequate Instruction
Training to Standards of Performance
Emphasizing the Positive
Minimizing Student Frustrations
Motivate students
Keep students informed
Approach students as individuals
Give credit when due
Criticize constructively
Be consistent
Admit errors

Professionalism

Sincerity
Acceptance of the Student
Personal Appearance and Habits
Demeanor
Proper Language

Practical Flight Instructor Strategies

The flight instructor should demonstrate good aviation sense at all times:
• Before the flight—discuss safety and the importance of a proper preflight and use of the checklist.
• During flight—prioritize the tasks of aviating, navigating, and communicating. Instill importance of “see and avoid” in the student.
• During landing—conduct stabilized approaches, maintain desired airspeed on final, demonstrate good judgment for go-arounds, wake turbulence, traffic, and terrain avoidance. Use ADM to correct faulty approaches and landing errors. Make power-off, stall-warning blaring, on centerline touchdowns in the first third of runway.
• Always—remember safety is paramount.

Obstacles to Learning During Flight Instruction

• Feeling of unfair treatment
• Impatience to proceed to more interesting operations
• Worry or lack of interest
• Physical discomfort, illness, fatigue, and dehydration
• Apathy due to inadequate instruction
• Anxiety

Acute fatigue

• Inattention
• Distractibility
• Errors in timing
• Neglect of secondary tasks
• Loss of accuracy and control
• Lack of awareness of error accumulation
• Irritability

Chronic fatigue

Physiological problems
Psychological issues

Warning Signs of Fatigue

Eyes going in and out of focus
Head bobs involuntarily
Persistent yawning
Spotty short-term memory
Wandering or poorly organized thoughts
Missed or erroneous performance of routine procedures
Degradation of control accuracy

Demonstration-Performance Training Delivery Method

Explanation Phase
Demonstration Phase
Student Performance and Instructor Supervision Phases
Evaluation Phase

The Telling-and-Doing Technique

• The flight instructor gives a carefully planned demonstration of the procedure or maneuver with accompanying verbal explanation.
• Student Tells—Instructor Does
• Student Tells—Student Does

Recognizing Hazardous Attitudes

Attitude: “Description” -> Antidote
Macho: “I can do it.” -> Taking chances is foolish.
Anti-authority” “Don’t tell me.” -> Follow the rules. They are usually right.
Impulsivity: “Do it quickly.” -> Not so fast. Think first.
Invulnerability: “It won’t happen to me.” -> It could happen to me.
Resignation: “What’s the use?” -> I’m not helpless. I can make a difference.

Defining Risk Management

Risk is defined as the probability and possible severity of accident or loss from exposure to various hazards, including injury to people and loss of resources.

Hazard—a present condition, event, object, or circumstance that could lead to or contribute to an unplanned or undesired event, such as an accident.
Risk—the future impact of a hazard that is not controlled or eliminated. It is the possibility of loss or injury. The level of risk is measured by the number of people or resources affected (exposure); the extent of possible loss (severity); and likelihood of loss (probability).
Safety—freedom from those conditions that can cause death, injury, occupational illness, or damage to or loss of equipment or property, or damage to the environment. Therefore, safety is a relative term that implies a level of risk that is both perceived and accepted.

Principles of Risk Management

Accept No Unnecessary Risk
Make Risk Decisions at the Appropriate Level
Accept Risk When Benefits Outweigh the Costs
Integrate Risk Management Into Planning at All Levels

Risk Management Process

Risk management is a simple process which identifies operational hazards and takes reasonable measures to reduce risk to personnel, equipment, and the mission.

Step 1: Identify the Hazard
Step 2: Assess the Risk
Step 3: Analyze Risk Control Measures
1. Probability of occurrence
2. Severity of the hazard
Step 4: Make Control Decisions
Step 5: Implement Risk Controls
Step 6: Supervise and Review

Implementing the Risk Management Process

• Apply the steps in sequence—each step is a building block for the next
• Allocate the time and resources to perform all steps in the process.
• Apply the process in a cycle—the “supervise and review” step should include a brand new look at the operation being analyzed to see whether new hazards can be identified.
• Involve people in the process—the people who are actually exposed to risks usually know best what works and what does not.

Level of Risk

The level of risk posed by a given hazard is measured in terms of:
• Severity (extent of possible loss)
• Probability (likelihood that a hazard will cause a loss)

IMSAFE Checklist

1. Illness—Am I sick? Illness is an obvious pilot risk.
2. Medication—Am I taking any medicines that might affect my judgment or make me drowsy?
3. Stress—Stress causes concentration and performance problems. While the regulations list medical conditions that require grounding, stress is not among them.
4. Alcohol—Have I been drinking within 8 hours? Within 24 hours?
5. Fatigue—Am I tired and not adequately rested? Fatigue continues to be one of the most insidious hazards to flight safety, as it may not be apparent to a pilot until serious errors are made.
6. Emotions-A pilot who experiences an emotionally upsetting event should refrain from flying until the pilot has satisfactorily recovered.

The PAVE Checklist

Pilot in command (PIC), Aircraft, enVironment, and External pressures

Three-P Model for Pilots

Risk management is a decision-making process designed to perceive hazards systematically, assess the degree of risk associated with a hazard, and determine the best course of action.

• Perceives the given set of circumstances for a flight.
• Processes by evaluating the impact of those circumstances on flight safety.
• Performs by implementing the best course of action.

SRM and the 5P Check

• The Plan
• The Plane
• The Pilot,
• The Passengers
• The Programming

Teaching Decision-Making Skills

It is also important for the flight instructor to remember that a good scenario:
• Is not a test.
• Will not have a single correct answer.
• Does not offer an obvious answer.
• Engages all three learning domains.
• Is interactive.
• Should not promote errors.
• Should promote situational awareness and opportunities for decision-making.
• Requires time-pressured decisions.

Aviation Instructor’s Handbook FAA-H-8083-9A: Glossary

September 19th, 2018

I’ve highlighted the definitions that are on the test and/or that I have trouble remembering.

Abstractions.  Words that are general rather than specific. Aircraft is an abstraction; airplane is less abstract; jet is more specific; and jet airliner is still more specific.

Aeronautical decision-making (ADM).  A systematic approach to the mental process used by aircraft pilots to consistently determine the best course of action in response to a given set of circumstances.

Affective domain.  A grouping of levels of learning associated with a person’s attitudes, personal beliefs, and values which range from receiving through responding, valuing, and organization to characterization.

Air traffic control (ATC).  A service provided by the FAA to promote the safe, orderly, and expeditious flow of air traffic.

Aircraft checkouts.  An instructional program designed to familiarize and qualify a pilot to act as pilot in command of a particular aircraft type.

Anxiety.  Mental discomfort that arises from the fear of anything, real or imagined. May have a potent effect on actions and the ability to learn from perceptions.

Application.  A basic level of learning at which the student puts something to use that has been learned and understood.

Application step.  The third step of the teaching process, where the student performs the procedure or demonstrates the knowledge required in the lesson. In the telling-and-doing technique of flight instruction, this step consists of the student doing the procedure while explaining it.

Area of operation.  A phase of the practical test within the PTS.

ATC.  See air traffic control.

Attitude.  A personal motivational predisposition to respond to persons, situations, or events in a given manner that can, nevertheless, be changed or modified through training as a sort of mental shortcut to decision-making.

Attitude management.  The ability to recognize one’s own hazardous attitudes and the willingness to modify them as necessary through the application of appropriate antidotal thoughts.

Authentic assessment.  An assessment in which the student is asked to perform real-world tasks, and demonstrate a meaningful application of skills and competencies.

Basic need.  A perception factor that describes a person’s ability to maintain and enhance the organized self.

Behaviorism.  Theory of learning that stresses the importance of having a particular form of behavior reinforced by someone other than the student to shape or control what is learned.

Bookmark.  A means of saving addresses on the World Wide Web (WWW) for easy future access. Usually done by selecting a button on the web browser screen, it saves the current web address so it does not have to be input again in a lengthy series of characters.

Branching.  A programming technique which allows users of interactive video, multimedia courseware, or online training to choose from several courses of action in moving from one sequence to another.

Briefing.  An oral presentation where the speaker presents a concise array of facts without inclusion of extensive supporting material.

Building block concept.  Concept of learning that new knowledge and skills are best based on a solid foundation of previous experience and/or old learning. As knowledge and skills increase, the base expands, supporting further learning.

CBI.  See computer-based instruction.

CBT.  See computer-based training.

CD.  See compact disk.

Cognitive domain.  A grouping of levels of learning associated with mental activity. In order of increasing complexity, the domains are knowledge, comprehension, application, analysis, synthesis, and evaluation.

Compact disk (CD).  A small plastic optical disk which contains recorded music or computer data. Also, a popular format for storing information digitally. The major advantage of a CD is its capability to store enormous amounts of information.

Comprehensiveness.  The degree to which a test measures the overall objective.

Computer-assisted instruction.  Instruction in which the instructor is responsible for the class and uses the computer to assist in the instruction.

Computer-based training (CBT).  The use of the computer as a training device. CBT is sometimes called computer-based instruction (CBI); the terms and acronyms are synonymous and may be used interchangeably.

Condition.  The second part of a performance-based objective which describes the framework under which the skill or behavior will be demonstrated.

Confusion between the symbol and the symbolized object.  Results when a word is confused with what it is meant to represent. Words and symbols create confusion when they mean different things to different people.

Cooperative or group learning.  An instructional strategy which organizes students into small groups so that they can work together to maximize their own and each other’s learning.

Correlation.  A basic level of learning where the student can associate what has been learned, understood, and applied with previous or subsequent learning.

Course of training.  A complete series of studies leading to attainment of a specific goal, such as a certificate of completion, graduation, or an academic degree.

Crew resource management (CRM).  The application of team management concepts in the flight deck environment. It was initially known as cockpit resource management, but as CRM programs evolved to include cabin crews, maintenance personnel and others, the phrase “crew resource management” has been adopted. This includes single pilots, as in most general aviation aircraft. Pilots of small aircraft, as well as crews of larger aircraft, must make effective use of all available resources; human resources, hardware, and information. A current definition includes all groups routinely working with the cockpit crew who are involved in decisions required to operate a flight safely. These groups include, but are not limited to: pilots, dispatchers, cabin crewmembers, maintenance personnel, and air traffic controllers. CRM is one way of addressing the challenge of optimizing the human/machine interface and accompanying interpersonal activities.

Criteria.  The third part of a performance-based objective, descriptions of standards that will be used to measure the accomplishment of the objective.

Criterion-referenced testing.  System of testing where students are graded against a carefully written, measurable standard or criterion rather than against each other.

CRM.  See crew resource management.

Curriculum.  A set of courses in an area of specialization offered by an educational institution. A curriculum for a pilot school usually includes courses for the various pilot certificates and ratings.

Cut-away.  Model of an object that is built in sections so it can be taken apart to reveal the inner structure.

Defense mechanisms.  Subconscious ego-protecting reactions to unpleasant situations.

Demonstration-performance method.  An educational presentation where an instructor first shows the student the correct way to perform an activity and then has the student attempt the same activity.

Description of the skill or behavior.  The first part of a performance-based objective which explains the desired outcome of instruction in concrete terms that can be measured.

Determiners.  In test items, words which give a clue to the answer. Words such as “always” and “never” are determiners in true-false questions. Since absolutes are rare, such words usually make the statement false.

Direct question.  A question used for follow-up purposes, but directed at a specific individual.

Discrimination.  The degree to which a test distinguishes the differences between students.

Distractors.  Incorrect responses to a multiple-choice test item.

Disuse.  A theory of forgetting that suggests a person forgets those things that are not used.

Drill and practice method.  A time-honored training delivery method based on the learning principle that connections are strengthened with practice.

Effect.  A principle of learning that learning is strengthened when accompanied by a pleasant or satisfying feeling, and that learning is weakened when associated with an unpleasant feeling.

Electronic learning (e-learning).  Any type of education that involves an electronic component such as the Internet, a network, a stand-alone computer, CD/DVDs, video conferencing, websites, or e-mail in its delivery.

Element of threat.  A perception factor that describes how a person is unlikely to easily comprehend an event if that person is feeling threatened since most of a person’s effort is focused on whatever is threatening them.

Exercise.  A principle of learning emphasizing that those things most often repeated are best remembered.

FAASTeam.  See Federal Aviation Administration Safety Team.

FAASTeam Program Manager.  The person who designs, implements, and evaluates the FAASTeam within the FAA flight standards district office (FSDO) area of responsibility.

FAASTeam Representative.  A volunteer within the aviation community who shares technical expertise and professional knowledge as a part of the FAASTeam.

Federal Aviation Administration Safety Team (FAASTeam).  An organization promoting safety standards and the reduction of aircraft related accidents. Each of the eight F A A Flight Standards regions have a dedicated FAASTeam office.

Flight review.  A 14 CFR 61.56 requirement designed to assess and update a pilot’s knowledge and skills.

Flight training devices (FTDs).  A full-size replica of the instruments, equipment, panels, and controls of an aircraft, or set of aircraft, in an open flight deck area or in an enclosed cockpit. A force (motion) cueing system or visual system is not required.

Follow-up question.  In the guided discussion method, a question used by an instructor to get the discussion back on track or to get the students to explain something more thoroughly.

Formal lecture.  An oral presentation where the purpose is to inform, persuade, or entertain with little or no verbal participation by the listeners.

FTD.  See flight training device.

Goals and values.  A perception factor that describes how a person’s perception of an event depends on beliefs. Motivation toward learning is affected by how much value a person puts on education. Instructors who have some idea of the goals and values of their students will be more successful in teaching them.

Guided discussion method.  An educational presentation typically used in the classroom where the topic to be covered by a group is introduced and the instructor participates only as necessary to keep the group focused on the subject.

Hierarchy of human needs.  A listing by Abraham Maslow of needs, from the most basic to the most fulfilling: physiological, security, belonging, esteem, cognitive and aesthetic, and self-actualization.

Human factors.  A multidisciplinary field devoted to optimizing human performance and reducing human error. It incorporates the methods and principles of the behavioral and social sciences, engineering, and physiology. It may be described as the applied science which studies people working together in concert with machines. Human factors involve variables that influence individual performance, as well as team or crew performance.

Human nature.  The general psychological characteristics, feelings, and behavioral traits shared by all humans.

Illustrated talk.  An oral presentation where the speaker relies heavily on visual aids to convey ideas to the listeners.

Insight.  The grouping of perceptions into meaningful wholes. Creating insight is one of the instructor’s major responsibilities.

Instructional aids.  Devices that assist an instructor in the teaching-learning process. They are supplementary training devices, and are not self-supporting.

Instrument proficiency check.  An evaluation ride based on the instrument rating practical test standard which is required to regain instrument flying privileges when the privileges have expired due to lack of currency.

Integrated flight instruction.  A technique of flight instruction in which students are taught to perform flight maneuvers by reference to both the flight instruments and to outside visual references from the time the maneuver is first introduced. Handling of the controls is the same regardless of whether flight instruments or outside references are being used.

Intensity.  A principle of learning in which a dramatic or exciting learning experience is likely to be remembered longer than a boring experience. Students experiencing the real thing will learn more than when they are merely told about the real thing.

Interactive video.  Software that responds quickly to certain choices and commands by the user. A typical system consists of a compact disk, computer, and video technology.

Interference.  (1) A theory of forgetting proposing that a person forgets something because a certain experience overshadows it, or the learning of similar things has intervened. (2) Barriers to effective communication that are caused by physiological, environmental, and psychological factors outside the direct control of the instructor. The instructor must take these factors into account in order to communicate effectively.

Internet.  An electronic network that connects computers around the world.

Judgment.  The mental process of recognizing and analyzing all pertinent information in a particular situation, a rational evaluation of alternative actions in response to it, and a timely decision on which action to take.

Knowledge.  Information that humans are consciously aware of and can articulate.

Lack of common experience.  In communication, a difficulty which arises because words have different meanings for the source and the receiver of information due to their differing backgrounds.

Lead-off question.  In the guided discussion method, a question used by an instructor to open up an area for discussion and get the discussion started.

Learning.  A change in behavior as a result of experience.

Learning plateau.  A learning phenomenon where progress appears to cease or slow down for a significant period of time before once again increasing.

Learning style.  Preferred way(s) by which people learn. Common learning styles include visual, auditory, and kinesthetic, or tactile (hands on). Learning skills can be loosely grouped into physical and cognitive styles.

Learning theory.  A body of principles advocated by psychologists and educators to explain how people acquire skills, knowledge, and attitudes.

Lecture method.  An educational presentation usually delivered by an instructor to a group of students with the use of instructional aids and training devices. Lectures are useful for the presentation of new material, summarizing ideas, and showing relationships between theory and practice.

Lesson plan.  An organized outline for a single instructional period. It is a necessary guide for the instructor in that it tells what to do, in what order to do it, and what procedure to use in teaching the material of a lesson.

Link.  On a website, an external web location that can be accessed by merely clicking on words identifying the new site. They are usually identified by a different color type, underlining, or a button (picture or icon) indicating access to a new site.

Long-term memory.  The portion of the brain that stores information that has been determined to be of sufficient value to be retained. In order for it to be retained in longterm memory, it must have been processed or coded in the working memory.

Matching-type test item.  A test item in which the student is asked to match alternatives on one list to related alternatives on a second list. The lists may include words, terms, illustrations, phrases, or sentences.

Memory.  The ability of people and other organisms to encode (initial perception and registration of information), store (retention of encoded information over time), and retrieve (processes involved in using stored information) information.

Mock-up.  A three-dimensional working model used in which the actual object is either unavailable or too expensive to use. Mock-ups may emphasize some elements while eliminating nonessential elements.

Model.  A copy of a real object which can be life-size, smaller, or larger than the original.

Motivation.  A need or desire that causes a person to act. Motivation can be positive or negative, tangible or intangible, subtle or obvious.

Multimedia.  A combination of more than one instructional medium. This format can include audio, text, graphics, animations, and video. Recently, multimedia implies a computer-based presentation.

Multiple-choice-type test item.  A test item consisting of a question or statement followed by a list of alternative answers or responses.

Navigate.  To move between websites on the internet. Navigation is often accomplished by means of links or connections between sites.

Norm-referenced testing.  System of testing in which students are ranked against the performance of other students.

Objectivity.  The singleness of scoring of a test; it does not reflect the biases of the person grading the test.

Overhead question.  In the guided discussion method, a question directed to the entire group in order to stimulate thought and discussion from the entire group. An overhead question may be used by an instructor as the lead-off question.

Perceptions.  The basis of all learning, perceptions result when a person gives meaning to external stimuli or sensations. Meaning derived from perception is influenced by an individual’s experience and many other factors.

Performance-based objectives.  A statement of purpose for a lesson or instructional period that includes three elements: a description of the skill or behavior desired of the student, a set of conditions under which the measurement will be taken, and a set of criteria describing the standard used to measure accomplishment of the objective.

Personal computer-based aviation training device (PCATD).  A device which uses software which can be displayed on a personal computer to replicate the instrument panel of an airplane. A PCATD must replicate a type of airplane or family of airplanes and meet the virtual control requirements specified in AC 61-126.

Personality.  The embodiment of personal traits and characteristics of an individual that are set at a very early age and are extremely resistant to change.

Physical organism.  A perception factor that describes a person’s ability to sense the world around them.

Pilot error.  Pilot action/inaction or decision/indecision causing or contributing to an accident or incident.

Poor judgment chain.  A series of mistakes that may lead to an accident or incident. Two basic principles generally associated with the creation of a poor judgment chain are: (1) one bad decision often leads to another; and (2) as a string of bad decisions grows, it reduces the number of subsequent alternatives for continued safe flight. Aeronautical decision-making is intended to break the poor judgment chain before it can cause an accident or incident.

Practical Test Standards (PTS).  An FAA published list of standards which must be met for the issuance of a particular pilot certificate or rating. F A A inspectors and designated pilot examiners use these standards when conducting pilot practical tests and flight instructors should use the PTS while preparing applicants for practical tests.

Preparation.  The first step of the teaching process, which consists of determining the scope of the lesson, the objectives, and the goals to be attained. This portion also includes making certain all necessary supplies are on hand. When using the telling-and-doing technique of flight instruction, this step is accomplished prior to the flight lesson.

Presentation.  The second step of the teaching process, which consists of delivering information or demonstrating the skills that make up the lesson. The delivery could be by either the lecture method or demonstration-performance method. In the telling-and-doing technique of flight instruction, this is the segment in which the instructor both talks about and performs the procedure.

Pretest.  A test used to determine whether a student has the necessary qualifications to begin a course of study. Also used to determine the level of knowledge a student has in relation to the material that will be presented in the course.

Primacy.  A principle of learning in which the first experience of something often creates a strong, almost unshakable impression. The importance to an instructor is that the first time something is demonstrated, it must be shown correctly since that experience is the one most likely to be remembered by the student.

Problem-based learning.  Lessons in such a way as to confront students with problems that are encountered in real life which force them to reach real-world solutions.

Psychomotor domain.  A grouping of levels of learning associated with physical skill levels which range from perception through set, guided response, mechanism, complex overt response, and adaptation to origination.

PTS.  See Practical Test Standards. [Superseded by Airman Certification Standards]

Readiness.  A principle of learning where the eagerness and single-mindedness of a person toward learning affect the outcome of the learning experience.

Receiver.  In communication, the listener, reader, or student who takes in a message containing information from a source, processes it, reacts with understanding, and changes behavior in accordance with the message.

Recency.  Principle of learning stating that things learned recently are remembered better than things learned some time ago. As time passes, less is remembered. Instructors use this principle when summarizing the important points at the end of a lecture in order for students to better remember them.

Relay question.  Used in response to a student’s question, the student question is redirected to another student.

Reliability.  The degree to which test results are consistent with repeated measurements.

Repression.  Theory of forgetting proposing that a person is more likely to forget information which is unpleasant or produces anxiety.

Response.  Possible answer to a multiple-choice test item. The correct response is often called the keyed response, and incorrect responses are called distractors.

Reverse question.  Used in response to a student’s question. Rather than give a direct answer to the student’s query, the instructor returns the question to the same student to provide the answer.

Review and evaluation.  The fourth and last step in the teaching process, which consists of a review of all material and an evaluation of the students. In the telling and doing technique of flight instruction, this step consists of the instructor evaluating the student’s performance while the student performs the required procedure.

Rhetorical question.  Generally, a question asked for a purpose other than to obtain the information the question asks. For this handbook’s purpose, a question asked to stimulate group thought. Normally answered by the instructor, it is more commonly used in lecturing rather than in guided discussions.

Risk elements in ADM.  Take into consideration the four fundamental risk elements: the pilot, the aircraft, the environment, and external pressures. [PAVE]

Risk management.  The part of the decision-making process which relies on situational awareness, problem recognition, and good judgment to reduce risks associated with each flight.

Rote learning.  A basic level of learning in which the student has the ability to repeat back something learned, with no understanding or ability to apply what was learned.

Rubric  A guide used to score performance assessments in a reliable, fair, and valid manner. It is generally composed of dimensions for judging student performance, a scale for rating performances on each dimension, and standards of excellence for specified performance levels.

Schema  The cognitive framework that helps people organize and interpret information. Schemas can be revised by any new information and are useful because they allow people to take shortcuts in interpreting a vast amount of information.

Scenario-based training (SBT).  Training method that uses a highly structured script of real world experiences to address aviation training objectives in an operational environment.

Selection-type test items.  Test items requiring the student to choose from two or more alternatives provided. True-false, matching, and multiple-choice type questions are examples of selection type test items.

Self-concept.  A perception factor that ties together how people feel about themselves with how well they receive experiences.

Sensory register.  That portion of the brain which receives input from the five senses. The individual’s preconceived concept of what is important determines how the register prioritizes the information for passing it on to the rest of the brain for action.

Single-Pilot Resource Management (SRM).  The art/science of managing all the resources (both onboard the aircraft and from outside sources) available to a single pilot (prior and during flight) to ensure that the successful outcome of the flight is never in doubt.

Sites.  Internet addresses which provide information and often are linked to other similar sites.

Situational awareness.  The accurate perception and understanding of all the factors and conditions within the four fundamental risk elements that affect safety before, during, and after the flight.

Skill knowledge.  Knowledge reflected in motor or manual skills and in cognitive or mental skills that manifests itself in the doing of something.

Skills and procedures.  The procedural, psychomotor, and perceptual skills used to control a specific aircraft or its systems. They are the stick and rudder or airmanship abilities that are gained through conventional training, are perfected, and become almost automatic through experience.

Source.  In communication, the sender, speaker, transmitter, or instructor who composes and transmits a message made up of symbols which are meaningful to listeners and readers.

Stem.  The part of a multiple choice test item consisting of the question, statement, or problem.

Stress management.  The personal analysis of the kinds of stress experienced while flying, the application of appropriate stress assessment tools, and other coping mechanisms.

Supply-type test item.  Question in which the student supplies answers as opposed to selecting from choices provided. Essay or fill-in-the blank type questions are examples of supply-type test items.

Symbols.  In communication, simple oral and visual codes such as words, gestures, and facial expressions which are formed into sentences, paragraphs, lectures, or chapters to compose and transmit a message that means something to the receiver of the information.

Task.  Knowledge area, flight procedure, or maneuver within an area of operation in a practical test standard.

Taxonomy of educational objectives.  A systematic classification scheme for sorting learning outcomes into three broad categories (cognitive, affective, and psychomotor) and ranking the desired outcomes in a developmental hierarchy from least complex to most complex.

Teaching.  Instructing, training, or imparting knowledge or skill; the profession of someone who teaches.

Teaching lecture.  An oral presentation that is directed toward desired learning outcomes. Some student participation is allowed.

Telling-and-doing technique  . A technique of flight instruction that consists of the instructor first telling the student about a new procedure and then demonstrating it. This is followed by the student telling and the instructor doing. Third, the student explains the new procedure while doing it. Last, the instructor evaluates while the student performs the procedure.

Test.  A set of questions, problems, or exercises for determining whether a person has a particular knowledge or skill.

Test item.  A question, problem, or exercise that measures a single objective and requires a single response.

Time and opportunity.  A perception factor in which learning something is dependent on the student having the time to sense and relate current experiences in context with previous events.

Traditional assessment.  Written testing, such as multiple choice, matching, true/false, or fill-in-the-blank.

Training course outline.  Within a curriculum, describes the content of a particular course by statement of objectives, descriptions of teaching aids, definition of evaluation criteria, and indication of desired outcome.

Training media.  Any physical means that communicates an instructional message to students.

Training syllabus.  A step by-step, building block progression of learning with provisions for regular review and evaluations at prescribed stages of learning. The syllabus defines the unit of training, states by objective what the student is expected to accomplish during the unit of training, shows an organized plan for instruction, and dictates the evaluation process for either the unit or stages of learning.

Transfer of learning.  The ability to apply knowledge or procedures learned in one context to new contexts.

Transition training.  An instructional program designed to familiarize and qualify a pilot to fly types of aircraft not previously flown, such as tail wheel aircraft, high performance aircraft, and aircraft capable of flying at high altitudes.

True-false test item.  A test item consisting of a statement followed by an opportunity for the student to determine whether the statement is true or false.

Understanding.  A basic level of learning at which a student comprehends or grasps the nature or meaning of something.

Usability.  The functionality of tests.

Validity.  The extent to which a test measures what it is supposed to measure.

Virtual Reality (VR).  A form of computer-based technology that creates a sensory experience allowing a participant to believe and barely distinguish a virtual experience from a real one. VR uses graphics with animation systems, sounds, and images to reproduce electronic versions of real-life experience.

Working or short-term memory.  The portion of the brain that receives information from the sensory register. This portion of the brain can store information in memory for only a short period of time. If the information is determined by an individual to be important enough to remember, it must be coded in some way for transmittal to long-term memory.

Aviation Instructor’s Handbook FAA-H-8083-9A: Chapters 7-9

September 19th, 2018

Chapter 7: Instructor Responsibilities and Professionalism

Aviation Instructor Responsibilities

The learning process can be made easier by helping students learn, providing adequate instruction to meet established standards, measuring student performance against those standards, and emphasizing the positive.

Helping Students Learn
The use of standards, and measurement against standards, is key to helping students learn.

Providing Adequate Instruction
The flight instructor analyzes the student’s personality, thinking, and ability.

Students who are permitted to complete every flight lesson without corrections and guidance will not retain what they have practiced as well as those students who have their attention constantly directed to an analysis of their performance.

Standards of Performance
An aviation instructor is responsible for training an applicant to acceptable standards in all subject matter areas, procedures, and maneuvers included in the tasks within each area of operation in the appropriate Practical Test Standard (PTS).

Emphasizing the Positive
Chapter 1, Human Behavior, emphasized that a negative self-concept inhibits the perceptual process, that fear adversely affects student perceptions, that the feeling of being threatened limits the ability to perceive, and that negative motivation is not as effective as positive motivation.

Every effort should be made to ensure instruction is given under positive conditions that reinforce training conducted to standard and modification of the method of instruction when students have difficulty grasping a task.

Minimizing Student Frustrations
Motivate students—more can be gained from wanting to learn than from being forced to learn.

Keep students informed—students feel insecure when they do not know what is expected of them or what is going to happen to them.

Approach students as individuals—when instructors limit their thinking to the whole group without considering the individuals who make up that group, their efforts are directed at an average personality that really fits no one.

Give credit when due—when students do something extremely well, they normally expect their abilities and efforts to be noticed.

Criticize constructively—although it is important to give praise and credit when deserved, it is equally important to identify mistakes and failures.

Be consistent—If the same thing is acceptable one day and unacceptable the next, the student becomes confused.

Admit errors—the instructor can win the respect of students by honestly acknowledging mistakes.

Flight Instructor Responsibilities

The flight instructor’s job is to mold the student pilot into a safe pilot who takes a professional approach to flying.

Instructors should not introduce the minimum acceptable standards for passing the check ride when introducing lesson tasks. The minimum standards to pass the check ride should be introduced during the “3 hours of preparation” for the check ride. [Exam question.]

Physiological Obstacles for Flight Students
Negative sensations can usually be overcome by understanding the nature of their causes. [e.g. What seems like light chop to the instructor may seem to the student like the airplane is coming apart.]

Ensuring Student Skill Set
Flight instructors must ensure student pilots develop the required skills and knowledge prior to solo flight.

The student pilot must show consistency in the required solo tasks: takeoffs and landings, ability to prioritize in maintaining control of the aircraft, proper navigation skills, proficiency in flight, proper radio procedures and communication skills, and traffic pattern operation.

Special emphasis items include, but are not limited to:
1. Positive aircraft control
2. Procedures for positive exchange of flight controls
3. Stall and spin awareness (if appropriate)
4. Collision avoidance
5. Wake turbulence and low-level wind turbulence and wind shear avoidance
6. Runway incursion avoidance
7. Controlled flight into terrain (CFIT)
8. Aeronautical decision-making (ADM)/risk management
9. Checklist usage
10. Spatial disorientation
11. Temporary flight restrictions (TFR)
12. Special use airspace (SUA)
13. Aviation security
14. Wire strike avoidance

Aviator’s Model Code of Conduct

1. General Responsibilities of Aviators
2. Passengers and People on the Surface
3. Training and Proficiency
4. Security
5. Environmental Issues
6. Use of Technology
7. Advancement and Promotion of General Aviation

Safety Practices and Accident Prevention

FAA regulations intended to promote safety by eliminating or mitigating conditions that can cause death, injury, or damage are comprehensive, but even the strictest compliance with regulations may not be sufficient to guarantee safety.

The instructor’s advocacy and description of safety practices mean little to a student if the instructor does not demonstrate them consistently. [Exam question.]

Professionalism

Instructors need to commit themselves to continuous, lifelong learning and professional development through study, service, and membership in professional organizations such as the National Association of Flight Instructors (NAFI). Professionals build a library of resources that keeps them in touch with their field through the most current procedures, publications, and educational opportunities.

Sincerity
Teaching an aviation student is based upon acceptance of the instructor as a competent, qualified teacher and an expert pilot.

Acceptance of the Student
The instructor must accept students as they are, including all their faults and problems.

Personal Appearance and Habits
Personal appearance has an important effect on the professional image of the instructor.

Demeanor
The instructor should avoid erratic movements, distracting speech habits, and capricious changes in mood. The successful instructor avoids contradictory directions, reacting differently to similar or identical errors at different times, demanding unreasonable performance or progress, or criticizing a student unfairly, and presenting an overbearing manner or air of flippancy.

Proper Language
Many people object to such language.

Evaluation of Student Ability

Evaluation refers to judging a student’s ability to perform a maneuver or procedure.

Demonstrated Ability
Evaluation of demonstrated ability during flight or maintenance instruction is based upon established standards of performance, suitably modified to apply to the student’s experience and stage of development as a pilot or mechanic.

Keeping the Student Informed
In evaluating student demonstrations of ability, it is important for the aviation instructor to keep the student informed of progress.

Correction of Student Errors
Safety permitting, it is frequently better to let students progress part of the way into the mistake and find a way out.

Students may perform a procedure or maneuver correctly but not fully understand the principles and objectives involved. If the instructor suspects this, students should be required to vary the performance of the maneuver or procedure slightly. [Exam question.]

Aviation Instructors and Exams

Knowledge Test
If the applicant fails a test, the aviation instructor must sign the test after he or she has provided additional training in the areas the applicant failed.

Practical Test
A flight instructor who makes a practical test recommendation for an applicant seeking a certificate or rating should require the applicant to thoroughly demonstrate the knowledge and skill level required for that certificate or rating.

Professional Development

Aviation is changing rapidly, and aviation instructors must continue to develop their knowledge and skills in order to teach successfully in this environment.

Continuing Education
A professional aviation instructor continually updates his or her knowledge and skills.

Government
Educational/Training Institutions
Commercial Organizations
Industry Organizations

Sources of Material
Printed Material
Electronic Sources
The more familiar aviation instructors become with the Internet, the better they are able to adapt to any changes that may occur.

Chapter Summary

This chapter discussed the responsibilities of aviation instructors to the student, the public, and the FAA in the training process. The additional responsibilities of flight instructors who teach new student pilots as well as rated pilots seeking add-on certification, the role of aviation instructors as safety advocates, and ways in which aviation instructors can enhance their professional image and development were explored.

Chapter 8: Techniques of Flight Instruction

According to one definition, safety is the freedom from conditions that can cause death, injury, or illness; damage to/ loss of equipment or property, or damage to the environment. FAA regulations are intended to promote safety by eliminating or mitigating conditions that can cause death, injury, or damage. These regulations are comprehensive, but there has been increasing recognition that even the strictest compliance with regulations may not be sufficient to guarantee safety.

This chapter introduces system safety—aeronautical decision-making (ADM), risk management, situational awareness, and single-pilot resource management (SRM)—in the modern flight training environment.

Flight Instructor Qualifications

A CFI must be thoroughly familiar with the functions, characteristics, and proper use of all flight instruments, avionics, and other aircraft systems being used for training.

Practical Flight Instructor Strategies

The flight instructor should demonstrate good aviation sense at all times:
• Before the flight—discuss safety and the importance of a proper preflight and use of the checklist.
• During flight—prioritize the tasks of aviating, navigating, and communicating. Instill importance of “see and avoid” in the student.
• During landing—conduct stabilized approaches, maintain desired airspeed on final, demonstrate good judgment for go-arounds, wake turbulence, traffic, and terrain avoidance. Use ADM to correct faulty approaches and landing errors. Make power-off, stall-warning blaring, on centerline touchdowns in the first third of runway.
• Always—remember safety is paramount.

Obstacles to Learning During Flight Instruction

• Feeling of unfair treatment
• Impatience to proceed to more interesting operations
• Worry or lack of interest
• Physical discomfort, illness, fatigue, and dehydration
• Apathy due to inadequate instruction
• Anxiety

Unfair Treatment
Students who believe their instruction is inadequate, or that their efforts are not conscientiously considered and evaluated, do not learn well.

Impatience
Impatience is a greater deterrent to learning pilot skills than is generally recognized.

The instructor can correct student impatience by presenting the necessary preliminary training one step at a time, with clearly stated goals for each step. The procedures and elements mastered in each step should be clearly identified in explaining or demonstrating the performance of the subsequent step. [Exam question.]

Worry or Lack of Interest
Students who are worried or emotionally upset are not ready to learn and derive little benefit from instruction. The instructor must be alert and ensure the students understand the objectives of each step of their training, and that they know at the completion of each lesson exactly how well they have progressed and what deficiencies are apparent. Discouragement and emotional upsets are rare when students feel that nothing is being withheld from them or is being neglected in their training.

Physical Discomfort, Illness, Fatigue, and Dehydration
Students who are not completely at ease, and whose attention is diverted by discomforts such as the extremes of temperature, poor ventilation, inadequate lighting, or noise and confusion, cannot learn at a normal rate.

Fatigue
Fatigue is one of the most treacherous hazards to flight safety as it may not be apparent to a pilot until serious errors are made. Fatigue can be either acute (short-term) or chronic (long-term). Acute fatigue, a normal occurrence of everyday living, is the tiredness felt after long periods of physical and mental strain, including strenuous muscular effort, immobility, heavy mental workload, strong emotional pressure, monotony, and lack of sleep. [Exam question.]

Acute fatigue is characterized by:
• Inattention
• Distractibility
• Errors in timing
• Neglect of secondary tasks
• Loss of accuracy and control
• Lack of awareness of error accumulation
• Irritability

Chronic fatigue which occurs when there is not enough time for a full recovery from repeated episodes of acute fatigue. Chronic fatigue is a combination of both physiological problems and psychological issues. [Exam question.] [Chronic fatigue can also be caused by sleep apnea.]

Dehydration and Heatstroke
Dehydration is the term given to a critical loss of water from the body. Dehydration reduces a pilot’s level of alertness, producing a subsequent slowing of decision-making processes or even the inability to control the aircraft. The first noticeable effect of dehydration is fatigue. [Exam question.]

Heatstroke is a condition caused by any inability of the body to control its temperature. Onset of this condition may be recognized by the symptoms of dehydration, but also has been known to be recognized only by complete collapse.

Apathy Due to Inadequate Instruction
To hold the student’s interest and to maintain the motivation necessary for efficient learning, well-planned, appropriate, and accurate instruction must be provided.

To be effective, the instructor must teach for the level of the student. The presentation must be adjusted to be meaningful to the person for whom it is intended.

Anxiety
The student must be comfortable, confident in the instructor and the aircraft, and at ease if effective learning is to occur.

Demonstration-Performance Training Delivery Method

This training method has been in use for a long time and is very effective in teaching kinesthetic skills.

Explanation Phase
The explanation phase is accomplished prior to the flight lesson with a discussion of lesson objectives and completion standards, as well as a thorough preflight briefing. In addition to the necessary steps, the instructor should describe the end result of these efforts.

Demonstration Phase
As little extraneous activity as possible should be included in the demonstration if students are to clearly understand that the instructor is accurately performing the actions previously explained.

Student Performance and Instructor Supervision Phases
It is important that students be given an opportunity to perform the skill as soon as possible after a demonstration. Then, the instructor reviews what has been covered during the instructional flight and determines to what extent the student has met the objectives outlined during the preflight discussion.

Evaluation Phase
PBL structures the lessons to confront students with problems that are encountered in real life and forces them to reach real-world solutions. Scenario-based training (SBT), a type of PBL, uses a highly structured script of real world experiences to address aviation training objectives in an operational environment. Collaborative assessment is used to evaluate whether certain learning criteria were met during the SBT.

The Telling-and-Doing Technique
First, the flight instructor gives a carefully planned demonstration of the procedure or maneuver with accompanying verbal explanation. It is important for the demonstration to conform to the explanation as closely as possible. In addition, it should be demonstrated in the same sequence in which it was explained so as to avoid confusion and provide reinforcement.

Student Tells—Instructor Does
In this step, the student actually plays the role of instructor, telling the instructor what to do and how to do it.

Student Tells—Student Does
This is where learning takes place and where performance habits are formed. If the student has been adequately prepared and the procedure or maneuver fully explained and demonstrated, meaningful learning occurs.

Positive Exchange of Flight Controls

Positive exchange of flight controls is an integral part of flight training. It is especially critical during the demonstration-performance method of flight instruction.

Background
Numerous accidents have occurred due to a lack of communication or misunderstanding regarding who had actual control of the aircraft, particularly between students and flight instructors.

Positive Exchange of Flight Controls

Sterile Cockpit Rule
Commonly known as the “sterile cockpit rule,” Title 14 of the Code of Federal Regulations (14 CFR) section 121.542 requires flight crewmembers to refrain from nonessential activities during critical phases of flight. As defined in the regulation, critical phases of flight are all ground operations involving taxi, takeoff, and landing, and all other flight operations below 10,000 feet except cruise flight.

Use of Distractions

National Transportation Safety Board (NTSB) statistics reveal that most stall/spin accidents occurred when the pilot’s attention was diverted from the primary task of flying the aircraft. Sixty percent of stall/spin accidents occurred during takeoff and landing, and twenty percent were preceded by engine failure. Preoccupation inside or outside the flight deck while changing aircraft configuration or trim, maneuvering to avoid other traffic, or clearing hazardous obstacles during takeoff and climb could create a potential stall/spin situation. The intentional practice of stalls and spins seldom resulted in an accident. The real danger was inadvertent stalls induced by distractions during routine flight situations.

Integrated Flight Instruction

Students are taught to perform flight maneuvers both by outside visual references and by reference to flight instruments. No distinction in the pilot’s operation of the flight controls is permitted, regardless of whether outside references or instrument indications are used for the performance of the maneuver. When this training technique is used, instruction in the control of an aircraft by outside visual references is integrated with instruction in the use of flight instrument indications for the same operations. [Exam question.]

Development of Habit Patterns
It important for the student to establish the habit of observing and relying on flight instruments from the beginning of flight training. It is equally important for the student to learn the feel of the airplane while conducting maneuvers, such as being able to feel when the airplane is out of trim or in a nose-high or nose-low attitude. Students who have been required to perform all normal flight maneuvers by reference to instruments, as well as by outside references, develop from the start the habit of continuously monitoring their own and the aircraft’s performance. The early establishment of proper habits of instrument cross-check, instrument interpretation, and aircraft control is highly useful to the student.

Operating Efficiency
The use of correct power settings and climb speeds and the accurate control of headings during climbs result in a measurable increase in climb performance. Holding precise headings and altitudes in cruising flight definitely increases average cruising performance.

The use of integrated flight instruction provides the student with the ability to control an aircraft in flight for limited periods if outside references are lost. In an emergency, this ability could save the pilot’s life and those of the passengers.

Procedures
Integrated flight instruction begins with the first briefing on the function of the flight controls. This briefing includes the instrument indications to be expected, as well as the outside references to be used to control the attitude of the aircraft.

See and Avoid
From the start of flight training, the instructor must ensure students develop the habit of looking for other air traffic at all times.

• Flight instructors were onboard the aircraft in 37 percent of the accidents in the study.
• Most midair collisions occur in VFR weather conditions during weekend daylight hours.
• The vast majority of accidents occurred at or near nontowered airports and at altitudes below 1,000 feet.

Assessment of Piloting Ability

A well designed assessment provides a student with something constructive upon which he or she can work or build. An assessment should provide direction and guidance to raise the level of performance. There are many types of assessment, but the flight instructor generally uses the review, collaborative assessment (LCG), written tests, and performance-based tests to ascertain knowledge or practical skill levels.

Demonstrated Ability
The assessment must consider the student’s mastery of the elements involved in the maneuver, rather than merely the overall performance.

Postflight Evaluation
Traditionally, flight instructors explained errors in performance, pointed out elements in which the deficiencies were believed to have originated and, if possible, suggested appropriate corrective measures.

With the advent of SBT, collaborative assessment is used whenever the student has completed a scenario. The self-assessment is followed by an in-depth discussion between the instructor and the student which compares the instructor’s assessment to the student’s self-assessment.

First Solo Flight
During the student’s first solo flight, the instructor must be present to assist in answering questions or resolving any issues that arise during the flight. A radio enables the instructor to terminate the solo operation if he or she observes a situation developing.

Post-Solo Debriefing
It is very important for the flight instructor to debrief a student immediately after a solo flight. With the flight vividly etched in the student’s memory, questions about the flight will come quickly.

Correction of Student Errors
Students may perform a procedure or maneuver correctly and not fully understand the principles and objectives involved. When the instructor suspects this, students should be required to vary the performance of the maneuver slightly, combine it with other operations, or apply the same elements to the performance of other maneuvers.

Pilot Supervision
Flight instructors have the responsibility to provide guidance and restraint with respect to the solo operations of their students. Before endorsing a student for solo flight, the instructor should require the student to demonstrate consistent ability to perform all of the fundamental maneuvers. [Exam question.]

Dealing with Normal Challenges
Instructors should teach students how to solve ordinary problems encountered during flight.

Visualization
For example, have a student visualize how the flight may occur under normal circumstances, with the student describing how he or she would fly the flight. Then, the instructor adds unforeseen circumstances. The job of the instructor is to challenge the student with realistic flying situations without overburdening him or her with unrealistic scenarios.

Practice Landings
The FAA recommends that in all student flights involving landings in an aircraft, the flight instructor should teach a full stop landing. Full stop landings help the student develop aircraft control and checklist usage. Aircraft speed and control take precedence over all other actions during landings and takeoffs.

Practical Test Recommendations
The CFI should require the applicant to demonstrate thoroughly the knowledge and skill level required for that certificate or rating. This demonstration should in no instance be less than the complete procedure prescribed in the applicable PTS.

Completion of prerequisites for a practical test is another instructor task that must be documented properly.

Flight instructor recommendations are evidence of qualification for certification, and proof that a review has been given of the subject areas found to be deficient on the appropriate knowledge test.

Aeronautical Decision-Making

The goal of system safety is for pilots to utilize all four concepts (ADM, risk management, situational awareness, and SRM) so that risk can be reduced to the lowest possible level.

ADM is a systematic approach to the mental process used by aircraft pilots to consistently determine the best course of action in response to a given set of circumstances. Risk management is a decision-making process designed to systematically identify hazards, assess the degree of risk, and determine the best course of action associated with each flight. Situational awareness is the accurate perception and understanding of all the factors and conditions within the four fundamental risk elements that affect safety before, during, and after the flight. SRM is the art and science of managing all resources (both onboard the aircraft and from outside sources) available to a single pilot (prior and during flight) to ensure the successful outcome of the flight.

It is estimated that approximately 80 percent of all aviation accidents are human factors related.

Pilot error means that an action or decision made by the pilot was the cause of, or contributing factor to, the accident. This definition also includes the pilot’s failure to make a decision or take action. From a broader perspective, the phrase “human factors related” more aptly describes these accidents since it is usually not a single decision that leads to an accident, but a chain of events triggered by a number of factors. Breaking one link in the chain is all that is usually necessary to change the outcome of the sequence of events.

Traditional pilot instruction has emphasized flying skills, knowledge of the aircraft, and familiarity with regulations. ADM training focuses on the decision-making process and the factors that affect a pilot’s ability to make effective choices.

Timely decision-making is an important tool for any pilot. The pilot who hesitates when prompt action is required, or who makes the decision to not decide, has made a wrong decision.

Declaring an emergency when one occurs is an appropriate reaction. 14 CFR §91.3, “In an inflight emergency requiring immediate action, the pilot in command may deviate from any rule of this part to the extent required to meet that emergency.”

The Decision-Making Process
An understanding of the decision-making process provides students with a foundation for developing ADM skills. Traditionally, pilots have been well trained to react to emergencies, but are not as well prepared to make decisions, which require a more reflective response.

Defining the Problem
This begins with recognizing that a change has occurred or that an expected change did not occur.

One critical error that can be made during the decision-making process is incorrectly defining the problem.

Choosing a Course of Action
After the problem has been identified, the pilot evaluates the need to react to it and determines the actions that may be taken to resolve the situation in the time available.

Implementing the Decision and Evaluating the Outcome
It is important to think ahead and determine how the decision could affect other phases of the flight.

Factors Affecting Decision-Making

Recognizing Hazardous Attitudes
Two steps to improve flight safety are identifying personal attitudes hazardous to safe flight and learning behavior modification techniques.

Attitude: “Description” -> Antidote
Macho: “I can do it.” -> Taking chances is foolish.
Anti-authority” “Don’t tell me.” -> Follow the rules. They are usually right.
Impulsivity: “Do it quickly.” -> It could happen to me.
Invulnerability: “It won’t happen to me.” -> Not so fast. Think first.
Resignation: “What’s the use?” -> I’m not helpless. I can make a difference.

Stress Management
The effects of stress are cumulative and, if not coped with adequately, they eventually add up to an intolerable burden. Performance generally increases with the onset of stress, peaks, and then begins to fall off rapidly as stress levels exceed a person’s ability to cope.[Exam question.]

Stressors

Use of Resources
During training, CFIs can routinely point out resources to students.

Internal Resources
A thorough understanding of all the equipment and systems in the aircraft is necessary to fully utilize all resources.

Checklists are essential flight deck resources for verifying that the aircraft instruments and systems are checked, set, and operating properly, as well as ensuring that the proper procedures are performed if there is a system malfunction or inflight emergency.

External Resources
ATC can help decrease pilot workload by providing traffic advisories, radar vectors, and assistance in emergency situations. AFSS can provide updates on weather, answer questions about airport conditions, and may offer direction-finding assistance.

Workload Management
Effective workload management ensures that essential operations are accomplished by planning, prioritizing, and sequencing tasks to avoid work overload. As experience is gained, a pilot learns to recognize future workload requirements and can prepare for high workload periods during times of low workload.

Another important part of managing workload is recognizing a work overload situation. The first effect of high workload is that the pilot begins to work faster. As workload increases, attention cannot be devoted to several tasks at one time, and the pilot may begin to focus on one item. When the pilot becomes task saturated, there is no awareness of inputs from various sources; decisions may be made on incomplete information, and the possibility of error increases.

Chapter Summary

This chapter discussed the demonstration-performance and telling-and-doing training delivery methods of flight instruction, SBT techniques, practical strategies flight instructors can use to enhance their instruction, integrated flight instruction, positive exchange of flight controls, use of distractions, obstacles to learning encountered during flight training, and how to evaluate students. After an intensive look at ADM with suggestions for how to interweave ADM, risk management, and SRM into the teaching process, it closes with a discussion of CFI recommendations. Additional information on recommendations and endorsements can be found in Appendix E, Flight Instructor Endorsements.

Chapter 9: Risk Management

Safety risk management in the aviation community is preemptive rather than reactive.

Defining Risk Management

Risk is defined as the probability and possible severity of accident or loss from exposure to various hazards, including injury to people and loss of resources.

Hazard—a present condition, event, object, or circumstance that could lead to or contribute to an unplanned or undesired event, such as an accident.
Risk—the future impact of a hazard that is not controlled or eliminated. It is the possibility of loss or injury. The level of risk is measured by the number of people or resources affected (exposure); the extent of possible loss (severity); and likelihood of loss (probability).
Safety—freedom from those conditions that can cause death, injury, occupational illness, or damage to or loss of equipment or property, or damage to the environment. Therefore, safety is a relative term that implies a level of risk that is both perceived and accepted.

Risk

Principles of Risk Management
Accept No Unnecessary Risk
Unnecessary risk is that which carries no commensurate return in terms of benefits or opportunities.

Make Risk Decisions at the Appropriate Level
The appropriate decision-maker is the person who can develop and implement risk controls. The decision-maker must be authorized to accept levels of risk typical of the planned operation.

Accept Risk When Benefits Outweigh the Costs
All identified benefits should be compared against all identified costs.

Integrate Risk Management Into Planning at All Levels
Risks are more easily assessed and managed in the planning stages of an operation. But safety requires the use of appropriate and effective risk management not just in the preflight planning stage, but in all stages of the flight.

Risk Management Process
Risk management is a simple process which identifies operational hazards and takes reasonable measures to reduce risk to personnel, equipment, and the mission.

Step 1: Identify the Hazard
A hazard is defined as any real or potential condition that can cause degradation, injury, illness, death, or damage to or loss of equipment or property. Experience, common sense, and specific analytical tools help identify risks.

Step 2: Assess the Risk
The assessment step is the application of quantitative and qualitative measures to determine the level of risk associated with specific hazards. This process defines the probability and severity of an accident that could result from the hazards based upon the exposure of humans or assets to the hazards.

Step 3: Analyze Risk Control Measures
Investigate specific strategies and tools that reduce, mitigate, or eliminate the risk. All risks have two components:
1. Probability of occurrence
2. Severity of the hazard
Effective control measures reduce or eliminate at least one of these. The analysis must take into account the overall costs and benefits of remedial actions, providing alternative choices if possible.

Step 4: Make Control Decisions
Identify the appropriate decision-maker.

Step 5: Implement Risk Controls
A plan for applying the selected controls must be formulated, the time, materials, and personnel needed to put these measures in place must be provided.

Step 6: Supervise and Review
Once controls are in place, the process must be reevaluated periodically to ensure their effectiveness. The risk management process continues throughout the life cycle of the system, mission, or activity.

Implementing the Risk Management Process
• Apply the steps in sequence—each step is a building block for the next
• Allocate the time and resources to perform all steps in the process.
• Apply the process in a cycle—the “supervise and review” step should include a brand new look at the operation being analyzed to see whether new hazards can be identified.
• Involve people in the process—the people who are actually exposed to risks usually know best what works and what does not.

Level of Risk

The level of risk posed by a given hazard is measured in terms of:
• Severity (extent of possible loss)
• Probability (likelihood that a hazard will cause a loss)

Assessing Risk
Every flight has hazards and some level of risk associated with it. It is critical that pilots and especially students are able to differentiate in advance between a low-risk flight and a high-risk flight, and then establish a review process and develop risk mitigation strategies to address flights throughout that range.

Likelihood of an Event
• Probable—an event will occur several times.
• Occasional—an event will probably occur sometime.
• Remote—an event is unlikely to occur, but is possible.
• Improbable—an event is highly unlikely to occur.

Severity of an Event
• Catastrophic—results in fatalities, total loss
• Critical—severe injury, major damage
• Marginal—minor injury, minor damage
• Negligible—less than minor injury, less than minor system damage

Mitigating Risk

After determining the level of risk, the pilot needs to mitigate the risk

IMSAFE Checklist
1. Illness—Am I sick? Illness is an obvious pilot risk.
2. Medication—Am I taking any medicines that might affect my judgment or make me drowsy?
3. Stress—Stress causes concentration and performance problems. While the regulations list medical conditions that require grounding, stress is not among them.
4. Alcohol—Have I been drinking within 8 hours? Within 24 hours?
5. Fatigue—Am I tired and not adequately rested? Fatigue continues to be one of the most insidious hazards to flight safety, as it may not be apparent to a pilot until serious errors are made.
6. Emotions-A pilot who experiences an emotionally upsetting event should refrain from flying until the pilot has satisfactorily recovered.

The PAVE Checklist
Pilot in command (PIC), Aircraft, enVironment, and External pressures

Three-P Model for Pilots

Risk management is a decision-making process designed to perceive hazards systematically, assess the degree of risk associated with a hazard, and determine the best course of action.

• Perceives the given set of circumstances for a flight.
• Processes by evaluating the impact of those circumstances on flight safety.
• Performs by implementing the best course of action.

In the first step, the goal is to develop situational awareness by perceiving hazards, which are present events, objects, or circumstances that could contribute to an undesired future event. In this step, the pilot systematically identifies and lists hazards associated with all aspects of the flight: pilot, aircraft, environment, and external pressures.

In the second step, the goal is to process this information to determine whether the identified hazards constitute risk, which is defined as the future impact of a hazard that is not controlled or eliminated. The degree of risk posed by a given hazard can be measured in terms of exposure (number of people or resources affected), severity (extent of possible loss), and probability (the likelihood that a hazard will cause a loss).

In the third step, the goal is to perform by taking action to eliminate hazards or mitigate risk, and then continuously evaluate the outcome of this action.

The decision-making process is a continuous loop of perceiving, processing, and performing.

Pilot Self-Assessment

Just as a checklist is used when preflighting an aircraft, a personal checklist based on such factors as experience, currency, and comfort level can help determine if a pilot is prepared for a particular flight.

Situational Awareness

Situational awareness is the accurate perception and understanding of all the factors and conditions within the four fundamental risk elements that affect safety before, during, and after the flight.

Obstacles to Maintaining Situational Awareness
Fatigue, stress, or work overload can cause the pilot to fixate on a single perceived important item rather than maintaining an overall awareness of the flight situation. A contributing factor in many accidents is a distraction, which diverts the pilot’s attention from monitoring the instruments or scanning outside the aircraft.

As fatigue progresses, it is responsible for increased errors of omission, followed by errors of commission, and microsleeps, or involuntary sleep lapses lasting from a few seconds to a few minutes.

Warning Signs of Fatigue
Eyes going in and out of focus
Head bobs involuntarily
Persistent yawning
Spotty short-term memory
Wandering or poorly organized thoughts
Missed or erroneous performance of routine procedures
Degradation of control accuracy

Complacency presents another obstacle to maintaining situational awareness. Defined as overconfidence from repeated experience on a specific activity, complacency has been implicated as a contributing factor in numerous aviation accidents and incidents.

Operational Pitfalls
There are numerous classic behavioral traps that can ensnare the unwary pilot. The basic drive to demonstrate achievements can have an adverse effect on safety, and can impose an unrealistic assessment of piloting skills under stressful conditions.

Single-Pilot Resource Management (SRM)

Single pilot resource management (SRM) is defined as the art and science of managing all the resources (both onboard the aircraft and from outside sources) available to a single pilot (prior to and during flight) to ensure the successful outcome of the flight. SRM includes the concepts of ADM, Risk Management (RM), Task Management (TM), Automation Management (AM), Controlled Flight Into Terrain (CFIT) Awareness, and Situational Awareness (SA).

SRM and the 5P Check
SRM is about gathering information, analyzing it, and making decisions. Learning how to identify problems, analyze the information, and make informed and timely decisions is not as straightforward as the training involved in learning specific maneuvers. Learning how to judge a situation and “how to think” in the endless variety of situations encountered while flying out in the “real world” is more difficult. There is no one right answer in ADM; rather, each pilot is expected to analyze each situation in light of experience level, personal minimums, and current physical and mental readiness level, and make his or her own decision.

The 5 Ps consist of “the Plan, the Plane, the Pilot, the Passengers, and the Programming.” The 5P concept relies on the pilot to adopt a scheduled review of the critical variables at points in the flight where decisions are most likely to be effective. The second easiest point in the flight to make a critical safety decision is just prior to takeoff. The third place to review the 5 Ps is at the midpoint of the flight. The last two decision points are just prior to decent into the terminal area and just prior to the final approach fix, or if VFR just prior to entering the traffic pattern, as preparations for landing commence.

The Plan
The plan can also be called the mission or the task. The plan should be reviewed and updated several times during the course of the flight.

The Plane
Both the plan and the plane are fairly familiar to most pilots. The plane consists of the usual array of mechanical and cosmetic issues that every aircraft pilot, owner, or operator can identify. With the advent of advanced avionics, the plane has expanded to include database currency, automation status, and emergency backup systems that were unknown a few years ago.

The Pilot
The traditional “IMSAFE” checklist is a good start.

The Passengers
One of the key differences between CRM and SRM is the way passengers interact with the pilot. Pilots also need to understand that non-pilots may not understand the level of risk involved in the flight.

The Programming
While programming and operation of these devices are fairly simple and straightforward, unlike the analog instruments they replace, they tend to capture the pilot’s attention and hold it for long periods of time.

The SRM process is simple. At least five times before and during the flight, the pilot should review and consider the “Plan, the Plane, the Pilot, the Passengers, and the Programming” and make the appropriate decision required by the current situation.

Information Management
The volume of information presented in aviation training is enormous, but part of the process of good SRM is a continuous flow of information in and actions out.

Task Management
Task management (TM), a significant factor in flight safety, is the process by which pilots manage the many, concurrent tasks that must be performed to safely and efficiently fly a modern aircraft. A task is a function performed by a human, as opposed to one performed by a machine.

Task management entails initiation of new tasks; monitoring of ongoing tasks to determine their status; prioritization of tasks based on their importance, status, urgency, and other factors; allocation of human and machine resources to high-priority tasks; interruption and subsequent resumption of lower priority tasks; and termination of tasks that are completed or no longer relevant.

Automation Management
Automation management is the demonstrated ability to control and navigate an aircraft by means of the automated systems installed in the aircraft. One of the most important concepts of automation management is knowing when to use it and when not to use it.

Teaching Decision-Making Skills

System safety flight training occurs in three phases. First, there are the traditional stick and rudder maneuvers. In order to apply the critical thinking skills that are to follow, pilots must first have a high degree of confidence in their ability to fly the aircraft. Next, the tenets of system safety are introduced into the training environment as students begin to learn how best to identify hazards, manage risk, and use all available resources to make each flight as safe as possible. This can be accomplished through scenarios that emphasize the skill sets being taught. Finally, the student is introduced to more complex scenarios demanding focus on several safety-of-flight issues. Thus, scenarios should start out rather simply, then progress in complexity and intensity as the student can handle the learning load.

It is also important for the flight instructor to remember that a good scenario:
• Is not a test.
• Will not have a single correct answer.
• Does not offer an obvious answer.
• Engages all three learning domains.
• Is interactive.
• Should not promote errors.
• Should promote situational awareness and opportunities for decision-making.
• Requires time-pressured decisions.

Assessing SRM Skills

In SRM assessment, instructors must learn to assess students on a different level. How did the student arrive at a particular decision? What resources were used? Was risk assessed accurately when a go/no-go decision was made? Did the student maintain situational awareness in the traffic pattern? Was workload managed effectively during a cross-country flight? How does the student handle stress and fatigue?

Instructors should utilize SBT to create lessons that are specifically designed to test whether students are applying SRM skills. Planning a flight lesson in which the student is presented with simulated emergencies, a heavy workload, or other operational problems can be valuable in assessing the student’s judgment and decision-making skills

SRM grades are based on these four components: Explain, Practice, Manage/Decide, Not Observed

The purpose of the self-assessment is to stimulate growth in the student’s thought processes and, in turn, behaviors. The self-assessment is followed by an in-depth discussion between the flight instructor and the student which compares the CFI’ s assessment to the student’ s self-assessment.

Chapter Summary

This chapter introduced aviation instructors to the underlying concepts of safety risk management, which the FAA is integrating into all levels of the aviation community.

Aviation Instructor’s Handbook FAA-H-8083-9A: Chapters 4-6

September 19th, 2018

Chapter 4: The Teaching Process

Use a combination of teaching methods (lecture, group learning, and discussion) and instructional aids (audio/visual and handouts), to achieve instructional objectives.

The Essence of Good Teaching (1985), psychologist Stanford C. Ericksen wrote “good teachers select and organize worthwhile course material, lead students to encode and integrate this material in memorable form, ensure competence in the procedures and methods of a discipline, sustain intellectual curiosity, and promote how to learn independently.”

Effective instructors come in many forms, but they generally possess four essential teaching skills: people skills, subject matter expertise, management skills, and assessment skills.

People skills are the ability to interact, talk, understand, empathize, and connect with people. Effective instructors relate well to people.

A subject matter expert (SME) is a person who possesses a high level of expertise, knowledge, or skill in a particular area.

Management skills generally include the ability to plan, organize, lead, and supervise.

Assessment of learning is a complex process and it is important to be clear about the purposes of the assessment. There are several points at which assessments can be made: before training, during training, and after training.

Instructor’s Code of Conduct

An aviation instructor needs to remember he or she is teaching a pilot or technician who should:
• Make safety the number one priority,
• Develop and exercise good judgment in making decisions,
• Recognize and manage risk effectively,
• Be accountable for his or her actions,
• Act with responsibility and courtesy,
• Adhere to prudent operating practices and personal operating parameters, and
• Adhere to applicable laws and regulations.

The Certificated Flight Instructor (CFI) needs to remember he or she is teaching a pilot who should:
• Seek proficiency in control of the aircraft,
• Use flight deck technology in a safe and appropriate way,
• Be confident in a wide variety of flight situations, and
• Be respectful of the privilege of flight.

The teaching process consists of four steps: preparation, presentation, application, and assessment. Regardless of the teaching or training delivery method used, the teaching process remains the same. To be effective, an instructor utilizes people skills, subject matter expertise, management skills, and assessment skills.

Course of Training

In education, a course of training is a complete series of studies leading to attainment of a specific goal.

A curriculum is a set of courses in an area of specialization offered by an educational institution.

A syllabus is a summary or outline of a course of study that generally contains a description of each lesson, including objectives and completion standards.

Atraining course outline within a curriculum is the content of a particular course. It normally includes statements of objectives, descriptions of teaching aids, definitions of assessment criteria, and indications of desired outcome.

Preparation of a Lesson

A determination of objectives and standards is necessary before any important instruction can be presented.

Training Objectives and Standards
Performance-based objectives are essential in defining exactly what needs to be done and how it is done during each lesson. As the student progresses through higher levels of performance and understanding, the instructor should shift the training focus to decision-based training objectives. Decision-based training objectives allow for a more dynamic training environment and are ideally suited to scenario type training. The instructor uses decision-based training objectives to teach aviation students critical thinking skills, such as risk management and aeronautical decision-making (ADM).

Standards are closely tied to objectives since they include a description of the desired knowledge, behavior, or skill stated in specific terms, along with conditions and criteria. When a student is able to perform according to well-defined standards, evidence of learning is apparent.

Performance-based objectives are used to set measurable, reasonable standards that describe the desired performance of the student. The objectives must be clear, measurable, and repeatable. (And written.)

Performance-based objectives consist of three elements: description of the skill or behavior, conditions, and criteria.

Description of the Skill or Behavior
The description of the skill or behavior explains the desired outcome of the instruction. It is actually a learned capability, which may be defined as knowledge, a skill, or an attitude.

Conditions
Conditions are necessary to specifically explain the rules under which the skill or behavior is demonstrated.

Criteria
Criteria are the standards that measure the accomplishment of the objective. The criteria should be stated so that there is no question whether the objective has been met.

The Importance of the PTS [ACS] in Aviation Training Curricula
PTS hold an important position in aviation training curricula because they supply the instructor with specific performance objectives based on the standards that must be met for the issuance of a particular aviation certificate or rating.

Content validity is reflected by a particular maneuver closely mimicking a maneuver required in actual flight. Criterion validity means that the completion standards for the test are reflective of acceptable standards in actual flight.

Decision-Based Objectives
Decision-based objectives are designed specifically to develop pilot judgment and ADM skills. Improper pilot decisions cause a significant percentage of all accidents, and the majority of fatal accidents in light single- and twin-engine aircraft.

Other Uses of Training Objectives
Performance-based and decision-based objectives are also helpful for an instructor designing a lesson plan. Training objectives apply to all three domains of learning—cognitive (knowledge), affective (attitudes, beliefs, values), and psychomotor (physical skills).

Presentation of a Lesson

Lesson Presentation

Organization of Material

Generally, the syllabus contains a description of each lesson, including objectives and completion standards. The traditional organization of a lesson plan is introduction, development, and conclusion.

Introduction
The introduction is made up of three elements: attention, motivation, and an overview of what is to be covered. The purpose of the attention element is to focus each student’s attention on the lesson. The purpose of the motivation element is to offer the students specific reasons why the lesson content is important to know, understand, apply, or perform concepts of Thorndike’s law of readiness. Every lesson introduction should contain an overview that tells the group what is to be covered during the period.

Development
Development is the main part of the lesson. The instructor must logically organize the material to show the relationships of the main points. The instructor usually shows these primary relationships by developing the main points in one of the following ways: from past to present, simple to complex, known to unknown, and most frequently used to least used.

Conclusion
An effective conclusion retraces the important elements of the lesson and relates them to the objective.

Training Delivery Methods

Instructors can choose from a wealth of ways to present instructional material: lecture, discussion, guided discussion, problem based, group learning, demonstration- performance, or e-learning. A training delivery method is rarely used by itself. In a typical lesson, an effective instructor normally uses a combination of methods.

Lecture Method
Lectures are used for introduction of new subjects, summarizing ideas, showing relationships between theory and practice, and reemphasizing main points.

The illustrated talk where the speaker relies heavily on visual aids to convey ideas to the listeners. With a briefing, the speaker presents a concise array of facts to the listeners who normally do not expect elaboration of supporting material. During a formal lecture, the speaker’s purpose is to inform, to persuade, or to entertain with little or no verbal participation by the students. When using a teaching lecture, the instructor plans and delivers an oral presentation in a manner that allows some participation by the students and helps direct them toward the desired learning outcomes.

Teaching Lecture
The teaching lecture is favored by aviation instructors because it allows some active participation by the students.

Preparing the Teaching Lecture
• Establishing the objective and desired outcomes
• Researching the subject
• Organizing the material
• Planning productive classroom activities

In the teaching lecture, simple rather than complex words should be used whenever possible. Picturesque slang and free-and-easy colloquialisms, if they suit the subject, can add variety and vividness to a teaching lecture. The instructor should not, however, use substandard English. [This paragraph is the source of several questions on the Knowledge Test.]

Types of Delivery
• Reading from a typed or written manuscript
• Reciting memorized material without the aid of a manuscript
• Speaking extemporaneously from an outline
• Speaking impromptu without preparation

Use of Notes
An instructor who is thoroughly prepared or who has made the presentation before can usually speak effectively without notes. If the lecture has been carefully prepared, and the instructor is completely familiar with the outline, there should be no real difficulty.

Formal Versus Informal Lectures
The lecture may be conducted in either a formal or an informal manner. The informal lecture includes active student participation. The primary consideration in the lecture method, as in all other teaching methods, is the achievement of desired learning outcomes. Learning is best achieved if students participate actively in a friendly, relaxed atmosphere. Therefore, the use of the informal lecture is encouraged. At the same time, it must be realized that a formal lecture is still to be preferred on some subjects and occasions, such as lectures introducing new subject matter and for explaining the necessary background information. [Exam question.]

Advantages and Disadvantages of the Lecture
There are a number of advantages to lectures. For example, a lecture is a convenient way to instruct large groups. If necessary, a public address system can be used to amplify the speaker’s voice. Lectures can be used to present information that would be difficult for the students to get in other ways, particularly if the students do not have the time required for research, or if they do not have access to reference material. Lectures also can usefully and successfully supplement other teaching devices and methods.

In a lecture, the instructor can present many ideas in a relatively short time. Facts and ideas that have been logically organized can be concisely presented in rapid sequence. Lecturing is unquestionably the most economical of all teaching methods in terms of the time required to present a given amount of material. [Exam question.]

The lecture method is useful in providing information, it is not an effective method of learning large amounts of information in a short time. [Exam question.]

An instructor who introduces some form of active student participation in the middle of a lecture greatly increases student retention.

Discussion Method
The discussion method modifies the pure lecture form by using lecture and then discussion to actively integrate the student into the learning process. Active student participation, it also allows students to develop higher order thinking skills (HOTS). The give and take of the discussion method also helps students learn to evaluate ideas, concepts, and principles.

Guided Discussion Method
The guided discussion method relies on student possession of a level of knowledge about the topic to be discussed, either through reading prior to class or a short lecture to set up the topic to be discussed. In a guided discussion, the instructor guides the discussion with the goal of reinforcing a learning objective related to the lesson. The instructor acts as a facilitator to encourage discussion between students.

Use of Questions in a Guided Discussion
The purpose of a lead-off question is to get the discussion started. The instructor may ask a follow-up question to guide the discussion.

Questions can be identified as overhead (A question directed to the entire group in order to stimulate thought and discussion from the entire group), rhetorical (A question asked for a purpose other than to obtain the information the question asks. Usually answered by the instructor.), direct (A question used for follow-up purposes, but directed at a specific individual.), reverse (Returns the question to the same student to provide the answer.), and relay (The student question is redirected to another student.).

Characteristics of an Effective Question
• Has a specific purpose
• Is clear in meaning
• Contains a single idea
• Stimulates thought
• Requires definite answers
• Relates to previously covered information

Planning a Guided Discussion
• Select a topic the students can profitably discuss.
• Establish a specific lesson objective with desired learning outcomes.
• Conduct adequate research to become familiar with the topic.
• Organize the main and subordinate points of the lesson in a logical sequence.
• Plan at least one lead-off question for each desired learning outcome.

The introduction should include an attention element, a motivation element, and an overview of key points. Discussion questions should be easy for students to understand, put forth decisively by the instructor, and followed by silence. In the conclusion the instructor should tie together the various points or topics discussed, and show the relationships between the facts brought forth and the practical application of these facts.

Problem-Based Learning

In 1966, the McMaster University School of Medicine in Canada pioneered a new approach to teaching and curriculum design called problem-based learning (PBL). In the intervening years, PBL has helped shift the focus of learning from an instructor-centered approach to a student-centered approach.

PBL starts with a carefully constructed problem to which there is no single solution.
Effective problems:
• Relate to the real world so students want to solve them.
• Require students to make decisions.
• Are open ended and not limited to one correct answer.
• Are connected to previously learned knowledge as well as new knowledge.
• Reflect lesson objective(s).
• Challenge students to think critically.

Teaching Higher Order Thinking Skills (HOTS)
Risk management, ADM, automation management, situational awareness, and Controlled Flight into Terrain (CFIT) awareness are the skills encompassed by HOTS.

Types of Problem-Based Instruction
Involves an incentive or need to solve the problem, a decision on how to find a solution, a possible solution, an explanation for the reasons used to reach that solution, and then reflection on the solution.

Scenario-Based Training Method (SBT)
SBT uses a highly structured script of real-world experiences to address aviation training objectives in an operational environment.

Collaborative Problem-Solving Method
The collaborative problem-solving method combines collaboration with problem solving when the instructor provides a problem to a group who then solves it.

Case Study Method
A case study is a written or oral account of a real world situation that contains a message that educates the student.

Electronic Learning (E-Learning)

Electronic learning or e-learning has become an umbrella term for any type of education that involves an electronic component such as the Internet, a network, a stand-alone computer, CD/DVDs, video conferencing, websites, or e- mail in its delivery.

Cooperative or Group Learning Method

Cooperative or group learning organizes students into small groups who can work together to maximize their own and each other’s learning.

Demonstration-Performance Method

Best used for the mastery of mental or physical skills that require practice, the demonstration-performance method is based on the principle that people learn by doing.
• Explanation
• Demonstration
• Student Performance
• Instructor Supervision
• Evaluation

Drill and Practice Method

Connections are strengthened with practice. It promotes learning through repetition because those things most often repeated are best remembered.

Conclusion
A successful instructor needs to be familiar with as many teaching methods as possible. Although lecture and demonstration-performance may be the methods used most often, being aware of other methods and teaching tools such as guided discussion, cooperative learning, and computer- assisted learning better prepares an instructor for a wide variety of teaching situations.

Application of the Lesson

Application is student use of the instructor’s presented material. It is very important that each student perform the maneuver or operation the right way the first few times to establish a good habit. Faulty habits are difficult to correct and must be addressed as soon as possible.

Assessment of the Lesson

Before the end of the instructional period, the instructor should review what has been covered during the lesson and require the students to demonstrate how well the lesson objectives have been met.

Instructional Aids and Training Technologies

Instructional aids are devices that assist an instructor in the teaching-learning process. Instructional aids are not self-supporting; they support, supplement, or reinforce what is being taught. [Exam question.]

Instructional Aid Theory
• During the communicative process, the sensory register of the memory acts as a filter. As stimuli are received, the individual’s sensory register works to sort out the important bits of information from the routine or less significant bits. Within seconds, what is perceived as the most important information is passed to the working or short-term memory where it is processed for possible storage in the long-term memory. This complex process is enhanced by the use of appropriate instructional aids that highlight and emphasize the main points or concepts.

• The working or short-term memory functions are limited by both time and capacity. Therefore, it is essential that the information be arranged in useful bits or chunks for effective coding, rehearsal, or recording. Carefully selected charts, graphs, pictures, or other well-organized visual aids are examples of items that help the student understand, as well as retain, essential information.

• Ideally, instructional aids should be designed to cover the key points and concepts. In addition, the coverage should be straightforward and factual so it is easy for students to remember and recall. Generally, instructional aids that are relatively simple are best suited for this purpose.

Reasons for Use of Instructional Aids
In addition to helping students remember important information, instructional aids have other advantages. When properly used, they help gain and hold the attention of students.

Clearly, a major goal of all instruction is for the student to be able to retain as much knowledge of the subject as possible, especially the key points. …a show a 10 to 15 percent increase in retention, to more optimistic results in which retention is increased by as much as 80 percent. Words or terms used in an instructional aid should be carefully selected to convey the same meaning for the student as they do for the instructor.

Another use for instructional aids is to clarify the relationships between material objects and concepts. When relationships are presented visually, they often are much easier to understand.

Instructional aids can help to teach more and more in a smaller time frame. [Exam question.]

Guidelines for Use of Instructional Aids
• Clearly establish the lesson objective.
• Gather the necessary data by researching for support material.
• Organize the material into an outline or a lesson plan.
• Select the ideas to be supported with instructional aids.
[Exam question.]

Instructional Aids

Types of Instructional Aids
Chalk or Marker Board
Supplemental Print Material
Enhanced Training Materials
e.g. training syllabi; pilot training is a maneuvers guide or handbook which includes the PTS; lists of typical examiner questions.
Projected Material
Video
Interactive CDs and DVDs
Computer-Assisted Learning (CAL)
Models, Mock-ups, and Cut-Aways

Test Preparation Material

While test preparation materials may be effective in preparing students for FAA tests, the danger is that students may learn to pass a given test, but fail to learn other critical information essential to safe piloting and maintenance practices. FAA inspectors and designated examiners have found that student applicants often exhibit a lack of knowledge during oral questioning, even though many have easily passed the FAA knowledge test. [Exam question.]

Future Developments

Spoken input, VR, simulators.

Chapter Summary

As indicated by this discussion, the teaching process organizes the material an instructor wishes to teach in such a way that the learner understands what is being taught. An effective instructor uses a combination of teaching methods as well as instructional aids to achieve this goal. By being well prepared, an effective instructor presents and applies lesson material, and also periodically assesses how well the learner is learning. An effective instructor never stops learning. He or she maintains currency in the subject matter being taught, as well as how to teach it by reading professional journals and other aviation publications, many of which can be viewed or purchased via the Internet, another source of valuable aviation information for professional instructors.

Chapter 5: Assessment

Assessment is an essential and continuous (ongoing) component of the teaching and learning processes. No skill is more important to an instructor than the ability to continuously analyze, appraise, and judge a student’s performance.

Assessment Terminology

Assessment is the process of gathering measurable information to meet evaluation needs. Assessment involves both judgment by the instructor and collaboration with the student during the evaluation stage.

Traditional assessment often involves the kind of written testing (e.g., multiple choice, matching) and grading that is most familiar to instructors and students. Traditional assessment is more likely to be used to judge, or evaluate, the student’s progress at the rote and understanding levels of learning.

Authentic assessment requires the student to demonstrate not just rote and understanding, but also the application and correlation levels of learning. Authentic assessment requires the student to exhibit in-depth knowledge by generating a solution instead of merely choosing a response.

The terms “criteria/criterion” and “standard” are often used interchangeably. They refer to the characteristics that define acceptable performance on a task. Another term used in association with authentic assessment is “rubric.” A rubric is a guide used to score performance assessments in a reliable, fair, and valid manner. It is generally composed of dimensions for judging student performance, a scale for rating performances on each dimension, and standards of excellence for specified performance levels.

Diagnostic assessments are used to assess student knowledge or skills prior to a course of instruction.
Formative assessments, which are not graded, are used as a wrap-up of the lesson and to set the stage for the next lesson.
Summative assessments are used periodically throughout the training to measure how well learning has progressed to that point.

Purpose of Assessment

An effective assessment provides critical information to both the instructor and the student. A well- designed and effective assessment process contributes to the development of aeronautical decision-making and judgment skills by helping develop the student’s ability to evaluate his or her own knowledge and performance accurately.

General Characteristics of Effective Assessment

In order to provide direction and raise the student’s level of performance, assessment must be factual, and it must be aligned with the completion standards of the lesson.

Objective
The effective assessment is objective, and focused on student performance.

Flexible
The instructor must evaluate the entire performance of a student in the context in which it is accomplished.

Acceptable
The student must accept the instructor in order to accept his or her assessment willingly. Students must have confidence in the instructor’s qualifications, teaching ability, sincerity, competence, and authority.

Comprehensive
An effective assessment covers strengths as well as weaknesses. The instructor’s task is to determine how to balance the two.

Constructive
Praise can be very effective in reinforcing and capitalizing on things that are done well, in order to inspire the student to improve in areas of lesser accomplishment.

Organized
Almost any pattern is acceptable, as long as it is logical and makes sense to the student. Sometimes an assessment can profitably begin at the point at which a demonstration failed, and work backward through the steps that led to the failure.… Breaking the whole into parts, or building the parts into a whole, is another possible organizational approach.

Thoughtful
An effective assessment reflects the instructor’s thoughtfulness toward the student’s need for self-esteem, recognition, and approval.

Specific
At the conclusion of an assessment, students should have no doubt about what they did well and what they did poorly and, most importantly, specifically how they can improve.

Traditional Assessment

As defined earlier, traditional assessment generally refers to written testing, such as multiple choice, matching, true/false, fill in the blank, etc.

Characteristics of a Good Written Assessment (Test)
A test item measures a single objective, and calls for a single response.

Reliability is the degree to which test results are consistent with repeated measurements.

Validity is the extent to which a test measures what it is supposed to measure, and it is the most important consideration in test evaluation.

Usability refers to the functionality of tests. The wording of the test items needs to be clear and concise. Graphics, charts, and illustrations appropriate to the test items must be clearly drawn, and the test should be easily graded.

Objectivity describes singleness of scoring of a test. It is nearly impossible to prevent an instructor’s own knowledge and experience in the subject area, writing style, or grammar from affecting the grade awarded in an essay question. Selection-type test items, such as true/false or multiple choice, are much easier to grade objectively.

Comprehensiveness is the degree to which a test measures the overall objectives. The instructor has to make certain the evaluation includes a representative and comprehensive sampling of the objectives of the course.

Discrimination is the degree to which a test distinguishes the difference between students.

Authentic Assessment

Authentic assessment is a type of assessment in which the student is asked to perform real-world tasks, and demonstrate a meaningful application of skills and competencies. Students must generate responses from skills and concepts they have learned. By using open-ended questions and established performance criteria, authentic assessment focuses on the learning process, enhances the development of real-world skills, encourages higher order thinking skills (HOTS), and teaches students to assess their own work and performance.

Collaborative Assessment
The instructor begins by using a four-step series of open-ended questions to guide the student through a complete self-assessment.

Replay—ask the student to verbally replay the flight or procedure.

Reconstruct—the reconstruction stage encourages the student to learn by identifying the key things that he or she would have, could have, or should have done differently during the flight or procedure.

Reflect—insights come from investing perceptions and experiences with meaning, requiring reflection on the events.

Redirect—the final step is to help the student relate lessons learned in this session to other experiences, and consider how they might help in future sessions.

The purpose of the self-assessment is to stimulate growth in the student’s thought processes and, in turn, behaviors. The self-assessment is followed by an in-depth discussion between the instructor and the student, which compares the instructor’s assessment to the student’s self-assessment.

The collaborative assessment process in student-centered grading uses two broad rubrics: one that assesses the student’s level of proficiency on skill-focused maneuvers or procedures, and one that assesses the student’s level of proficiency on single-pilot resource management (SRM), which is the cognitive or decision-making aspect of flight training.

Maneuver or Procedure “Grades”
Describe—the student is able to describe the physical characteristics and cognitive elements of the scenario activities, but needs assistance to execute the maneuver or procedure successfully.

Explain—the student is able to describe the scenario activity and understand the underlying concepts, principles, and procedures that comprise the activity, but needs assistance to execute the maneuver or procedure successfully.

Practice—the student is able to plan and execute the scenario. Coaching, instruction, and/or assistance will correct deviations and errors identified by the instructor.

Perform—the student is able to perform the activity without instructor assistance. The student will identify and correct errors and deviations in an expeditious manner. At no time will the successful completion of the activity be in doubt.

Not observed—any event not accomplished or required.

Single-Pilot Resource Management (SRM) “Grades”
Explain—the student can verbally identify, describe, and understand the risks inherent in the flight scenario, but needs to be prompted to identify risks and make decisions.

Practice—the student is able to identify, understand, and apply SRM principles to the actual flight situation. Coaching, instruction, and/or assistance quickly corrects minor deviations and errors identified by the instructor. The student is an active decision maker.

Manage-Decide—the student can correctly gather the most important data available both inside and outside the flight deck, identify possible courses of action, evaluate the risk inherent in each course of action, and make the appropriate decision. Instructor intervention is not required for the safe completion of the flight.

Choosing an Effective Assessment Method

When deciding how to assess student progress, aviation instructors can follow a four-step process.
• Determine level-of-learning objectives.
• List indicators of desired behaviors.
• Establish criterion objectives.
• Develop criterion-referenced test items.

Determine Level-of-Learning Objectives
The first step in developing an appropriate assessment is to state the individual objectives as general, level-of-learning objectives.

List Indicators/Samples of Desired Behaviors
The second step is to list the indicators or samples of behavior that give the best indication of the achievement of the objective.

Establish Criterion Objectives
The next step in the test development process is to define criterion (performance-based) objectives.

Develop Criterion-Referenced Assessment Items
The last step is to develop criterion-referenced assessment items.

Practical tests for maintenance technicians and pilots are criterion-referenced tests. The practical tests, defined in the Practical Test Standards (PTS), are criterion referenced because the objective is for all successful applicants to meet the high standards of knowledge, skill, and safety required by the regulations.

Critiques and Oral Assessments

Used in conjunction with either traditional or authentic assessment, the critique is an instructor-to-student assessment. These methods can also be used either individually, or in a classroom setting.

An effective critique considers good as well as bad performance, the individual parts, relationships of the individual parts, and the overall performance. A critique can and usually should be as varied in content as the performance being evaluated. A critique may be oral, written, or both. It should come immediately after a student’s performance, while the details of the performance are easy to recall.

Instructor/Student Critique
The instructor leads a group discussion in an instructor/ student critique in which members of the class are invited to offer criticism of a performance.

Student-Led Critique
The instructor can specify the pattern of organization and the techniques or can leave it to the discretion of the student leader.

Small Group Critique
The class is divided into small groups, each assigned a specific area to analyze. Each group must present its findings to the class. It is desirable for the instructor to furnish the criteria and guidelines. The combined reports from the groups can result in a comprehensive assessment.

Individual Student Critique by Another Student
The instructor may require another student to present the entire assessment. A variation is for the instructor to ask a number of students questions about the manner and quality of performance.

Self-Critique
A student critiques personal performance in a self-critique. Like all other methods, a self-critique must be controlled and supervised by the instructor.

Written Critique
A written critique has three advantages. First, the instructor can devote more time and thought to it than to an oral assessment in the classroom. Second, students can keep written assessments and refer to them whenever they wish. Third, when the instructor requires all students to write an assessment of a performance, the student-performer has the permanent record of the suggestions, recommendations.

Oral Assessment
The most common means of assessment is direct or indirect oral questioning of students by the instructor.

Fact questions are based on memory or recall. This type of question usually concerns who, what, when, and where. HOTS questions involve why or how, and require the student to combine knowledge of facts with an ability to analyze situations, solve problems, and arrive at conclusions.

Proper quizzing by the instructor can have a number of desirable results:
• Reveals the effectiveness of the instructor’s training methods
• Checks student retention of what has been learned
• Reviews material already presented to the student
• Can be used to retain student interest and stimulate thinking
• Emphasizes the important points of training
• Identifies points that need more emphasis
• Checks student comprehension of what has been learned
• Promotes active student participation, which is important to effective learning

Characteristics of Effective Questions
Prepared questions merely serve as a framework, and as the lesson progresses, should be supplemented by such impromptu questions as the instructor considers appropriate.

To be effective, questions must:
• Apply to the subject of instruction.
• Be brief and concise, but also clear and definite.
• Be adapted to the ability, experience, and stage of training of the students.
• Center on only one idea (limited to who, what, when, where, how, or why, not a combination).
• Present a challenge to the students.

Types of Questions To Avoid
Effective quizzing does not ever include yes/no questions such as “Do you understand?” or “Do you have any questions?” Also avoid the following types of questions: puzzle, oversize, toss-up, bewilderment, trick questions, irrelevant questions.

Answering Student Questions
• Be sure that you clearly understand the question before attempting to answer.
• Display interest in the student’s question and frame an answer that is as direct and accurate as possible.
• After responding, determine whether or not the student is satisfied with the answer.

Chapter Summary

This chapter has offered the aviation instructor techniques and methods for assessing how, what, and how well a student is learning. Well-designed assessments define what is worth knowing, thereby improving student learning. Since today’s students want to know the criteria by which they are assessed, as well as practical and specific feedback, it is important for aviation instructors to be familiar with the different types of assessments available for monitoring student progress throughout a course of training, and how to select the most appropriate assessment method.

Chapter 6: Planning Instructional Activity

The key to developing well- planned and organized aviation instruction includes using lesson plans and a training syllabus that meet all regulatory certification requirements.

Course of Training

An instructor plans instructional content around the course of training by determining the objectives and standards, which in turn determine individual lesson plans, test items, and levels of learning.

Blocks of Learning

After the overall training objectives have been established, the next step is the identification of the blocks of learning which constitute the necessary parts of the total objective. A student can master the segments or blocks individually and can progressively combine these with other related segments until their sum meets the overall training objectives.

The blocks of learning should be fairly consistent in scope. They should represent units of learning which can be measured and evaluated—not a sequence of periods of instruction.

Training Syllabus

Aviation instructors use a training syllabus because as technology advances, training requirements become more demanding.

Syllabus Format and Content
The format and organization of the syllabus may vary, but it always should be in the form of an abstract or digest of the course of training. It should contain blocks of learning to be completed in the most efficient order. Since effective training relies on organized blocks of learning, all syllabi should stress well-defined objectives and standards for each lesson.

How To Use a Training Syllabus
Any practical training syllabus must be flexible and should be used primarily as a guide. When necessary, the order of training can and should be altered to suit the progress of the student and the demands of special circumstances.

Ground training lessons and classroom lectures concentrate on the cognitive domain of learning. A typical lesson might include defining, labeling, or listing what the student has learned so far. Many of the knowledge areas are directly or indirectly concerned with safety, ADM, and judgment. Since these subjects are associated with the affective domain of learning (emotion), instructors who find a way to stress safety, ADM, and judgment, along with the traditional aviation subjects, can favorably influence a student’s attitude, beliefs, and values.

Flight training lessons or aviation technical lab sessions also include knowledge areas, but they generally emphasize the psychomotor domain of learning because the student is “doing” something.

The flight training syllabus should include special emphasis items that have been determined to be cause factors in aircraft accidents or incidents.

Lesson Plans

A lesson plan is an organized outline for a single instructional period. It is a necessary guide for the instructor because it tells what to do, in what order to do it, and what procedure to use in teaching the material of a lesson.

A mental outline of a lesson is not a lesson plan. A lesson plan should be put into writing. Another instructor should be able to take the lesson plan and know what to do in conducting the same period of instruction. Written out, the lesson plan can be analyzed for adequacy and completeness. [Exam questions.]

Purpose of the Lesson Plan
Lesson plans are designed to assure that each student receives the best possible instruction under the existing conditions.

Characteristics of a Well-Planned Lesson
After the objective is determined, the instructor must research the subject as it is defined by the objective. Once the research is complete, the instructor determines the method of instruction and identifies a useful lesson planning format. The decision of how to organize the lesson and the selection of suitable support material come next. The final steps include assembling training aids and writing the lesson plan outline. [Exam questions.]

Unity—each lesson should be a unified segment of instruction. A lesson is concerned with certain limited objectives, which are stated in terms of desired student learning outcomes. All teaching procedures and materials should be selected to attain these objectives.

Content—each lesson should contain new material. However, the new facts, principles, procedures, or skills should be related to the lesson previously presented. A short review of earlier lessons is usually necessary, particularly in flight training.

Scope—each lesson should be reasonable in scope. A person can master only a few principles or skills at a time, the number depending on complexity. Presenting too much material in a lesson results in confusion; presenting too little material results in inefficiency.

Practicality—each lesson should be planned in terms of the conditions under which the training is to be conducted. Lesson plans conducted in an airplane or ground trainer will differ from those conducted in a classroom. Also, the kinds and quantities of instructional aids available have a great influence on lesson planning and instructional procedures.

Flexibility—although the lesson plan provides an outline and sequence for the training to be conducted, a degree of flexibility should be incorporated. For example, the outline of content may include blank spaces for add-on material, if required.

Relation to course of training—each lesson should be planned and taught so that its relation to the course objectives is clear to each student. For example, a lesson on short field takeoffs and landings should be related to both the certification and safety objectives of the course of training.

Instructional steps—every lesson, when adequately developed, falls logically into the four steps of the teaching process: preparation, presentation, application, and review and evaluation.

How To Use a Lesson Plan Properly
Use the lesson plan as a guide.
Adapt the lesson plan to the class or student.
Revise the lesson plan periodically.

Lesson Plan Formats
Each lesson should have somewhat limited objectives that are achievable within a reasonable period of time.

Commercially developed lesson plans are acceptable for most training situations, including use by flight instructor applicants during their practical tests.

The traditional type of training lesson plan with its focus on the task and maneuver or procedure continues to meet many aviation learning requirements, but as discussed earlier in the chapter, it is being augmented by more realistic and fluid forms of problem-based learning such as SBT. For the CFI, this type of training does not preclude traditional maneuver- based training. Rather, flight maneuvers are integrated into the flight scenarios and conducted as they would occur in the real world. Those maneuvers requiring repetition are still taught during concentrated settings; once learned, they are then integrated into realistic flight situations.

Scenario-Based Training (SBT)

Since humans develop cognitive skills through active interaction with the world, an effective aviation instructor uses the maneuver- or procedure-based approach of the PTS but presents the objectives in a scenario situation.

Advanced avionics have contributed to a shift in the focus of aviation training to include aeronautical decision-making (ADM) and risk management. For the pilot, this is called Single-Pilot Resource Management (SRM). Since SRM training requires the student or transitioning pilot to practice the decision-making process in real-world situations, it combines traditional task and maneuver-based training with SBT to enhance ADM, risk management, and SRM skills without compromising basic stick and rudder skills.

Duties, Responsibilities, and Authority of the Aviation Instructor

1. Orient new learners to the SBT approach.
2. Help the learner become a confident planner and a critical evaluator of his or her own performance.
3. Help the learner understand the knowledge requirements present in real world applications.
4. Diagnose learning difficulties and help the individual overcome them.
5. Evaluate student progress and maintain appropriate records.
6. Provide continuous review of student learning.

Reality is the ultimate learning situation and SBT attempts to get as close as possible to this ideal. It addresses learning that occurs in a context or situation. It is based on the concept of situated cognition, which is the idea that knowledge cannot be known and fully understood independent of its context. In other words, humans learn better, the more realistic the situation is and the more they are counted on to perform.

Scenario Based Training

Single-Pilot Resource Management

SRM is the art and science of managing all the resources (both on-board the aircraft and from outside sources) available to a single pilot (prior and during flight) to ensure that the successful outcome of the flight is never in doubt.

Chapter Summary

As indicated by this chapter, it is possible to develop well-planned and organized instruction by using a training syllabus and lesson plans that meet all regulatory certification requirements. By identifying and incorporating “blocks of learning” into the teaching of objectives, the instructor can plan lessons that build on prior knowledge. Maneuver and/or procedure training coupled with SBT will help the aviation instructor train professional aviators and technicians who are able to gather and analyze information to aid in making good aeronautical decisions and decrease risk factors, leading to a successful flight or maintenance outcome.

Aviation Instructor’s Handbook FAA-H-8083-9A: Chapters 1-3

September 17th, 2018

To get your CFI or ground instructor certificate you need to pass the Fundamentals of Instructing written test. The questions are taken from this FAA publication. There is a lot of good information in the book as well as some long-debunked ideas. The old test that was published by the FAA has lots of questions with more than one right answer and you are supposed to remember from the reading which one is the “best” answer. After reading the text once, I took a couple of practice exams and scored 7/10 or 8/10 so I could probably pass the test. However, I purchased the Gleim book so that I can be sure of passing on the first try. And I decided to write up some notes from the book. This is the first of three summaries, the glossary, notes on all of the lists that you need to memorize, and a list of questions where the “correct” answer doesn’t make any sense.

There are lots of questions on the exam that require you to remember which items belong to a list of items e.g. Hazardous attitudes, PAVE, IMSAFE. I don’t really care about any of them and can never remember what belongs with which acronym so I write them down and review them before taking the exam. Some of them are relevant though and are worth memorizing. All of them are included in the Lists post.

The text below is copied directly from the handbook. My comments are in brackets. [ … ]. Highlights are for things that I can’t remember or that are supposedly on the test and need to be memorized,.

Chapter I: Human Behavior

Learning is the acquisition of knowledge or understanding of a subject or skill through education, experience, practice, or study. A change of behavior results from learning.

To successfully bring about learning, the instructor must know why people act the way they do, how people learn, and then use this understanding to teach.

Definitions of Human Behavior

[These aren’t definitions as much as ways of thinking about behaviour.]

A complex topic, human behavior is a product both of innate human nature and of individual experience and environment. Human behavior is also defined as the result of attempts to satisfy certain needs.

Another definition of human behavior focuses on the typical life course of humans. This approach emphasizes human development or the successive phases of growth in which human behavior is characterized by a distinct set of physical, physiological, and behavioral features.

Personality Types
[Personality testing is mostly nonsense. You might want to learn it for the test, but ignore it when teaching.]

Human Needs and Motivation

Henry A. Murray, one of the founders of personality psychology who was active in developing a theory of motivation, identified a list of core psychological needs in 1938. He described these needs as being either primary (based on biological needs, such as the need for food) or secondary (generally psychological, such as the need for independence). Murray believed the interplay of these needs produce distinct personality types and are internal influences on behavior.

Abraham Maslow who also studied human needs, motivation, and personality. According to Maslow, human needs go beyond the obvious physical needs of food and shelter to include psychological needs, safety and security, love and belongingness, self esteem, and self actualization to achieve one’s goals.

Human Needs That Must Be Met To Encourage Learning
Physiological
These are biological needs. They consist of the need for air, food, water, and maintenance of the human body.

Security
Security needs are about keeping oneself from harm.

Belonging
Maslow states that people seek to overcome feelings of loneliness and alienation. This involves both giving and receiving love, affection, and the sense of belonging.

Esteem
Humans get esteem in two ways: internally or externally. Internally, a person judges himself or herself worthy by personally defined standards. High self-esteem results in self-confidence, independence, achievement, competence, and knowledge.

Cognitive and Aesthetic
In later years, Maslow added cognitive (need to know and understand) and aesthetic (the emotional need of the artist) needs to the pyramid.

Self-Actualization
When all of the foregoing needs are satisfied, then and only then are the needs for self-actualization activated. Maslow describes self-actualization as a person’s need to be and do that which the person was “born to do.”

Human Nature and Motivation

Douglas McGregor set out two opposing assumptions about human nature and motivation in 1960.
Theory X assumes that management’s role is to coerce and control employees because people need control and direction.

Theory X
• People have an inherent dislike for work and will avoid it whenever possible.
• People must be coerced, controlled, directed, or threatened with punishment in order to get them to achieve the organizational objectives.
• People prefer to be directed, do not want responsibility, and have little or no ambition.
• People seek security above all else.

Theory Y
• Work is as natural as play and rest. The average person does not inherently dislike work. Depending on conditions, work may be a source of satisfaction and, if so, it is performed voluntarily. On the other hand, when work is a form of punishment, it is avoided, if possible.
• People exercise self-direction if they are committed to the goals (they are not lazy).
• Commitment to goals relates directly to the rewards associated with their achievement.
• People learn to accept and seek responsibility. Shirking responsibility and lack of ambition are not inherent in human nature, but are usually the consequences of experience.
• Creativity, ingenuity, and imagination are widely distributed among the population. People are capable of using these abilities to solve problems.
• People have potential.

Human Factors That Inhibit Learning

Defense Mechanisms
Defense mechanisms can be biological or psychological. The biological defense mechanism is a physiological response that protects or preserves organisms. The ego defense mechanism is an unconscious mental process to protect oneself from anxiety, unpleasant emotions, or to provide a refuge from a situation with which the individual cannot currently cope.

Repression is the defense mechanism whereby a person places uncomfortable thoughts into inaccessible areas of the unconscious mind.

Denial is a refusal to accept external reality because it is too threatening.

Compensation is a process of psychologically counterbalancing perceived weaknesses by emphasizing strength in other areas.

Through projection, an individual places his or her own unacceptable impulses onto someone else.

Rationalization is a subconscious technique for justifying actions that otherwise would be unacceptable.

In reaction formation a person fakes a belief opposite to the true belief because the true belief causes anxiety.

Fantasy occurs when a student engages in daydreams about how things should be rather than doing anything about how things are.

Displacement results in an unconscious shift of emotion, affect, or desire from the original object to a more acceptable, less threatening substitute.

Student Emotional Reactions

Anxiety is a feeling of worry, nervousness, or unease, often about something that is going to happen, typically something with an uncertain outcome. It results from the fear of anything, real or imagined, which threatens the person who experiences it, and may have a potent effect on actions and the ability to learn from perceptions.

Normal Reactions to Stress
The affected individual thinks rationally, acts rapidly, and is extremely sensitive to all aspects of the surroundings.

Abnormal Reactions to Stress
…response to anxiety or stress may be completely absent or at least inadequate. Their responses may be random or illogical, or they may do more than is called for by the situation.

• Inappropriate reactions, such as extreme over-cooperation, painstaking self-control, inappropriate laughter or singing, and very rapid changes in emotions.
• Marked changes in mood on different lessons, such as excellent morale followed by deep depression.
• Severe anger directed toward the flight instructor, service personnel, and others.

[These are labelled as Normal and Abnormal, but are probably more accurately described as Appropriate and Inappropriate. Or actions that resolve the cause of the stress or actions that exacerbate the stress.]

Flight Instructor Actions Regarding Seriously Abnormal Students
…a flight instructor has the personal responsibility of assuring that such a person does not continue flight training or become certificated as a pilot.

Teaching the Adult Student

[This section is mostly unsubstantiated generalizations. A couple of things are true though.]
• Adults who are motivated to seek out a learning experience do so primarily because they have a use for the knowledge or skill being sought. Learning is a means to an end, not an end in itself.
• Adults have accumulated a foundation of life experiences and knowledge and draw upon this reservoir of experience for learning.

Chapter 2:The Learning Process

What Is Learning?
Learning can be defined in many ways:
• A change in the behavior of the learner as a result of experience. The behavior can be physical and overt, or it can be intellectual or attitudinal.
• The process by which experience brings about a relatively permanent change in behavior.
• The change in behavior that results from experience and practice.
• Gaining knowledge or skills, or developing a behavior, through study, instruction, or experience.
• The process of acquiring knowledge or skill through study, experience, or teaching. It depends on experience and leads to long-term changes in behavior potential. Behavior potential describes the possible behavior of an individual (not actual behavior) in a given situation in order to achieve a goal.
• A relatively permanent change in cognition, resulting from experience and directly influencing behavior.

The Framework for Learning
Research into how people learn gained momentum with the Swiss scientist and psychologist Jean Piaget, who studied the intellectual development of children in the early twentieth century.

Learning Theory

Learning theory is a body of principles advocated by psychologists and educators to explain how people acquire skills, knowledge, and attitudes.

Behaviorism is a school of psychology that explains animal and human behavior entirely in terms of observable and measurable responses to stimuli.

Cognitive theory focuses on what is going on inside the mind. It is more concerned with cognition (the process of thinking and learning)—knowing, perceiving, problem-solving, decision-making, awareness, and related intellectual activities—than with stimulus and response. Learning is not just a change in behavior; it is a change in the way a learner thinks, understands, or feels.

For [John] Dewey, the concept of reflective thought carried deep meaning. He saw reflection as a process that moves a learner from one experience into the next with deeper understanding of its relationships with and connections to other experiences and ideas. Thus, reflection leads the learner from the unclear to the clear.

Jean Piaget, who spent 50 years studying how children develop intellectually, became a major figure in the school of cognitive thought. His research led him to conclude there is always tension between assimilation (old ideas meeting new situations) and accommodation (changing the old ideas to meet the new situations). The resolution of this tension results in intellectual growth.

Jerome Bruner became interested in how intellectual development related to the process of learning, His research led him to advocate learning from the known to the unknown, or from the concrete to the abstract, because humans best learn when relating new knowledge to existing knowledge.

In the mid-1900s, a group of educators led by Benjamin Bloom tried to classify the levels of thinking behaviors thought to be important in the processes of learning.… Bloom’s Taxonomy of the Cognitive Domain is a taxonomy (a classification system according to presumed relationships) comprised of six levels of intellectual behavior and progresses from the simplest to the most complex: knowledge, comprehension, application, analysis, synthesis, and evaluation.

Information Processing Theory
Information processing theory uses a computer system as a model for human learning. The human brain processes incoming information, stores and retrieves it, and generates responses to the information. This involves a number of cognitive processes: gathering and representing information (encoding), retaining of information, and retrieving the information when needed.

Constructivism
A derivative of cognitive theory, constructivism is a philosophy of learning that can be traced to the eighteenth century. This theory holds that learners do not acquire knowledge and skills passively but actively build or construct them based on their experiences.

…humans construct a unique mental image by combining preexisting information with the information received from sense organs. Learning is the result of the learner matching new information against this preexisting information and integrating it into meaningful connections.

Higher Order Thinking Skills (HOTS)
The constructivist theory of learning explains and supports the learning of HOTS, which is commonly called aeronautical decision-making (ADM) in aviation. HOTS lie in the last three categories on Bloom’s Taxonomy of Learning: analysis, synthesis, and evaluation skills.

HOTS are taught like other cognitive skills, from simple to complex and from concrete to abstract. To teach HOTS effectively involves strategies and methods that include (1) using problem-based learning (PBL) instruction, (2) authentic problems, (3) real world problems, (4) student-centered learning, (5) active learning, (6) cooperative learning, and (7) customized instruction to meet the individual learner’s needs.

It must be remembered that critical thinking skills should be taught in the context of subject matter. Learners progress from simple to complex; therefore, they need some information before they can think about a subject beyond rote learning.

Scenario Based Training (SBT)
At the heart of HOTS [Higher Order Thinking Skills] lies scenario-based training (SBT) which is an example of the PBL [problem-based learning] instructional method and facilitates the enhancement of learning and the development and transference of thinking skills. SBT provides more realistic decision-making opportunities because it presents tasks in an operational environment; it correlates new information with previous knowledge, and introduces new information in a realistic context.

The best use of scenarios draws the learner into formulating possible solutions, evaluating the possible solutions, deciding on a solution, judging the appropriateness of that decision and finally, reflecting on the mental process used in solving the problem. It causes the learner to consider whether the decision led to the best possible outcome and challenges the learner to consider other solutions.

Perceptions

Perception involves more than the reception of stimuli from the five senses; it also involves a person giving meaning to sensations.

Factors That Affect Perception
Both internal and external factors affect an individual’s ability to perceive:
• Physical organism
• Goals and values
• Self-concept
• Time and opportunity
• Element of threat

Physical Organism

Goals and Values
Every experience and sensation, which is funneled into one’s central nervous system, is colored by the individual’s own beliefs and value structures.

Self-Concept
If a student’s experiences tend to support a favorable self-image, the student tends to remain receptive to subsequent experiences. If a student has negative experiences, which tend to contradict self-concept, there is a tendency to reject additional training. [This sounds like nonsense to me. I’d like to see some data to back up this claim.]

Time and Opportunity
…proper sequence and time are necessary.

Element of Threat
…fear adversely affects perception by narrowing the perceptual field. Confronted with threat, students tend to limit their attention to the threatening object or condition.

Insight

The mental relating and grouping of associated perceptions is called insight.

It is a major responsibility of the instructor to organize demonstrations and explanations, and to direct practice so that the student has better opportunities to understand the interrelationship of the many kinds of experiences that have been perceived.

Acquiring Knowledge

Knowledge refers to information that humans are consciously aware of and can articulate.

Memorization
A student’s first attempt to acquire knowledge about a new topic amounts to memorizing facts about steps in a procedure.

Understanding
Understanding is the ability to notice similarities and make associations between the facts and procedural steps learned.

Concept Learning
Concept learning is based on the assumption that humans tend to group objects, events, ideas, people, etc., that share one or more major attributes that set them apart. It also involves discrimination between types of things or ideas inside or outside of a concept set.

Another type of generalization is a schema (the cognitive framework that helps people organize and interpret information). Schemas can be revised by any new information and are useful because they allow people to take shortcuts in interpreting a vast amount of information.

Thorndike and the Laws of Learning

Laws of learning: the law of readiness, the law of exercise, and the law of effect, the law of primacy, the law of intensity, and the law of recency. [Better referred to as Principles.]

Readiness
The basic needs of the learner must be satisfied before they are ready or capable of learning.

Teachable moments present opportunities to convey information in a way that is relevant, effective, and memorable to the student. They occur when a learner can clearly see how specific information or skills can be used in the real world.

Effect
All learning involves the formation of connections and connections are strengthened or weakened according to the law of effect. Responses to a situation that are followed by satisfaction are strengthened; responses followed by discomfort are weakened, either strengthening or weakening the connection of learning. [Again, sounds like nonsense to me. Is there data to back this claim up? Seems to be in conflict with Intensity.]

Exercise
Practice strengthens the learning connection; disuse weakens it. Exercise is most meaningful and effective when a skill is learned within the context of a real world application.

Primacy
Primacy, the state of being first, often creates a strong, almost unshakable impression and underlies the reason an instructor must teach correctly the first time and the student must learn correctly the first time. [I’d like to see some data on this. It sounds fishy to me.]

Intensity
Real world applications (scenarios) that integrate procedures and tasks the learner is capable of learning make a vivid impression and he or she is least likely to forget the experience.

Recency
The principle of recency states that things most recently learned are best remembered.

[Not covered in the book are two more “Laws” that were added later. Wiki]

Freedom
The principle of freedom states that things freely learned are best learned.

Requirement
The law of requirement states that “we must have something to obtain or do something.” It can be an ability, skill, instrument or anything that may help us to learn or gain something.

Domains of Learning

Domains of Learning
Cognitive Domain
The four practical learning levels are rote, understanding, application, and correlation.
Rote: The key verbs which describe or measure this activity are words such as define, identify, and label.
The comprehension or understanding level puts two or more concepts together and uses verbs such as describe, estimate, or explain.

Application: consolidate old and new perceptions into an insight. Develop the skill to apply what has been learned.

The correlation level of learning, which should be the objective of aviation instruction, is that level at which the student becomes able to associate an element which has been learned with other segments or blocks of learning.

Bloom’s Taxonomy of Learning

Affective Domain
The affective domain addresses a learner’s emotions toward the learning experience. It includes feelings, values, enthusiasms, motivations, and attitudes.

Five levels: awareness, response, value, organizing, and integration.

Psychomotor Domain
The psychomotor domain is skill based and includes physical movement, coordination, and use of the motor-skill areas.

Summary of Instructor Actions
To help students acquire knowledge, the instructor should:
• Ask students to recite or practice newly acquired knowledge.
• Ask questions that probe student understanding and prompt them to think about what they have learned in different ways.
• Present opportunities for students to apply what they know to solving problems or making decisions.
• Present students with problems and decisions that test the limits of their knowledge.
• Demonstrate the benefits of understanding and being able to apply knowledge.
• Introduce new topics as they support the objectives of the lesson, whenever possible.

Characteristics of Learning

Learning is a change in behavior as a result of experience, so instruction must include a careful and systematic creation of those experiences that promote learning.

Learning

Learning Is Purposeful
In the process of learning, the student’s goals are of paramount significance. To be effective, aviation instructors need to find ways to relate new learning to the student’s goals.

Learning Is a Result of Experience
Aviation instructors are faced with the problem of providing learning experiences that are meaningful, varied, and appropriate.

Learning Is Multifaceted
Learning is multifaceted in still another way. While learning the subject at hand, students may be learning other things as well.… This type of learning is sometimes referred to as incidental, but it may have a great impact on the total development of the student.

Learning Is an Active Process
For students to learn, they need to react and respond, perhaps outwardly, perhaps only inwardly, emotionally, or intellectually.

Learning Styles

Learning styles are simply different approaches or ways of learning based on the fact that people absorb and process information in different ways. Learning style is an individual’s preference for understanding experiences and changing them into knowledge.

Controversy exists over the scientific value of learning styles as well as approaches to learning, many educational psychologists advocate their use in the learning process. [I tend to side with the people who say that the information in this section is bullshit.]

Right Brain/Left Brain
Right-brain dominance are characterized as being spatially oriented, creative, intuitive, and emotional. Those with left-brain dominance are more verbal, analytical, and objective.

Holistic/Serialist Theory
Left brain learners have preferences for how they process information. Based on information processing theory, left brain learners or serialist learners have an analytic approach to learning. Because they gain understanding in linear steps, with each step logically following the previous one, these learners need well-defined, sequential steps where the overall picture is developed slowly, thoroughly, and logically. This is a bottom-up strategy.

Right brain or holistic learners favor the holist strategy and prefer a big picture or global perspective. This is a top-down strategy and learners tend to learn in large jumps, absorbing material almost randomly without seeing connections, until suddenly “it” clicks and they get it. Global learners solve complex problems rapidly once they have grasped the big picture, but they often have difficulty explaining how they did it. This type of learner seeks overall comprehension; analogies help this learner.

Index of Learning Styles (ILS)
Classifies students as having learning preferences in sensing or intuitive, visual or verbal, active or reflective, sequential or global

Visual, Auditory, Kinesthetic Learners (VAK)
One of the most popular learning styles is based on the three main sensory receptors: vision, hearing, and touch. These are called visual, auditory, and kinesthetic learning styles (VAK).

Superlinks
In a theory proposed by Ricki Linksman, the learning style ideas discussed in the preceding paragraphs have been melded into a concept based on the VAKT (visual, auditory, kinesthetic, and tactile) learning styles plus brain hemisphere preference.

Acquiring Skill Knowledge

Skill knowledge is knowledge reflected in motor or manual skills and in cognitive or mental skills, that manifests itself in the doing of something.

Stages of Skill Acquisition
Skill acquisition (or the learning process) has three characteristic stages: cognitive, associative, and automaticity.

Cognitive Stage
The best way to prepare the student to perform a task is to provide a clear, step-by-step example.

Associative Stage
As the storage of a skill via practice continues, the student learns to associate individual steps in performance with likely outcomes.

Automatic Response Stage
As procedures become automatic, less attention is required to carry them out, so it is possible to do other things simultaneously, or at least do other things more comfortably.

Knowledge of Results
One way to make students aware of their progress is to repeat a demonstration or example and to show them the standards their performance must ultimately meet.

How To Develop Skills
Making progress toward automating a skill seems to be largely a matter of performing the skill over and over again. In skill learning, the first trials are slow and coordination is lacking. Mistakes are frequent, but each trial provides clues for improvement in subsequent trials.

Learning Plateau

Learning Plateaus
Learning plateaus are a normal part of the learning process and tend to be temporary, but instructors and students should be prepared for them.

Types of Practice

There are three types of practice, each of which yields particular results in acquiring skills: deliberate, blocked, and random.

Deliberate Practice
During deliberate practice, the student practices specific areas for improvement and receives specific feedback after practice.

Blocked Practice
Blocked practice is practicing the same drill until the movement becomes automatic. While blocked practice enhances current performance, it does not improve either concept learning or retrieval from long-term memory.

Random Practice
Random practice mixes up the skills to be acquired throughout the practice session. This type of practice leads to better retention because by performing a series of separate skills in a random order, the student starts to recognize the similarities and differences of each skill which makes it more meaningful.

Evaluation Versus Critique

In the initial stages of skill acquisition, practical suggestions are more valuable to the student than a grade. Early evaluation is usually teacher oriented. It provides a check on teaching effectiveness, can be used to predict eventual student learning proficiency, and can help the teacher locate special problem areas.

Overlearning of Knowledge
Overlearning is the continued study of a skill after initial proficiency has been achieved.

Application of Skill
The final and critical question is “Can the student use what has been learned?”

Summary of Instructor Actions
To help students acquire skills, the instructor should:
• Explain that the key to acquiring and improving any skill is continued practice.
• Monitor student practice of skills and provide immediate feedback.
• Avoid conversation and other distractions when students are practicing individual skills.
• Explain that learning plateaus are common and that continued practice leads to continued improvement.

Putting It All Together

Multitasking
Multitasking is the simultaneous execution of two or more tasks. In aviation, multitasking involves two different abilities: attention switching and simultaneous performance.

Attention Switching
Continuously switching attention back and forth between two or more tasks is attention switching.

Simultaneous Performance
Performing several tasks at once, or simultaneous performance, is the second type of multitasking. This type of multitasking becomes possible when no bottlenecks are present and when one or more of the tasks being performed are skills developed to the point of being automatic.

Learning To Multitask
Before students are asked to perform several tasks at once, instructors should ensure that the student has devoted enough time to study and practice such that the individual tasks can be performed reasonably well in isolation.

Distractions and Interruptions
A distraction is an unexpected event that causes the student’s attention to be momentarily diverted. Students must learn to decide whether or not a distraction warrants further attention or action on their part.

An interruption is an unexpected event for which the student voluntarily suspends performance of one task in order to complete a different one.

[Examples of distractions are: light chop, radio calls to other aircraft, passenger chatting. Examples of interruptions are: radio calls to you, reaching an assigned or intended altitude, fuel truck arriving in the middle of pre-flight.]

Fixation and Inattention
Fixation occurs when a student becomes absorbed in performing one task to the exclusion of other tasks.

Inattention occurs when a student fails to pay attention to a task that is important. Inattention is sometimes a natural by-product of fixation.

How To Identify Fixation or Inattention Problems
One way for instructors to identify problems with fixation and inattention is to try and follow where students look.

Scenario-Based Training

Research and practical experience have demonstrated the usefulness of practicing in realistic scenarios—ones that resemble the environment in which knowledge and skills are later used.

A good scenario:
• Has a clear set of objectives.
• Is tailored to the needs of the student.
• Capitalizes on the nuances of the local environment.

The Learning Route to Expertise

Cognitive Strategies
Cognitive strategies refer to the knowledge of procedures or knowledge about how to do something in contrast with the knowledge of facts.

…study and identify the strategies that experts use and then teach these strategies to the students.

Problem-Solving Tactics
Problem-solving tactics are specific actions intended to get a particular result, and this type of knowledge represents the most targeted knowledge in the expert’s arsenal.

Awareness of Existence of Unknowns
An important aspect of an expert’s knowledge is an awareness of what he or she does not know.

Summary of Instructor Actions
To help students exercise their knowledge and skills in a concerted fashion, the instructor should:
• Explain the two types of multitasking (Attention Switching and Simultaneous Performance) and give examples of each type.
• Ensure that individual skills are reasonably well-practiced before asking students to perform several tasks at once.
• Teach students how to deal with distractions and interruptions and provide them with opportunities to practice.
• Point out fixation and inattention when it occurs.
• Devise scenarios that allow students to use their knowledge and skill to solve realistic problems and make decisions.
• Explain to the student that continued practice with the goal of improving leads to continued improvement.

Errors

Kinds of Error
There are two kinds of error: slip and mistake.

Slip
A slip occurs when a person plans to do one thing, but then inadvertently does something else. Slips are errors of action.

Mistake
A mistake occurs when a person plans to do the wrong thing and is successful. Mistakes are errors of thought.

Reducing Error
Learning and Practicing
Taking Time
Checking for Errors
Using Reminders
Developing Routines
Raising Awareness
Another line of defense against errors is to raise one’s awareness when operating in conditions under which errors are known to happen (e.g., changes in routine, time pressure), or in conditions under which defenses against errors have been compromised (e.g., fatigue, lack of recent practice).

Error Recovery
Given that the occasional error is inevitable, it is a worthwhile exercise to practice recovering from commonly made errors, or those that pose serious consequences.

Learning From Error
When a student makes an error, it is useful to ask the student to consider why the error happened, and what could be done differently to prevent the error from happening again in the future.

Summary of Instructor Actions
To help students learn from errors they make and be prepared for them in the future, an instructor should:
• Explain that pilots and mechanics at all levels of skill and experience make occasional errors.
• Explain that the magnitude and frequency of errors tend to decrease as skill and experience increases.
• Explain the difference between slips and mistakes and provide examples of each.
• Explain ways in which the student can help minimize errors.
• Allow the student to practice recovering from common errors.
• Point out errors when they occur and ask the student to explain why they occurred.

Motivation

Motivation prompts students to engage in hard work and affects student success. Motivation may be tangible or intangible.

Students seeking intangible rewards are motivated by the desires for personal comfort and security, group approval, and the achievement of a favorable self-image.

Maintaining Motivation

Rewarding Success
Positive feedback encourages students. Practice positive feedback frequently by:
• Praising incremental successes during training.
• Relating daily accomplishments to lesson objectives.
• Commenting favorably on student progress and level ability.

Presenting New Challenges
With each declaration of success, be sure to present students with the next challenge.

Drops in Motivation
Learning plateaus are a common source of frustration, discouragement, and decreased student motivation.

Summary of Instructor Actions
To ensure that students continue to work hard, the instructor should:
• Ask new students about their aviation training goals.
• Reward incremental successes in learning.
• Present new challenges.
• Occasionally remind students about their own stated goals for aviation training.
• Assure students that learning plateaus are normal and that improvement will resume with continued effort.

Memory

Although there is no universal agreement of how memory works, a widely accepted model has three components: sensory memory, short-term memory, and long-term memory.

Sensory Memory
Sensory memory is the part of the memory system that receives initial stimuli from the environment and processes them according to the individual’s preconceived concept of what is important.

The sensory memory processes stimuli from the environment within seconds, discards what is considered extraneous, and processes what is determined by the individual to be relevant. This is a selective process where the sensory register is set to recognize certain stimuli and immediately transmit them to the short-term memory (STM) for action. The process is called precoding.

Short-Term Memory (STM)
Short-term memory is the part of the memory system where information is stored for roughly 30 seconds, after which it may rapidly fade or be consolidated into long-term memory. Several common steps help retention in STM. These include rehearsal or repetition of the information and sorting or categorization into systematic chunks. The sorting process is usually called coding or chunking. A key limitation of STM is that it takes 5–10 seconds to properly code information and if the coding process is interrupted, that information is easily lost since it is stored for only 30 seconds. The goal of the STM is to put the information to immediate use.

STM has three basic operations: iconic memory, acoustic memory, and working memory. Iconic memory is the brief sensory memory of visual images. Acoustic memory is the encoded memory of a brief sound memory or the ability to hold sounds in STM. Of the two, acoustic memory can be held longer than iconic memory. Working memory is an active process to keep information until it is put to use.

Long-Term Memory (LTM)
Long-term memory (LTM) is relatively permanent storage of unlimited information and it is possible for memories in LTM to remain there for a lifetime.

How Usage Affects Memory
The ability to retrieve knowledge or skills from memory is primarily related to two things: (1) how often that knowledge has been used in the past; and (2) how recently the knowledge has been used.

Forgetting
Forgetting, which refers to loss of a memory, typically involves a failure in memory retrieval. The failure may be due to the decay or overwriting of information which has been temporarily stored in STM, but generally forgetting refers to loss of information from LTM.

Retrieval Failure
Retrieval failure is simply the inability to retrieve information, that tip-of-the-tongue phenomenon when a person knows the meaning of a word, or the answer to a question, but cannot retrieve it. It is also caused by the fact that sometimes people simply do not encode information well, and the information never makes it to LTM or is lost before it can attach itself to the LTM. This is sometimes referred to as failure to store.

Fading
The theory of fading or decay suggests that a person forgets information that is not used for an extended period of time, that it fades away or decays. It had been suggested that humans are physiologically preprogrammed to eventually erase data that no longer appears pertinent.

Interference
Interference theory suggests that people forget something because a certain experience has overshadowed it, or that the learning of similar things has intervened.

Repression or Suppression
Freudian psychology advances the view that some forgetting is caused by repression or suppression. In repression or suppression, a memory is pushed out of reach because the individual does not want to remember the feelings associated with it. Repression is an unconscious form of forgetting while suppression is a conscious form. [This theory has no scientific basis.]

Retention of Learning

Meaningful learning goes deep because it involves principles and concepts anchored in the student’s own experiences.

Praise Stimulates Remembering
Responses that give a pleasurable return tend to be repeated. Absence of praise or recognition tends to discourage, and any form of negativism in the acceptance of a response tends to make its recall less likely. [This sounds like nonsense to me. I’d like to see some evidence.]

Recall Is Promoted by Association
As discussed earlier, each bit of information or action, which is associated with something to be learned, tends to facilitate its later recall by the student. Unique or disassociated facts tend to be forgotten unless they are of special interest or application.

Favorable Attitudes Aid Retention
People learn and remember only what they wish to know. Without motivation there is little chance for recall. The most effective motivation is based on positive or rewarding objectives.

Learning With All Senses Is Most Effective
Although people generally receive what is learned through the eyes and ears, other senses also contribute to most perceptions. When several senses respond together, a fuller understanding and greater chance of recall is achieved.

Meaningful Repetition Aids Recall
Each repetition gives the student an opportunity to gain a clearer and more accurate perception of the subject to be learned, but mere repetition does not guarantee retention. Practice provides an opportunity for learning, but does not cause it. Further, some research indicates that three or four repetitions provide the maximum effect, after which the rate of learning and probability of retention fall off rapidly.

Mnemonics
A mnemonic uses a pattern of letters, ideas, visual images, or associations to assist in remembering information.

Transfer of Learning

Transfer of learning is broadly defined as the ability to apply knowledge or procedures learned in one context to new contexts. Learning occurs more quickly and the learner develops a deeper understanding of the task if he or she brings some knowledge or skills from previous learning. A positive transfer of learning occurs when the learner practices under a variety of conditions, underscoring again the value of SBT.

• Plan for transfer as a primary objective. As in all areas of teaching, the chance for success is increased if the instructor deliberately plans to achieve it.
• Ensure that the students understand that what is learned can be applied to other situations. Prepare them to seek other applications.
• Maintain high-order learning standards. Overlearning may be appropriate. The more thoroughly the students understand the material, the more likely they are to see its relationship to new situations. Avoid unnecessary rote learning, since it does not foster transfer.
• Provide meaningful learning experiences that build student confidence in their ability to transfer learning. This suggests activities that challenge them to exercise their imagination and ingenuity in applying their knowledge and skills.
• Use instructional material that helps form valid concepts and generalizations. Use materials that make relationships clear.

Habit Formation
Due to the high level of knowledge and skill required in aviation for both pilots and maintenance technicians, training has traditionally followed a building block concept.

How Understanding Affects Memory
The ability to remember is greatly affected by the level of understanding of what has been learned.

Remembering During Training
The first threat to newly acquired knowledge is a lack of frequent usage in the past.

A second threat to newly acquired knowledge is a lack of understanding that might serve to assist the student in recalling it.

Summary of Instructor Actions
To help students remember what they have learned, the instructor should:
• Discuss the difference between short-term memory and long-term memory.
• Explain the effect of frequent and recent usage of knowledge on remembering and forgetting.
• Explain the effect of depth of understanding on remembering and forgetting.
• Encourage student use of mnemonic devices while studying.
• Explain the benefits of studying at regularly spaced intervals, and the disadvantages of “cramming.”

Chapter 2 Summary

Learning theory has caused instruction to move from basic skills and pure facts to linking new information with prior knowledge, from relying on a single authority to recognizing multiple sources of knowledge, and from novice-like to expert-like problem-solving. While educational theories facilitate learning, no one learning theory is good for all learning situations and all learners. Instruction in aviation should utilize a combination of learning theories.

Chapter 3:Effective Communication

The elements of effective communication, the barriers to communication, and the development of communication skills are discussed in this chapter.

Basic Elements of Communication

Communication takes place when one person transmits ideas or feelings to another person or group of people. The effectiveness of the communication is measured by the similarity between the idea transmitted and the idea received. The process of communication is composed of three elements:
• Source (sender, speaker, writer, encoder, transmitter, or instructor)
• Symbols used in composing and transmitting the message (words or signs (model prop))
• Receiver (listener, reader, decoder, or student)

Source
Their ability to select and use language is essential for transmitting symbols that are meaningful to listeners and readers.

Communicators consciously or unconsciously reveal attitudes toward themselves as a communicator, toward the ideas being communicated, and toward the receivers.

Communicators are more likely to be successful when they speak or write from accurate, up-to-date, and stimulating material.

Symbols
At its basic level, communication is achieved through symbols, which are simple oral and visual codes.

Communication through symbols is achieved by their interpretation through different perceptions, sometimes referred to as channels. While many theories have been proposed, one popular theory indicates that the symbols are perceived through one of three sensory channels: either visual, auditory, or kinethestic.

Feedback not only informs the students of their performance, but can also serve as a valuable source of motivation.

Receiver
The receiver is the listener, reader, decoder, or student—the individual or individuals to whom the message is directed.… When the receiver reacts with understanding and changes his or her behavior according to the intent of the source, effective communication has taken place.

In order to understand the process of communication, three characteristics of receivers must be understood: abilities, attitudes, and experiences.

An instructor needs to determine the abilities of the student in order to properly communicate.

The attitudes students exhibit may indicate resistance, willingness, or passive neutrality. To gain and hold student attention, attitudes should be molded into forms that promote reception of information.

Student experience, background, and educational level determine the approach an instructor takes.

Barriers to Effective Communication

Four barriers to effective communication: lack of common experience, confusion between the symbol and the symbolized object, overuse of abstractions, and interference.

Lack of Common Experience
Lack of common experience between the communicator (instructor) and the receiver (student) is probably the greatest single barrier to effective communication.… In order for communication to be effective, the students’ understanding of the meaning of the words needs to be the same as the instructor’s understanding.

Confusion Between the Symbol and the Symbolized Object
Confusion between the symbol and the symbolized object results when a word is confused with what it is meant to represent.

Overuse of Abstractions
Abstractions are words that are general rather than specific.

Interference
Some barriers to effective communication can be controlled by the instructor. Interference, or the prevention of a process or activity from being carried out properly, is composed of factors outside the control of the instructor These factors include physiological, environmental, and psychological interference. To communicate effectively, the instructor should consider the effects of these factors.

Developing Communication Skills

Role Playing
Role playing is a method of learning in which students perform a particular role. In role playing, the learner is provided with a general description of a situation and then applies a new skill or knowledge to perform the role.

Current Federal Aviation Administration (FAA) training emphasis has moved from a maneuvers-based training standard to what is called scenario-based training (SBT). SBT is a highly effective approach that allows students to learn, then apply their knowledge as they participate in realistic scenarios. This method of instruction and learning allows students to move from theory to practical application of skills during their training. Instructor applicants, flight or maintenance, need to learn to think in terms of SBT while they are students.

Instructional Communication
Instruction has taken place when the instructor has explained a particular procedure and subsequently determined that the desired student response has occurred.

Listening
Instructors must know something about their students in order to communicate effectively.

Listening is more than hearing. Most instructors are familiar with the concept that listening is “hearing with comprehension.”

Listening for main ideas is another listening technique.
The instructor must ensure that the student is aware of the danger of daydreaming.

Questioning
Good questioning can determine how well the student understands what is being taught. It also shows the student that the instructor is paying attention and that the instructor is interested in the student’s response. An instructor should ask focused, open-ended questions and avoid closed-ended questions.

Open-ended questions are designed to encourage full, meaningful answers using the student’s own knowledge and perceptions while closed-ended questions encourage a short or single-word answer.

Effective questions and, therefore, effective communications center on only one idea. A single question should be limited to who, what, when, where, why, or how and not a combination of these.

Two ways of confirming that the student and instructor understand things in the same way are the use of paraphrasing and perception checking.

Chapter 3 Summary

An awareness of the basic elements of the communicative process (source, symbols, and receiver) indicates the beginning of the understanding required for the successful communicator. Recognizing the various barriers to communication further enhances the flow of ideas between an instructor and the student. The instructor must develop communications skills in order to convey desired information to the students and must recognize that communication is a two-way process. In the end, the true test of whether successful communication has taken place is to determine if the desired results have been achieved.

You picked a fine time to leave me, loose wheel.

September 16th, 2018

Importing Pilot Data from the FAA

July 28th, 2018

We wanted to send out postcards to area pilots to encourage them to attend the EAA meetings that we have every third Sunday of the month. The more pilots we can get to attend, the better the quality of speaker that we can attract.

The FAA makes pilot names and addresses available in fixed format and csv format. It needs a little massaging before it is useable in my database of choice MySQL with phpMyAdmin as the import/viewing engine.

The first thing I did was to create the table and try to import a dozen lines from the file. There is a pdf on the FAA website that describes the file layout and field sizes so I just used that as my guide.

However, I got an error about incorrect number of fields when I tried to import. That is because they terminated each line with a comma. I removed them but ran into another problem when there was no medical info. So I put them back and then I changed a occurrences of ,,,, to ,NULL,NULL,NULL,. Then I removed the last comma in each line.

I got an error when I tried to import the whole database because PHP was not set up to import that file large.
I updated the /etc/php5/apache2php.ini file to account for the large size of the csv file. The file I was working with is 74 MB and my defaults are around 10 MB so it wouldn’t import.

I changed two lines


     post_max_size = 210M
     upload_max_filesize = 200M

and restarted Apache.

I still got an error trying to import line 495922 because the zip code had a comma in it. I fixed it and copied the rest of the file to a new file and it worked fine. I now have 571,598 records that I can use to print postcards from.

Which aircraft maintenance logs should I have available for a checkride?

July 27th, 2018

The answer is in the Private Pilot ‒ Airplane Airman Certification Standards

Task B. Airworthiness Requirements References 14 CFR parts 39, 43, 91; FAA-H-8083-2, FAA-H-8083-25
You should read and understand CFR §91.203, §91.205, §91.207, §91.209, §91.213, and §91.215.

The DPE will want to know that the aircraft is legally permitted to operate in the airspace that you will be taking your practical test in.

They will check that the AR(R)OW documents are in the aircraft—Airworthiness Certificate, Current Registration, probably not Radio License since it is not likely your test will take you to another country, Operating Limitations, and current Weight and Balance. Operating Limitations would include placards; any documents required by STCs for installed equipment e.g. GPS or autopilot; and for newer aircraft flight manual (AFM) for that aircraft.

They will check the logbooks to make sure that the aircraft is legal maintenance-wise. Annual Inspection and 100-hour AD items obviously. Some 100 hour items can be signed off by the pilot e.g. Bendix switch check while others need to be signed off by an A&P e.g. muffler shroud inspection. Most A&Ps use a computer system that prints out a list of ADs and when they were complied with. It is a good idea to put a sticky-note on the annual and recurring ADs pages so you don’t waste time looking for them.

ELTs are checked at annual time, but battery replacement can get out of sync with the annual and they will check that. If you are taking the test in an aircraft with an electrical system, the transponder must be working and checked in the last 24 months. You don’t need a pitot-static check for a private checkride, but if you are going to file and fly IFR flight plan—as you will for your instrument rating you will need one.

If there is any inoperative equipment e.g. LORAN then it must be logged and marked.

ADSB isn’t required yet, but after January 1, 2020 if you fly in the covered areas your aircraft must have it installed. CFR §91.225.

On an instrument check-ride the DPE will want to see that the database is current if you have a GPS and that the VOR check has been done.

They will also check to make sure that your medical is current and that you have all of the endorsements necessary for the aircraft e.g. complex, high performance, tailwheel.

Wind Tee

July 8th, 2018

I’ve read about these, but never saw one until we went to Auburn, KAUN, to pick up an airplane.

Wind Tee

Here’s a view from the back. Notice the whit blocks on the ground at 20° intervals that help you judge the wind direction.

Wind Tee Back

Cessna 210 Article

May 27th, 2018

1959 Cessna 210
I ran across this article in the November 1959 Flying Magazine about the new Cessna 210. Not a whole lot changed in the flying characteristics of the original and my 1973 T210L.

CalFire

March 19th, 2018

We had a guy from CalFire give a talk about their airplanes at our EAA meeting. One of the videos he showed was a drop from their S2T. Notice how the pilot grabs the yoke when he drops his load.

Cal Fire S2T

Hollister Base

Maintenance base at McClellan

Taxi and takeoff at PRB

Winter Storm in the Northeast

March 2nd, 2018

Wow. I like the note on BWI—no significant weather observed at this time.

KJFK

KBOS

KBWI

First flight in friend’s Dakota

February 15th, 2018

Sunset

Click to embiggen.

High to low, look out below.

January 8th, 2018

We a high pressure system sitting over the Central Coast for much of the fall which meant great flying weather, but no rain. On December 23 I went out to fly and noticed that the altimeter registered negative 20′. I thought it was unusual so I took a picture. Then set the altimeter and got the normal elevation at my tiedown.

Dec_23_Start

Altimeter before listening to ATIS.

Dec_23_Actual

Altimeter after listening to ATIS.

Two days later, it was off by 100’—in the bad direction. We still had high pressure, but it was lower than a couple of days earlier. Not an issue for my VFR flight, but 100′ is a big deal when flying the ILS.

Dec_25_Start

Altimeter before listening to ATIS.

Dec_25_Actual

Altimeter after listening to ATIS.

I didn’t fly again until January 9th, just before a major storm passed through, but others had taken the plane up. This is what the altimeter looked like when I got in the plane. Wow. A 300′ difference.

Jan_9_Start

Altimeter before listening to ATIS.

Jan_9_Actual

Altimeter after listening to ATIS.

Here’s another extreme change when the high pressure returned.

Jan_22_Start

Altimeter before listening to ATIS.

Jan_22_Actual

Altimeter after listening to ATIS.

Keeping track of maintenance items.

January 4th, 2018

I was talking to a new plane owner and he was wondering how you keep track of all the different times when you need to do inspections. The annual is pretty easy to remember, but other things often get out of sync with the annual. e.g. ELT, transponder check, IFR check, Aspen check.

I suggested that since he uses a Google calendar for scheduling his plane with others who fly it, that he just go through the logbooks and schedule when things that require an A&P/IA or avionics guy are due. Alternatively, he could just schedule all of the biennial items at the same time as the annual and not have to worry about it.

This is my list of items that an aircraft owner needs to keep up with.

Monthly
Check tach time to see if an oil change is needed. Recommended time is 35-50 hours.
Lube yoke and hinges.
Lube seat tracks.
Check tire pressures.
Check brake fluid level
Inspect brake pads and tires for wear.
Clean belly
You could do the VOR check if you need an excuse to fly.

Airworthy Aircraft
Current Registration – You should get a postcard in 2 years to renew.
Annual

100 Hour AD items – You might want to put a sticker next to your tach.
You can do the Bendix switch AD. A&P probably has to do the rest.

Transponder – 24 months
ELT – 12 Months Inspection, Replaced at 24 months or 50%
Have them checked at the annual even if you have a few months left and you are good for two years.

Airworthy Aircraft for IFR
Altimeter- 24 months
Pitot/Static- 24 months

Your avionics guy can do the transponder and IFR check at the same time on the ramp.

My transponder and IFR check are way out of sync with the annual since I usually schedule them the first day of the month after they expire. I get 25 months that way instead of 24. Yes I’m cheap.

GPS Database Current (New version every 28 days.)
Only need this if you are doing RNAV approach or departure.

VOR check within 30 days
The pilot of a plane with two VORs can do this in the air if they are going to use VORs or an ILS for the approach. I only have one VOR in my IFR plane, so I check it every month and log it when I do my GPS update.

I looked up the Aspen inspection items (Instructions for continued airworthiness) and it looks like the IA can do them at annual. Note that they are recommended items, not required by the STC. The document gives guidelines for the inspection.

Other avionics may have required or recommended inspections, so you’ll need to look them up in the documents for your particular installation. For example, I fly a plane with a Sandel HSI and the bulb needs to be replaced at regular intervals.

Chocks

January 3rd, 2018

While cleaning out the garage I found a five foot piece of angled steel that was purchased for some long-forgotten project. Rather than recycling it, I made chocks. I picked up some stretchy cord for 99¢ and cut the steel with my sawzall. A quick cleanup with a file and a couple of knots and voila, three sets of lightweight chocks.

home made chocks

Christmas Present 2017

December 24th, 2017

Christmas Present 2017

Bead Boxes for Storing Hardware

November 15th, 2017

I don’t think I have ever mentioned how handy bead boxes are for storing the screws and bolts when taking apart a plane for the annual.

Bead Boxes

I use 40% off coupons at Michaels to purchase these bead boxes. The 210 takes four—one for each wing, two for the interior. I also use a half-dozen salsa containers for things like the baggage compartment, wheels, spinner, muffler shroud, etc. The Cherokee has far fewer inspection panels so it takes half the time and fewer boxes. I have eight of these that I use to hold hardware that I purchased from Aircraft Spruce. I got tired of spending $10 on shipping for a few 25¢ bolts when my IA didn’t have the ones we needed, so I bought a couple of screw and bolt kits and ordered 10 of everything that wasn’t in the kit.

How much you save on your annual depends on your mechanic. My original IA knocked $100 off the price claiming that he could take apart and put back together a C210 in 10 hours. (It takes me at least 25 hours to clean the plugs, pack the bearings, and disassemble/reassemble a 210 so I have a hard time believing he could do it in 10 hours.). My current IA charges $700 for the inspection and his time for squawks. And we do them in my hangar so I can work when he isn’t around. Annuals are $2,000 cheaper with him and usually take only a couple of days instead of the week or more with the old IA.

First Mention of AOA indicator?

November 4th, 2017

I ran across this article in the February 1914 issue of Popular Mechanics.

AOA Indicator Popular Mechanics Feb 1914

Transcript of FAASTeam sUAS Course

October 26th, 2017

This is a transcript of the excellent sUAS course that a private pilot can take to get their sUAS rating. If you are not a pilot, you might want to refer to these notes when studying for the Knowledge Test.

Overview
If you wish to operate small unmanned aircraft systems in the National Airspace System, or NAS, under 14 CFR part 107, this course will describe the certification and operational requirements you must satisfy.

This initial lesson defines the target audience and scope of this course.

The lesson then describes how the course prepares part 61 certificate holders (with a current flight review in accordance with 14 CFR part 61.56) to obtain a part 107 remote pilot certificate with a small unmanned aircraft system rating. The lesson also provides an overview of certification requirements for all other applicants.

The lesson then describes the structure for the remaining modules and lessons in this course.

If you wish to operate small unmanned aircraft systems in the National Airspace System, or NAS, under 14 CFR part 107, this course will describe the certification and operational requirements you must satisfy.

Primary Audience:
This course is designed for part 61 pilot certificate holders who have a current flight review (in accordance with 14 CFR part 61.56) and wish to obtain a part 107 remote pilot certificate with a small UAS rating.

References to “part 61 pilot certificate holders” specifically refer to holders of pilot certificates other than student pilot certificates. Part 61 pilot certificates include sport pilot, recreational pilot, private pilot, commercial pilot and air transport pilot certificates.

Secondary Audiences:
Applicants for a part 107 remote pilot certificate who do not hold a part 61 pilot certificate (or part 61 pilot certificate holders who do not meet the requirements of a current flight review or other provisions of 14 CFR part 61.56) may incorporate this training into their self-study curriculum to help prepare for the FAA Unmanned Aircraft General (UAG) Knowledge Test
Aviation Safety Inspectors (ASIs)
Those interested in learning more about 14 CFR part 107

This course assumes the learner has operational knowledge of 14 CFR part 61, Certification: Pilots, Flight Instructors, and Ground Instructors, and 14 CFR part 91, General Operating and Flight Rules.

The course focuses on the knowledge areas of 14 CFR part 107 that are beyond the operational knowledge of parts 61 and 91:

Applicable regulations relating to small UAS rating privileges, limitations, and flight operation
Effects of weather on small unmanned aircraft performance
Small unmanned aircraft loading
Determining the performance of small unmanned aircraft
Emergency procedures
Crew resource management
Maintenance and preflight inspection procedures

Eligibility for a Part 107 Remote Pilot Certificate
To apply for a part 107 remote pilot certificate with a small UAS rating, you must satisfy the following eligibility requirements:

Be at least 16 years old
Be able to read, speak, write, and understand the English language (FAA may make exceptions for medical reasons)
Be in a physical and mental condition that would not interfere with the safe operation of the small UAS

Sources: 14 CFR part 107.61; Advisory Circular (AC) 107-2, Small Unmanned Aircraft Systems (sUAS) (as amended)

Training and Knowledge Testing Requirements
Applicants for a part 107 remote pilot certificate with a small UAS rating must meet the requirements below in order to gain and retain the knowledge necessary to safely operate a small UAS in the NAS.

Applicant Initial Requirements
Recurrent Requirements (every 24 months)

Part 61 Pilot Certificate Holder with a Current Flight Review (per 14 CFR part 61.56)
This initial online course, or
The initial FAA Unmanned Aircraft General (UAG) Knowledge Test at a Knowledge Testing Center (KTC)
The recurrent online course, or
The recurrent FAA Unmanned Aircraft General (UAG) Knowledge Test at a Knowledge Testing Center (KTC)

Any Other Applicant
The initial FAA Unmanned Aircraft General (UAG) Knowledge Test at a Knowledge Testing Center (KTC)
The recurrent FAA Unmanned Aircraft General (UAG) Knowledge Test at a Knowledge Testing Center (KTC)
Visit the FAA Airman Testing website for more information about initial and recurrent requirements and for a list of Commercial Test Centers (Knowledge Testing Centers (KTCs)). All web resources described in this course are compiled on the Resources page for your future reference.

Testing: Part 61 Pilot Certificate Holders
If you are a part 61 pilot certificate holder and you have a current flight review (per 14 CFR part 61.56), successful completion of this course is your ONLY training and testing requirement. At the end of this course, complete the online knowledge check (exam) and print your completion certificate.

If you prefer, you have the option to complete the initial FAA Unmanned Aircraft General (UAG) Knowledge Test at a Knowledge Testing Center (KTC).

Testing: Non-Part 61 Pilot Certificate Holders
If you are not a part 61 pilot certificate holder (or do not hold a current flight review in accordance with 14 CFR part 61.56), you are required to demonstrate an understanding of all areas of knowledge specified in 14 CFR part 107.73(a) by passing the initial FAA Unmanned Aircraft General (UAG) Knowledge Test at an FAA-approved Knowledge Testing Center (KTC).

You may complete this course as one part of your independent study efforts to prepare for the FAA Unmanned Aircraft General (UAG) Knowledge Test. However, you are also encouraged to review reference materials that focus on the topic areas that are required in part 107.73(a), but are not covered in this course:

Airspace classification, operating requirements and flight restrictions affecting small unmanned aircraft operation
Aviation weather sources
Radio communication procedures
Physiological effects of drugs and alcohol
Aeronautical decision-making and judgment
Airport operations
Visit the Resources page to access the FAA Airman Testing website, UAS Airman Certification Standards (ACS), and other reference materials.

Self-Study Resources
To prepare for the FAA Unmanned Aircraft General (UAG) Knowledge Test at a Knowledge Testing Center (KTC), thoroughly review the materials provided on the Resources page for this course, including:

Regulations and policy documents, such as 14 CFR part 107 and Advisory Circular (AC) 107-2, Small Unmanned Aircraft Systems (sUAS) (as amended)
The FAA Airman Testing Standards Branch (AFS-630) Website that provides:
Reference Handbooks, such as the Aircraft Weight and Balance Handbook, Pilot’s Handbook of Aeronautical Knowledge (PHAK), Aeronautical Information Manual (AIM), and Risk Management Handbook
UAS Airman Certification Standards (ACS)
General information about the FAA Unmanned Aircraft General (UAG) Knowledge Test
Sample FAA Unmanned Aircraft General (UAG) Knowledge Test questions
The FAA Unmanned Aircraft Systems Website

This module focuses on certification requirements and other responsibilities related to the aircraft and Remote Pilot in Command. You must satisfy all such requirements before operating small unmanned aircraft systems in the National Airspace System as a Remote Pilot in Command.

This lesson focuses on the aircraft itself. The lesson defines the characteristics of small unmanned aircraft systems, as stipulated in part 107. The lesson then identifies exclusions from the requirements in part 107 for model aircraft, other equipment, and certain operating conditions. The lesson examines the requirements for registering small unmanned aircraft systems with the FAA, displaying appropriate registration markings (per 14 CFR parts 47 or 48), and ensuring that the aircraft is in condition for safe operation.

Aircraft and Remote Pilot in Command Requirements: Small UAS Characteristics and Requirements
Small Unmanned Aircraft Systems
14 CFR part 107 applies to the operation of certain civil small unmanned aircraft within the NAS. Except for certain excluded aircraft operations, any aircraft that meets the criteria below is considered a small unmanned aircraft.

Small unmanned aircraft:
Weigh less than 55 pounds (25 kg), including everything that is onboard or otherwise attached to the aircraft
Are operated without the possibility of direct human intervention from within or on the aircraft
A small unmanned aircraft system includes the unmanned aircraft itself and its associated elements that are required for safe operation, such as communication links and components that control the aircraft.

Not all small unmanned aircraft are subject to 14 CFR part 107.

Sources: 14 CFR parts 107.1 and 107.3; AC 107-2, Small Unmanned Aircraft Systems (sUAS) (as amended)

Registration of Small UAS
Like other types of civil aircraft, most small UAS must be registered with the FAA prior to operating in the NAS.

Owners must register any small UAS that is greater than 0.55 lbs and operated under part 107.

The owner must satisfy the registration requirements described in 14 CFR part 47, Aircraft Registration, or part 48, Registration and Marking Requirements for Small Unmanned Aircraft for commercial operations. If the owner is less than 13 years of age, then the small unmanned aircraft must be registered by a person who is at least 13 years of age.

14 CFR part 48 establishes the streamlined online registration option for a small UAS that will be operated only within the territorial limits of the United States.

Sources: 14 CFR parts 107.13 and 48.25; AC 107-2, Small Unmanned Aircraft Systems (sUAS) (as amended)

Foreign Aircraft Permit Requirements
A small UAS operation requires a Foreign Aircraft Permit if it involves a civil aircraft that is:

Registered in a foreign country, or
Owned, controlled, or operated by someone who is not a U.S. citizen or permanent resident.
If either criteria is met, the Remote PIC should obtain a Foreign Aircraft Permit pursuant to 14 CFR part 375.41 before conducting any operations. Application instructions are specified in 14 CFR part 375.43, Navigation of Foreign Civil Aircraft within the United States. Submit the application by electronic mail to the Department of Transportation (DOT) Office of International Aviation, Foreign Air Carrier Licensing Division.

Sources: 14 CFR part 107.13; AC 107-2, Small Unmanned Aircraft Systems (sUAS) (as amended)

Registration Markings
Before operation, mark the small UAS to identify that it is registered with the FAA.

The registration marking must be:

A unique identifier number. This is typically the registration number or N-number.
Legible and durable. Sample methods include engraving, permanent marker, or self-adhesive label.
Visible or accessible. The number may be enclosed in a compartment only if you can access the compartment without tools.

Sources: 14 CFR part 48; AC 107-2, Small Unmanned Aircraft Systems (sUAS) (as amended); FAA Small UAS Registration FAQs (https://www.faa.gov/uas/registration/faqs/#mou)

Condition for Safe Operation
An FAA airworthiness certification is not required for a small UAS. However, the Remote PIC must maintain and inspect the small UAS prior to each flight to ensure that it is in a condition for safe operation. For example, inspect all components of the small UAS for equipment damage or malfunctions.

Sources: 14 CFR part 107.15; AC 107-2, Small Unmanned Aircraft Systems (sUAS) (as amended)

Review Questions
Scenario 1: You are operating a 1280 g (2.8lb) quadcopter for your own enjoyment. Is this small UAS operation subject to 14 CFR part 107?
NO

Scenario 2: You have accepted football tickets in exchange for using your small UAS to videotape the field before and after the game. Is this small UAS operation subject to 14 CFR part 107?
YES

Scenario 3: You plan to operate a 33lb small UAS to capture aerial imagery over real estate for use in sales listings. Is this small UAS operation subject to 14 CFR part 107?
YES

Summary
This lesson examined the requirements for small UAS registration, markings, and condition.

In summary, 14 CFR part 107 applies to certain civil small unmanned aircraft operations conducted within the NAS for purposes other than hobby or recreation. Most small UAS must be registered with FAA and appropriately marked. Airworthiness certification is not required for small UAS. However, the Remote PIC must ensure that the small UAS is in a condition for safe operation prior to flight.

You should now be able to:

Define small unmanned aircraft systems (small UAS)
Identify exclusions from the requirements in part 107
Identify the requirements for small UAS registration, markings, and condition

Aircraft and Remote Pilot in Command Requirements: Remote Pilot in Command Responsibilities
In the previous lesson, you learned the characteristics of small unmanned aircraft systems and the requirements for registration, markings, and condition prior to flight.

This lesson defines the role of the Remote Pilot in Command. The lesson then identifies two crew roles that may support small unmanned aircraft system operations: a person manipulating the controls and a visual observer.

The lesson examines best practices for crew resource management to ensure the safety of small unmanned aircraft system operations.

The previous lesson described small UAS and the requirements for registration, markings, and condition prior to flight.

This lesson describes:

The role of the Remote Pilot in Command
Other supporting crew roles
Best practices for crew resource management

Defined Crew Roles in a Team Environment
A small UAS operation may involve one individual or a team of crewmembers. Part 107 defines the following small UAS crew roles:

Remote Pilot in Command (Remote PIC): A person who holds a current remote pilot certificate with a small UAS rating and has the final authority and responsibility for the operation and safety of the small UAS
Person manipulating the controls: A person controlling the small UAS under direct supervision of the Remote PIC
Visual observer: A person acting as a flight crewmember to help see and avoid air traffic or other objects in the sky, overhead, or on the ground
This section of the lesson examines each of these roles and describes best practices for crew resource management.

Remote Pilot in Command
The Remote PIC is directly responsible for and is the final authority as to the operation of the small UAS conducted under 14 CFR part 107.

He or she must:

Be designated before each flight (but can change during the flight)
Ensure that the operation:
Poses no undue hazard to people, aircraft, or property in the event of a loss of control of the aircraft for any reason
Complies with all applicable regulations of part 107
Operate the small unmanned aircraft to ensure compliance with all applicable provisions
Part 107 permits transfer of control of the small UAS between two or more certificated Remote PICs. The transfer of aircraft control (i.e. the Remote PIC designation) to each other must be accomplished while maintaining visual line of sight of the small UAS and without loss of control.

Sources: 14 CFR parts 107.19; AC 107-2, Small Unmanned Aircraft Systems (sUAS) (as amended)

Supporting Crew Roles: Person Manipulating the Controls
Person controlling a UAS
A non-certificated person may operate the small UAS under Part 107 only if:

He or she is directly supervised by the Remote PIC
and
The Remote PIC has the ability to immediately take direct control of the small UAS
The Remote PIC is ultimately responsible for identifying hazardous conditions. The Remote PIC’s ability to regain control of the small UAS is necessary to ensure that he or she can quickly intervene to ensure the safety of the flight and prevent a hazardous situation before an accident or incident occurs.

Source: AC 107-2, Small Unmanned Aircraft Systems (sUAS) (as amended)

Sample Methods to Regain Control
The ability for the Remote PIC to immediately take over the flight controls may be achieved by using a number of different methods.

For example, the Remote PIC could:

Stand close enough to physically take over the control station
Use a “buddy box” system with two control stations:
One for the person manipulating the flight controls
One that allows the Remote PIC to immediately override the other control station
Use a preprogrammed safe-mode system with “home” or “hover” functions

Source: AC 107-2, Small Unmanned Aircraft Systems (sUAS) (as amended)

Supporting Crew Roles: Visual Observer
The role of visual observers (VOs) is to alert the rest of the crew about potential hazards during operations involving a small UAS. The use of VOs is optional. However, the Remote PIC may use one or more VOs to supplement situational awareness and visual-line-of-sight responsibilities while the Remote PIC is conducting other mission-critical duties (such as checking displays).

The Remote PIC must make certain that all VOs:

Are positioned in a location where they are able to see the small UAS continuously and sufficiently to maintain visual line of sight
Possess a means to effectively communicate the small UAS position and the position of other aircraft to the Remote PIC and person manipulating the controls

Situational Awareness and Decision Making
The Remote PIC attains situational awareness by obtaining as much information as possible prior to a flight and becoming familiar with the performance capabilities of the small UAS, weather conditions, surrounding airspace, and Air Traffic Control (ATC) requirements. Sources of information include a weather briefing, ATC, FAA, local pilots, and landowners.

Technology, such as global positioning systems (GPS), mapping systems, and computer applications, can assist in collecting and managing information to improve your situational awareness and risk-based aeronautical decision making (ADM).

Source: PHAK, page 17-3; AC 60-22, Aeronautical Decision Making, Chapters 1, 3, and 4

Crew Resource Management
Crew resource management (CRM) is the effective use of all available resources—human, hardware, and information—prior to and during flight to ensure a successful outcome of the operation. The Remote PIC must integrate crew resource management techniques into all phases of the small UAS operation.

Many of the crew resource management techniques traditionally used in manned aircraft operations are also applicable for a small UAS, such as the ability to:

Delegate operational tasks and manage crewmembers
Recognize and address hazardous attitudes
Establish effective team communication procedures

Sources: AC 107-2, Small Unmanned Aircraft Systems (sUAS) (as amended); AC 60-22, Chapters 1 through 4; PHAK, page 17-4

Task Management
The Remote PIC identifies, delegates, and manages tasks for each small UAS operation.

Tasks can vary greatly depending on the complexity of the small UAS operation. Supporting crewmembers can help accomplish those tasks and ensure the safety of flight. For example, visual observers and other ground crew can provide valuable information about traffic, airspace, weather, equipment, and aircraft loading and performance.

The Remote PIC:

Assesses the operating environment (airspace, surrounding terrain, weather, hazards, etc.)
Determines the appropriate number of crewmembers that are needed to safely conduct a given operation. The Remote PIC must ensure sufficient crew support so that no one on the team becomes over-tasked, which increases the possibility of an incident or accident.
Informs participants of delegated tasks and sets expectations
Manages and supervises the crew to ensure that everyone completes their assigned tasks

Recognizing Hazardous Attitudes
Person with arms crossed and annoyed expression
Studies have identified five hazardous attitudes that can interfere with the ability to make sound decisions and properly exercise authority: anti-authority, impulsivity, invulnerability, machoism, and resignation.

Remote PICs should be alert for hazardous attitudes (in themselves or in other crewmembers), label it as hazardous, and correct the behavior.

Five Hazardous Attitudes
Attitude Motto Indicators Antidote

Anti-Authority “Don’t tell me what to do.” The person does not like or may resent anyone telling him or her what to do. The person may regard rules, regulations, and procedures as silly or unnecessary. (Note: it is always your prerogative to question authority if you feel it is in error.) “Follow the rules. They are usually right.”

Impulsivity “Do it quickly.” The person frequently feels the need to do something, anything, immediately. He or she does not stop to think about the best alternative and does the first thing that comes to mind. “Not so fast. Think first.”

Invulnerability “It won’t happen to me.” The person falsely believes that accidents happen to others, but never to him or her. The person knows accidents can happen and that anyone can be affected. However, the person never really feels or believes that he or she will be personally involved. Such people are more likely than others to take chances and increase risk. “It could happen to me.”

Machoism “I can do it—I’ll show them.” The person tries to prove that he or she is better than anyone else. The person takes risks to impress others. (Note: While this pattern is thought to be a male characteristic, women are equally susceptible.) “Taking chances is foolish.”
Resignation “What’s the use?” The person does not believe his or her actions make a difference in what happens. The person attributes outcomes to good or bad luck. He or she leaves the action to others, for better or worse. Sometimes, the person even goes along with unreasonable requests just to be a “nice guy.” “I’m not helpless. I can make a difference.”

Source: PHAK, 17-5

Effective Communication
The FAA requires that the Remote PIC and other crewmembers coordinate to:

Scan the airspace in the operational area for any potential collision hazard; and
Maintain awareness of the position of the small UAS through direct visual observation.
To achieve this goal, the Remote PIC should:

Foster an environment where open communication is encouraged and expected between the entire crew to maximize team performance
Establish effective communication procedures prior to flight
Select an appropriate method of communication, such as the use of hand-held radio or other effective means that would not create a distraction and allows all crewmembers to understand each other
Inform the crew as conditions change about any needed adjustments to ensure a safe outcome of the operation

Source: AC 107-2, Small Unmanned Aircraft Systems (sUAS) (as amended)

Review Questions
Crew Role 1: Who is responsible for ensuring that there are enough crewmembers for a given small UAS operation?

Remote Pilot in Command (Remote PIC)

Crew Role 2: Whose sole task during a small UAS operation is to watch the small UAS and report potential hazards to the rest of the crew?
Visual Observer

Crew Role 3: Which crewmember must hold a remote pilot certificate with a small UAS rating?
Remote Pilot in Command (Remote PIC)

Crew Role 4: Who is ultimately responsible for preventing a hazardous situation before an accident occurs?
Remote Pilot in Command (Remote PIC)

Crew Role 5: Which crewmember is required to be under the direct supervision of the Remote PIC when operating a small UAS?
Person manipulating the controls

Summary
This lesson described the Remote PIC’s responsibilities during a small UAS operation and best practices for crew resource management.

In summary, the Remote PIC holds a remote pilot certificate with a small UAS rating. He or she is ultimately responsible for the safe operation of the small UAS. The Remote PIC designates, prepares, and closely supervises any individuals serving as supporting crew members, such as the person manipulating the controls or visual observers.

You should now be able to:

Describe certification requirements for the Remote PIC
Define possible supporting crew roles in small UAS operations
Describe best practices for crew resource management

Rules for Safe Operation of Small UAS: Preflight Considerations
The previous modules examined the certification process, registration requirements, and crew roles for small UAS operations. This module focuses on requirements before, during, and after flight.

This lesson examines:

Recommended maintenance
Preflight inspection requirements
Loading considerations
Performance considerations
The effects of weather

The previous modules described the application process for a part 107 remote pilot certificate with a small unmanned aircraft system rating, small unmanned aircraft system characteristics and registration requirements, and crew roles.

This module describes operating rules for small unmanned aircraft systems operating in the National Airspace System. The Remote Pilot in Command has the final authority and responsibility to maintain and inspect the unmanned aircraft before flight, safely operate during flight, recognize abnormal and emergency situations, and report certain accidents.

This lesson focuses on preflight requirements. The lesson describes recommended maintenance and preflight inspection procedures to verify that the aircraft is in a condition for safe operation.

The lesson then describes the restrictions and best practices for safe loading of small unmanned aircraft systems.

The lesson examines factors to consider when evaluating performance in flight and the effects of weather.

Maintenance Requirements
Crewmembers check the condition of unmanned aircraft before flight
Maintenance for a small UAS includes scheduled and unscheduled overhaul, repair, inspection, modification, replacement, and system software upgrades for the unmanned aircraft itself and all components necessary for flight.

This first section of the lesson examines maintenance requirements and best practices.

Source: AC 107-2, Small Unmanned Aircraft Systems (sUAS) (as amended)

Manufacturers may recommend a maintenance or replacement schedule for the unmanned aircraft and system components based on time-in-service limits and other factors. Follow all manufacturer maintenance recommendations to achieve the longest and safest service life of the small UAS.

If the small UAS or component manufacturer does not provide scheduled maintenance instructions, it is recommended that you establish your own scheduled maintenance protocol.

For example:

Document any repair, modification, overhaul, or replacement of a system component resulting from normal flight operations
Record the time-in-service for that component at the time of the maintenance procedure
Assess these records over time to establish a reliable maintenance schedule for the small UAS and its components

Source: AC 107-2, Small Unmanned Aircraft Systems (sUAS) (as amended)

Unscheduled Maintenance
During the course of a preflight inspection, you may discover that a small UAS component requires some form of maintenance outside of the scheduled maintenance period.

For example, a small UAS component may require servicing (such as lubrication), repair, modification, overhaul, or replacement as a result of normal or abnormal flight operations. Or, the small UAS manufacturer or component manufacturer may require an unscheduled system software update to correct a problem.

In the event such a condition is found, do not conduct flight operations until the discrepancy is corrected.

Source: AC 107-2, Small Unmanned Aircraft Systems (sUAS) (as amended)

Performing Maintenance
In some instances, the small UAS or component manufacturer may require certain maintenance tasks be performed by the manufacturer or by a person or facility (personnel) specified by the manufacturer.

It is highly recommended that the maintenance be performed in accordance with the manufacturer’s instructions. However, if you decide not to use the manufacturer or the personnel recommended by the manufacturer and you are unable to perform the required maintenance yourself, you should:

Solicit the expertise of maintenance personnel familiar with the specific small UAS and its components
Consider using certificated maintenance providers, such as repair stations, holders of mechanic and repairman certificates, and persons working under the supervision of a mechanic or repairman
If you or the maintenance personnel are unable to repair, modify, or overhaul a small UAS or component back to its safe operational specification, then it is advisable to replace the small UAS or component with one that is in a condition for safe operation.

Complete all required maintenance before each flight—preferably in accordance with the manufacturer’s instructions or, in lieu of that, within known industry best practices.

Preflight Inspection
Before beginning any flight operation involving a small UAS:

Assess the operating environment
Inform any supporting crewmembers about the operation and their roles
Inspect the small UAS to ensure that it is in a condition for safe operation
Maintain documents required in the event of an on-site FAA inspection

Sources: 14 CFR parts 107.15 and 107.49; AC 107-2, Small Unmanned Aircraft Systems (sUAS) (as amended)

Operating Environment
Before a small UAS operation, assess the operating environment.

The assessment must include, but is not limited to:

Local weather conditions
Local airspace and any flight restrictions
The location of persons and property on the surface
Other ground hazards

Source: AC 107-2, Small Unmanned Aircraft Systems (sUAS) (as amended)

Information for the Crew
Before any small UAS operation, at a minimum, ensure that all persons directly participating in the small UAS operation are informed about:

Operating conditions
Emergency procedures
Contingency procedures
Roles and responsibilities of each person involved in the operation
Potential hazards

Source: AC 107-2, Small Unmanned Aircraft Systems (sUAS) (as amended)

Condition of Aircraft
Before any small UAS operation, inspect the aircraft for equipment damage or malfunctions.

For example, ensure that:

All control links between the control station and the small unmanned aircraft are working properly
There is sufficient power to continue controlled flight operations to a normal landing
Any object attached or carried by the small unmanned aircraft is secure and does not adversely affect the flight characteristics or controllability of the aircraft
The unique identifier is readily accessible and visible upon inspection of the small unmanned aircraft

Benefits of Recordkeeping
Careful recordkeeping can be highly beneficial for small UAS owners and operators. For example, recordkeeping provides essential safety support for commercial operators who may experience rapidly accumulated flight operational hours/cycles.

Consider maintaining a hardcopy and/or electronic logbook of all periodic inspections, maintenance, preventative maintenance, repairs, and alterations performed on the small UAS.

Such records should include all components of the sUAS, including the:

Small unmanned aircraft itself
Control station
Launch and recovery equipment
Data link equipment
Payload
Any other components required to safely operate the small UAS

FAA Inspections
You must make available to the FAA, upon request, the small UAS for inspection or testing.

In addition, you must verify before flight that all required documentation is physically or electronically available in the event of an on-site FAA inspection. Such documentation may include:

Pilot certificate
Aircraft registration
Any necessary waiver, authorization, or exemption
Other documentation related to the operation

Loading and Performance
Prior to each flight, the Remote PIC must ensure that any object attached to or carried by the small unmanned aircraft is secure and does not adversely affect the flight characteristics or controllability of the aircraft.

Source: 14 CFR part 107.49; AC 107-2, Small Unmanned Aircraft Systems (sUAS) (as amended)

Loading Considerations: General Weight and Balance
As with any aircraft, compliance with weight and balance limits is critical to the safety of flight for a small UAS. An unmanned aircraft that is loaded out of balance may exhibit unexpected and unsafe flight characteristics.

Before any flight, verify that the unmanned aircraft is correctly loaded by determining the weight and balance condition.

Review any available manufacturer weight and balance data and follow all warnings, cautions, notes, and limitations
If the manufacturer does not provide specific weight and balance data, apply general weight and balance principals to determine limits for a given flight. For example, add weight to the unmanned aircraft in a manner that does not adversely affect the aircraft’s center of gravity (CG) location—a point at which the unmanned aircraft would balance if it were suspended at that point.

Sources: PHAK; FAA-H-8083-1, Weight & Balance Handbook, 4-4-5

Factors that Affect Maximum Gross Takeoff Weight
Although a maximum gross takeoff weight is normally specified for a given unmanned aircraft, the aircraft may not be able to launch with this load under all conditions. Or if it does become airborne, the unmanned aircraft may exhibit unexpected and unusually poor flight characteristics.

Factors that may require a reduction in weight prior to flight include:

High density altitude conditions
High elevations
High air temperatures
High humidity
Runway/launch area length
Surface
Slope
Surface wind
Presence of obstacles

Sources: AC 107-2, Small Unmanned Aircraft Systems (sUAS) (as amended); PHAK

Common Performance Deficiencies of Overloaded Aircraft
Excessive weight reduces the flight performance in almost every respect. In addition, operating above the maximum weight limitation can compromise the structural integrity of an unmanned aircraft.

The most common performance deficiencies of an overloaded aircraft are:

Reduced rate of climb
Lower maximum altitude
Shorter endurance
Reduced maneuverability

Loading Considerations: Effects of Weight Changes
Weight changes have a direct effect on aircraft performance.

Fuel burn is the most common weight change that takes place during flight.

For battery-powered unmanned aircraft, weight change during flight may occur when expendable items are used on board (e.g., water or other liquids dispensed for authorized agricultural use). Changes of mounted equipment between flights, such as the installation of cameras, battery packs, or other instruments, may also affect the weight and balance and performance of a small UAS.

Sources: AC 107-2, Small Unmanned Aircraft Systems (sUAS) (as amended); PHAK

Loading Considerations: Effects of Load Factor
Unmanned airplane performance can be decreased due to an increase in load factor when the airplane is operated in maneuvers other than straight and level flight.

The load factor increases at a terrific rate after a bank has reached 45° or 50°. The load factor for any aircraft in a coordinated level turn at 60° bank is 2 Gs. The load factor in an 80° bank is 5.76 Gs. The wing must produce lift equal to these load factors if altitude is to be maintained. The Remote PIC should be mindful of the increased load factor and its possible effects on the aircraft’s structural integrity and the results of an increase in stall speed.

As with manned aircraft, an unmanned airplane will stall when critical angle of attack is exceeded. Due to the low altitude operating environment, consideration should be given to ensure aircraft control is maintained and the aircraft isn’t operated outside its performance limits.

Carriage of Hazardous Material
A small unmanned aircraft may not carry hazardous material as defined in 49 CFR part 171.8:

Hazardous material means, “a substance or material that the Secretary of Transportation has determined is capable of posing an unreasonable risk to health, safety, and property when transported in commerce, and has designated as hazardous under section 5103 of Federal hazardous materials transportation law (49 U.S.C. 5103). The term includes hazardous substances, hazardous wastes, marine pollutants, elevated temperature materials, materials designated as hazardous in the Hazardous Materials Table (see 49 CFR 172.101), and materials that meet the defining criteria for hazard classes and divisions in part 173 of subchapter C of this chapter.”

Sources: 14 CFR part 107.36; 49 CFR part 171.8

Carriage of Lithium Batteries
Lithium batteries that are installed in a small UAS for power during the operation are not considered a hazardous material under part 107.

However, spare (uninstalled) lithium batteries would meet the definition of hazardous material and may not be carried as cargo on the small UAS.

Determining Performance: Sources of Performance Data
Performance or operational information may be provided by the manufacturer in the form of an Aircraft Flight Manual, Pilot’s Operating Handbook, or owner’s manual. Follow all manufacturer recommendations for evaluating performance to ensure safe and efficient operation.

Even when specific performance data is not provided, the Remote PIC should be familiar with:
The operating environment
All available information regarding the safe and manufacturer’s recommended operation of the small UAS

Source: PHAK

Remote PIC Responsibilities for Determining Performance
The Remote PIC is responsible for ensuring that every flight can be accomplished safely, does not pose an undue hazard, and does not increase the likelihood of a loss of positive control.

Consider how your decisions affect the safety of flight. For example:

If you attempt flight in windy conditions, the unmanned aircraft may require an unusually high power setting to ascend. This action may cause a rapid depletion of battery power and result in a failure mode.
If you attempt flight in wintery weather conditions, ice may accumulate on the unmanned aircraft’s surface. Ice increases the weight and adversely affects performance characteristics of the small unmanned aircraft.
Due to the diversity and rapidly-evolving nature of small UAS operations, individual Remote PICs have flexibility to determine what equipage methods, if any, mitigate risk sufficiently to meet performance-based requirements, such as the prohibition on creating an undue hazard if there is a loss of aircraft control.

Determining Performance: Operational Data
The FAA acknowledges that some manufacturers provide comprehensive operational data and manuals, such as Aircraft Flight Manuals or Pilot’s Operating Handbooks, and others do not. When operational data is provided, follow the manufacturer’s instructions and recommendations.

Even when operational data is not supplied by the manufacturer, the Remote PIC can better understand the unmanned aircraft’s capabilities and limitations by establishing a process for tracking malfunctions, defects, and flight characteristics in various environments and conditions. Use this operational data to establish a baseline for determining performance, reliability, and risk assessment for your particular system.

Effects of Weather on Performance
Field with storm clouds approaching
Even though small UAS operations are often conducted at very low altitudes, weather factors can greatly influence performance and safety of flight.

Specifically, factors that affect small UAS performance and risk management include:

Atmospheric pressure and stability
Wind and currents
Uneven surface heating
Visibility and cloud clearance
As with any flight, the Remote PIC should check and consider the weather conditions prior to and during every small UAS flight.

Wind
Wind and currents can affect small UAS performance and maneuverability during all phases of flight. Be vigilant when operating a small UAS at low altitudes, in confined areas, near buildings or other manmade structures, and near natural obstructions (such as mountains, bluffs, or canyons).

Consider the following effects of wind on performance:

Obstructions on the ground affect the flow of wind, may create rapidly changing wind gusts, and can be an unseen danger
The intensity of the turbulence associated with ground obstructions depends on the size of the obstacle and the primary velocity of the wind
Even when operating in an open field, wind blowing against surrounding trees can create significant low level turbulence
High winds may make it difficult to maintain a geographical position in flight and may consume more battery power

Source: PHAK

Example: Operations Near Buildings
Remember that local conditions, geological features, and other anomalies can change the wind direction and speed close to the Earth’s surface.

For example, when operating close to a building, winds blowing against the building could cause strong updrafts that can result in ballooning or a loss of positive control. On the other hand, winds blowing over the building from the opposite side can cause significant downdrafts that can have a dramatic sinking effect on the unmanned aircraft.

Currents
Different surfaces radiate heat in varying amounts.

The resulting uneven heating of the air creates small areas of local circulation called convective currents. Convective currents can cause bumpy, turbulent air that can dramatically affect the Remote PIC’s ability to control unmanned aircraft at lower altitudes.

For example:
Plowed ground, rocks, sand, and barren land give off a large amount of heat and are likely to result in updrafts
Water, trees, and other areas of vegetation tend to absorb and retain heat and are likely to result in downdrafts

Source: PHAK

Visibility and Clouds
As in manned aircraft operations, good visibility and safe distance from clouds enhances the Remote PIC’s ability to see and avoid other aircraft. Similarly, good visibility and cloud clearance may be the only means for other aircraft to see and avoid the unmanned aircraft.

The regulatory requirements for visibility and cloud clearance are discussed in a later module. But it should be noted here that adherence to the regulatory requirements in conjunction with good airmanship and effective scanning techniques can preclude in-flight collisions. And collision avoidance is an essential aspect to the safe integration of a small UAS into the NAS.

Source: PHAK

Review Questions
Which of the following source of information should you consult first when determining what maintenance should be performed on a small UAS or its components?
Manufacturer guidance

How often is the Remote PIC required to inspect the small UAS to ensure that it is in a condition for safe operation?
Before each flight

When loading cameras or other equipment on a small UAS, mount the items in a manner that:
A Is visible to the visual observer or other crewmembers.
B Does not adversely affect the center of gravity.
*C Can be easily removed without the use of tools.

Which of the following considerations is most relevant to a Remote PIC when evaluating unmanned aircraft performance?
*A Current weather conditions
B The number of available ground crew
C The type of the small UAS operation

Summary
This lesson examined preflight requirements for a small UAS.

In summary, the Remote PIC is responsible for maintenance and pre-flight inspection of the small UAS, safe loading techniques, and continuous assessment of performance in flight.

You should now be able to describe:

Recommended maintenance procedures for a small UAS
Inspection requirements to verify that the small UAS is in a condition for safe operation
Considerations for safe loading of unmanned aircraft
Procedures for evaluating performance
The effects of weather on performance

Rules for Safe Operation of Small UAS: Operating Rules
Introduction
The previous lesson examined preflight requirements for small unmanned aircraft system operations, including scheduled and unscheduled maintenance, preflight inspection procedures, loading considerations, and factors that could affect aircraft performance.

This lesson focuses on rules for safe operation of small unmanned aircraft systems in the National Airspace System. The lesson describes operational requirements and limitations and available certificates of waiver for select requirements in 14 CFR part 107.

Daylight Only Operations
14 CFR part 107 prohibits operation of a small UAS at night, defined in 14 CFR part 1 as the time between the end of evening civil twilight and the beginning of morning civil twilight, as published in the Federal Air Almanac, and converted to local time.

The Federal Air Almanac provides tables to determine sunrise and sunset at various latitudes. For example:

In the contiguous United States, evening civil twilight is the period of sunset until 30 minutes after sunset and morning civil twilight is the period of 30 minutes prior to sunrise until sunrise
In Alaska, the definition of civil twilight differs and is described in the Federal Air Almanac
Visit the Resources page to access the Naval Observatory website where you can download these tables and customize them for your location.

Source: 14 CFR part 107.29; AC 107-2, Small Unmanned Aircraft Systems (sUAS) (as amended)

Daylight: Operations in Civil Twilight
When small UAS operations are conducted during civil twilight, the small UAS must be equipped with anti-collision lights that are capable of being visible for at least 3 statute miles from the control station.

However, the Remote PIC may reduce the intensity of the lighting if he or she has determined that it would be in the interest of operational safety to do so. For example, the Remote PIC may momentarily reduce the lighting intensity if it impacts his or her night-vision.

Source: 14 CFR part 107.29; AC 107-2, Small Unmanned Aircraft Systems (sUAS) (as amended)

Visual Line of Sight
The small unmanned aircraft must remain within visual line-of-sight (VLOS) of flight crewmembers. Visual line of sight means any flight crewmember (i.e. the Remote PIC; person manipulating the controls; and visual observers, if used) is capable of seeing the aircraft with vision unaided by any device other than corrective lenses (spectacles or contact lenses).

Crewmembers must operate within the following limitations.

Minimum visibility, as observed from the location of the control station, must be no less than 3 statute miles
Minimum distance from clouds must be no less than 500 feet below a cloud and 2000 feet horizontally from the cloud
Crewmembers must be able to see the small unmanned aircraft at all times during flight. Therefore, the small unmanned aircraft must be operated closely enough to the control station to ensure visibility requirements are met during small unmanned aircraft operations.

Sources: 14 CFR parts 107.31 and 107.51; AC 107-2, Small Unmanned Aircraft Systems (sUAS) (as amended)

Restrictions on Vision Aids
Visual line of sight must be accomplished and maintained by unaided vision, except vision that is corrected by the use of eyeglasses (spectacles) or contact lenses.

Vision aids, such as binoculars, may be used only momentarily to enhance situational awareness. For example, the Remote PIC, person manipulating the controls, or visual observer may use vision aids briefly to avoid flying over persons or to avoid conflicting with other aircraft.

Regaining Visual Line of Sight
The Remote PIC or person manipulating the controls may have brief moments in which he or she is not looking directly at or cannot see the small unmanned aircraft, but still retains the capability to see it or quickly maneuver it back to line of sight.

These moments should be for:

The safety of the operation, such as briefly looking down at the control station or scanning the airspace. To scan for traffic, the crew should systematically focus on different segments of the sky for short intervals.
Operational necessity, such as intentionally maneuvering the aircraft for a brief period behind an obstruction
There is no specific time interval for which interruption of visual contact is permissible. Such parameters could potentially allow a hazardous interruption or prohibit a reasonable one.

The Remote PIC or person manipulating the controls must attempt to regain visual line of sight:

Immediately, if he or she unintentionally loses sight of the aircraft
As soon as practicable, if he or she loses sight of the aircraft for operational necessity

Operating Limitations for Small Unmanned Aircraft
The small unmanned aircraft must be operated in accordance with the following limitations:

Cannot be flown faster than a groundspeed of 87 knots (100 miles per hour)
Cannot be flown higher than 400 feet above ground level (AGL) unless flown within a 400-foot radius of a structure and is not flown higher than 400 feet above the structure’s immediate uppermost limit

Sources: 14 CFR part 107.51; AC 107-2, Small Unmanned Aircraft Systems (sUAS) (as amended)

Operation near Aircraft: Right of Way Rules
No person may operate a small unmanned aircraft in a manner that interferes with operations and traffic patterns at any airport, heliport, or seaplane base. The Remote PIC also has a responsibility to remain clear of and yield right-of-way to all other aircraft, manned or unmanned, and avoid other potential hazards that may affect the Remote PIC’s operation of the aircraft. This is traditionally referred to as “see and avoid”.

To satisfy this responsibility, the Remote PIC must:

Know the location and flight path of his or her small unmanned aircraft at all times
Be aware of other aircraft, persons, and property in the vicinity of the operating area
Be able to maneuver the small unmanned aircraft to:
Avoid a collision
Prevent other aircraft from having to take evasive action
Avoid operating anywhere where the presence of his or her unmanned aircraft may interfere with operations at the airport, such as approach corridors, taxiways, runways, or helipads
Yield right-of-way to all other aircraft, including aircraft operating on the surface of the airport
First-person view camera cannot satisfy ‘‘see-and-avoid’’ requirement. However, such cameras can be used as long as the “see-and-avoid” requirement is satisfied in other ways.

Sources: 14 CFR parts 107.37 and 43; AC 107-2, Small Unmanned Aircraft Systems (sUAS) (as amended)

Operation in Certain Airspace
Many small UAS operations can be conducted in uncontrolled, Class G airspace without further permission or authorization. However, operations require prior authorization from Air Traffic Control (ATC) in Class B, C, and D airspace and within the lateral boundaries of the surface area of Class E airspace designated for an airport.

It is incumbent on the Remote PIC to be aware of the type of airspace in which they will be operating their small UAS. As with other flight operations, the Remote PIC should refer to current aeronautical charts and other navigation tools to determine position and related airspace.

Notices to Airmen (NOTAMs)

Smartphone displaying the FAA mobile application
Temporary Flight Restrictions (TFRs) are inclusive of small UAS operations. For that reason, it is necessary for the Remote PIC to check for Notices to Airmen (NOTAMs) before each flight to determine if there are any applicable airspace restrictions.

Common TFRs that relate to small UAS operations include, but are not limited to:

Presidential TFRs and NOTAMs
Emergency response TFRs and NOTAMs
Standing TFRs that go into and out of effect (e.g., stadiums for sporting events)

Operation in Prohibited or Restricted Areas or Areas Designated in NOTAMs
Cover of 14 CFR, labeled with parts 99.7 and 91 subpart B
No person may operate a small unmanned aircraft in prohibited or restricted areas unless that person has permission from the using or controlling agency, as appropriate.

The Remote PIC must comply with the following provisions:

The provisions of 14 CFR part 99.7, Special Security Instructions
The following provisions of 14 CFR part 91 subpart B, Flight Rules:
14 CFR part 91.137 Temporary flight restrictions in the vicinity of disaster/hazard areas
14 CFR part 91.138 Temporary flight restrictions in national disaster areas in the State of Hawaii
14 CFR part 91.139 Emergency air traffic rules
14 CFR part 91.141 Flight restrictions in the proximity of the Presidential and other parties
14 CFR part 91.143 Flight limitation in the proximity of space flight operations
14 CFR part 91.144 Temporary restriction on flight operations during abnormally high barometric pressure conditions
14 CFR part 91.145 Management of aircraft operations in the vicinity of aerial demonstrations and major sporting events
Visit the Resources page to access these provisions.

Sources: 14 CFR parts 107.45 and 107.47; AC 107-2, Small Unmanned Aircraft Systems (sUAS) (as amended)

Obtaining Airspace Authorizations
ATC has the authority to approve or deny aircraft operations based on traffic density, controller workload, communication issues, or any other type of operations that could potentially impact the safe and expeditious flow of air traffic in that airspace.

When ATC authorization is required, it must be requested and granted before any operation in that airspace. There is currently no established timeline for approval after ATC permission has been requested because the time required for approval will vary based on the resources available at the ATC facility and the complexity and safety issues raised by each specific request.

For this reason, Remote PICs should request ATC authorization as soon as possible prior to any operation in Class B, C and D airspace and within the lateral boundaries of the surface area of Class E airspace designated for an airport.

Source: AC 107-2, Small Unmanned Aircraft Systems (sUAS) (as amended)

Frequency Spectrum
Most small UAS use radio frequencies to establish the data link between the control station and the small unmanned aircraft.

Considerations for radio frequencies used in small UAS operations include:

Frequency interference
Line of sight/obstructions

Source: AC 107-2, Small Unmanned Aircraft Systems (sUAS) (as amended)

Frequency Interference
Many small UAS utilize the same unlicensed frequency bands for control and command and video links

These unlicensed radio frequency bands used during a small UAS operation are regulated by the Federal Communications Commission (FCC) and may require an FCC license.

These same frequency bands are also commonly used for wifi and other remote/wireless devices. Frequency congestion and interference may affect operation of the small UAS.

Before conducting a small UAS operation, consult the manufacturer’s operating manual to determine the frequencies for your specific small UAS.

Source: AC 107-2, Small Unmanned Aircraft Systems (sUAS) (as amended)

Line of Sight and Frequency Obstructions
Small UAS radio frequency bands are considered line of sight.

Be aware that the command and control link between the control station and the small unmanned aircraft may not work properly when barriers are between the control station and the unmanned aircraft.

Source: AC 107-2, Small Unmanned Aircraft Systems (sUAS) (as amended)

Spectrum Authorization
Radio transmissions, such as those used to control an unmanned aircraft and to downlink real-time video, must use frequency bands that are approved for use by the operating agency. Operations on licensed band frequencies require a user-specific license for all civil users, except federal agencies, to be obtained from the FCC.

Source: AC 107-2, Small Unmanned Aircraft Systems (sUAS) (as amended)

No Operation Over People
You may not operate a small unmanned aircraft directly over another person unless that person is:

Directly involved in the operation (such as a visual observer or other crewmember)
OR
Within a safe cover, such as inside a stationary vehicle or a protective structure that would protect a person from harm if the small unmanned aircraft were to crash into that structure

Protecting Non-Participants
To comply with limitations on small UAS operations near persons not participating in the operation, the Remote PIC should employ the strategies described below.

Select an appropriate operational area for the small UAS flight
Ideally, select an operational area (site) that is sparsely populated
If operating in populated/inhabited areas, make a plan to keep non-participants clear, indoors, or under cover
If operating from a moving vehicle, choose a sparsely populated (or unpopulated) area and make a plan to keep the small UAS clear of anyone who may approach
Adopt an appropriate operating distance from non-participants
Take reasonable precautions to keep the operational area free of non-participants

Operation from Moving Vehicles or Aircraft
14 CFR part 107 permits operation of a small UAS from a moving land or water-borne vehicle over a sparsely populated (or unpopulated) area. However, operation from a moving aircraft is prohibited.

Additionally, small unmanned aircraft that are transporting another person’s property for compensation or hire may not be operated from any moving vehicle.

Source: 14 CFR part 107.25; AC 107-2, Small Unmanned Aircraft Systems (sUAS) (as amended)

Transporting Another Person’s Property
You may also operate a small UAS to transport another person’s property (cargo) for compensation or hire provided you comply with the additional requirements described below.

The total weight of the small UAS (including the cargo) must remain below 55lbs
The small UAS operation must be within the boundaries of a State (intrastate)
No items may be dropped from the small unmanned aircraft in a manner that creates an undue hazard to persons or property
You may not operate the small UAS from a moving land vehicle or water-born vessel

Moving Vehicles: Part 107 Restrictions
Operations from moving vehicles are subject to the same restrictions that apply to all other part 107 small UAS operations.

Examples include:

Visual Line of Sight: The Remote PIC (and the person manipulating the controls, if applicable) operating from a moving vehicle or watercraft is still required to maintain visual line of sight for the small UAS
Operations over People: Operations are still prohibited over persons not directly involved in the operation of the small UAS, unless under safe cover. The Remote PIC is also responsible for ensuring that no person is subject to undue risk as a result of loss of control of the small unmanned aircraft for any reason.
Communication: The visual observer and Remote PIC must still maintain effective communication
No Careless or Reckless Operation: Part 107 also prohibits careless or reckless operation of a small UAS. Operating a small UAS while driving a moving vehicle is considered to be careless or reckless because the driver’s attention would be hazardously divided. Therefore, the driver of a land vehicle or the operator of a water-borne vehicle must not serve as the Remote PIC, person manipulating the controls, or visual observer.

Source: 14 CFR part 107.25

Moving Vehicles: State and Local Traffic Laws
Other laws, such as State and local traffic laws, may also apply to the conduct of a person driving a vehicle.

Many states currently prohibit distracted driving and state or local laws may also be amended in the future to impose restrictions on how cars and public roads may be used with regard to a small UAS operation. The FAA emphasizes that people involved in a small UAS operation are responsible for complying with all applicable laws and not just the FAA’s regulations.

No Operations While Impaired
Part 107 does not allow operation of a small UAS if the Remote PIC, person manipulating the controls, or visual observer is unable to safely carry out his or her duties and responsibilities.

While drug and alcohol use are known to impair judgment, certain over-the-counter medications and medical conditions could also affect the ability to safely operate a small unmanned aircraft. For example, certain antihistamines and decongestants may cause drowsiness.

You may not directly participate in the operation of a small UAS if you know or have reason to know that you have a physical or mental condition that would interfere with the safe operation of the small UAS.

Impaired Judgement: Prohibition Thresholds
Part 107 prohibits a person from serving as any small UAS crewmember if he or she:

Consumed any alcoholic beverage within the preceding 8 hours
Is under the influence of alcohol
Has a blood alcohol concentration of .04% or greater
Is using a drug that affects the person’s mental or physical capabilities

No Hazardous Operation
No person may operate a small UAS in a careless or reckless manner so as to endanger another person’s life or property. Part 107 also prohibits allowing an object to be dropped from a small UAS in a manner that creates an undue hazard to persons or property.

Examples of hazardous operation include, but are not limited to:

Operations that interfere with manned aircraft operations
Operating a small UAS over persons not directly participating in the operation
Loading the small UAS beyond its capabilities to the point of losing control

Privacy and Other Considerations
Other laws, such as State and local privacy laws, may apply to small UAS operations. The Remote PIC is responsible for reviewing and complying with such laws prior to operation.

In addition, Remote PICs are encouraged to review the Department of Commerce National Telecommunications and Information Administration (NTIA) best practices that address privacy, transparency and accountability issues related to private and commercial use of a small UAS.

Certificates of Waiver

If the Remote PIC determines that the operation cannot be conducted within the regulatory structure of part 107, he or she is responsible for applying for a Certificate of Waiver in accordance with 14 CFR part 107.200 and proposing a safe alternative to the operation.

The application for a Certificate of Waiver must be submitted at least 90 days prior to planned use.

This Certificate of Waiver will allow a small UAS operation to deviate from certain provisions of part 107 as long as the FAA finds that the proposed operation can be safely conducted under the terms of that Certificate of Waiver.

Visit the Resources page to access the online application for a UAS Certificate of Waiver.

Waivable Sections of Part 107
Your request for a waiver may be granted if the FAA finds that the proposed operation can be safely conducted under the terms of that Certificate of Waiver.

A list of the waivable sections of part 107 can be found in 14 CFR part 107.205 and are listed below:

§ 107.25 Operation from a moving vehicle or aircraft. However, no waiver of this provision will be issued to allow the carriage of property of another by aircraft for compensation or hire
§ 107.29 Daylight operation
§ 107.31 Visual line of sight aircraft operation. However, no waiver of this provision will be issued to allow the carriage of property of another by aircraft for compensation or hire
§ 107.33 Visual observer
§ 107.35 Operation of multiple small unmanned aircraft systems
§ 107.37(a) Yielding the right of way
§ 107.39 Operation over people
§ 107.41 Operation in certain airspace
§ 107.51 Operating limitations for small unmanned aircraft

FAA Waiver Review Process
After submitting your online application, the FAA will determine if the proposed operation can be safely conducted under the terms of that Certificate of Waiver.

If the application is denied, you will receive notification stating the reasons for denial.

If the waiver or authorization is granted, you will receive direct notification with:

The completed FAA Form 7711-1, Certificate of Waiver or Authorization, dated, signed, and approved by the FAA
Specific special provisions that become regulatory for the waiver holder

Review Questions
Scenario 1: A professional wildlife photographer operates a small UAS from a moving truck to capture aerial images of migrating birds in remote wetlands. The driver of the truck does not serve any crewmember role in the operation.

Is this small UAS operation in compliance with 14 CFR part 107?
YES Compliant with part 107

Scenario 2: Power company employees use a small UAS to inspect a long stretch of high voltage powerlines. Due to muddy conditions, their vehicle must stay beside the road and the crew uses binoculars to maintain visual line of sight with the aircraft.

Is this small UAS operation in compliance with 14 CFR part 107?
NO Not compliant with part 107

Scenario 3: Personnel at an outdoor concert venue use a small UAS to drop promotional t-shirts and CDs over the audience.

Is this small UAS operation in compliance with 14 CFR part 107?
NO Not compliant with part 107

Summary
This lesson examined rules for safe operation of a small UAS.

In summary, the Remote PIC and all crewmembers must comply with part 107 requirements by operating at appropriate times, in approved locations, and in a manner that protects the safety of persons, property, and the NAS.

You should now be able to:

Describe operational requirements and limitations for a small UAS
Describe potential certificates of waiver for select requirements in part 107

Rules for Safe Operation of Small UAS: Abnormal and Emergency Situations
Introduction
The previous lessons described preflight considerations and the rules for safe operation of a small UAS.

This lesson examines:

Abnormal and emergency situations
Accident reporting requirements

The previous lessons in this module described preflight considerations and the rules for safe operation of small unmanned aircraft systems.

This lesson examines the abnormal and emergency situations that could arise during a small unmanned aircraft system operation. The lesson then defines the requirements for notifying the FAA should an accident occur involving small unmanned aircraft.

Emergency Planning and Communication
A visitor inspects the Schiebel Camcopter during the unmanned aerial vehicle flight demonstration at Naval Air Station, Patuxent River, Maryland
In case of an in-flight emergency, the Remote PIC is permitted to deviate from any rule of part 107 to the extent necessary to meet that emergency. Upon FAA request, you must send a written report to the FAA explaining the deviation.

Become familiar with any manufacturer suggested emergency procedures prior to flight. Review emergency actions during preflight planning and inform crew members of their responsibilities.

Sources: 14 CFR part 107.21; AC 107-2, Small Unmanned Aircraft Systems (sUAS) (as amended)

Common Abnormal and Emergency Situations
The Remote PIC must be prepared to respond to abnormal and emergency situations during small UAS operations.

Refer to the manufacturer’s guidance for appropriate procedures in the following situations:

Abnormal situations, such as lost link, alternate landing/recovery sites, and flight termination (controlled flight to the ground)
Emergency situations, such as flyaways, loss of Global Positioning System (GPS), and battery fires

Lost Link
Without an onboard pilot, small UAS crewmembers rely on the command and control link to operate the aircraft. For example, an uplink transmits command instructions to the aircraft and a downlink transmits the status of the aircraft and provides situational awareness to the Remote PIC or person manipulating the controls.

Lost link is an interruption or loss of the control link between the control station and the unmanned aircraft, preventing control of the aircraft. As a result, the unmanned aircraft performs pre-set lost link procedures. Such procedures ensure that the unmanned aircraft:

Remains airborne in a predictable or planned maneuver, allowing time to re-establish the communication link
Autolands, if available, after a predetermined length of time or terminates the flight when the power source is depleted
A lost link is an abnormal situation, but not an emergency. A lost link is not considered a flyaway.

Lost Link: Pre-Flight Preparations
Follow the manufacturer’s recommendations for programming lost link procedures prior to the flight.

Examples of lost link procedures may include, when applicable:

A lost link route of flight that avoids flight over populated areas
Communications procedures
Plan contingency measures in the event recovery of the small UAS is not feasible.

Contingency Planning
Contingency planning should include an alternate landing/recovery site to be used in the event of an abnormal condition that requires a precautionary landing away from the original launch location.

Incorporate the means of communication with ATC throughout the descent and landing (if required for the flight operation) as well as a plan for ground operations and securing/parking the aircraft on the ground. This includes the availability of control stations capable of launch/recovery, communication equipment, and an adequate power source to operate all required equipment.

Take into consideration all airspace constructs and minimize risk to other aircraft by avoiding congested areas to the maximum extent possible.

Flight Termination
Flight termination is the intentional and deliberate process of performing controlled flight to the ground. Flight termination may be part of lost link procedures, or it may be a contingency that you elect to use if further flight of the aircraft cannot be safely achieved, or if other potential hazards exist that require immediate discontinuation of flight.

Execute flight termination procedures if you have exhausted all other contingencies.

Flight termination points (FTPs), if used, or alternative contingency planning measures must:

Be located within power-off glide distance of the aircraft during all phases of flight
Be based on the assumption of an unrecoverable system failure
Take into consideration altitude, winds, and other factors

Flyaways
A flyaway begins as a lost link—an interruption or loss of the control link prevents control of the aircraft. As a result, the unmanned aircraft is not operating in a predicable or planned manner. However in a flyaway, the pre-set lost link procedures are not established or are not being executed by the unmanned aircraft, creating an emergency situation.

If a flyaway occurs while operating in airspace that requires authorization, notify ATC as outlined in the authorization.

Loss of Global Positioning System (GPS)
Global positioning system (GPS) tools can be a valuable resource for flight planning and situational awareness during a small UAS operation.

However, as with manned aviation, Remote PICs in small UAS operations must avoid overreliance on automation and must be prepared to operate the unmanned aircraft manually, if necessary.

Prior to flight, check NOTAMs for any known GPS service disruptions in the planned location of the small UAS operation
Make a plan of action to prevent or minimize damage in the event of equipment malfunction or failure

Risk of Battery Fires
Battery fires pose a significant hazard to a small UAS.

Both Lithium metal and lithium-ion batteries are:

Highly flammable
Capable of self-ignition when a battery short circuits or is overcharged, heated to extreme temperatures, mishandled, or otherwise defective
Subject to thermal runaway
During thermal runaway, lithium metal batteries generate sufficient heat to cause adjacent cells to go into thermal runaway. As a result, the lithium metal cell releases an explosive combination of a flammable electrolyte and molten lithium metal, accompanied by a large pressure pulse.

Source: Safety Alert for Operators (SAFO) 10017, Risks in Transporting Lithium Batteries in Cargo by Aircraft

Preventing Battery Fires: Storage
Ensure careful storage of spare (uninstalled) lithium batteries.

Take the following precautions to prevent a battery fire:

Prevent short circuits by placing each individual battery in the original retail packaging, a separate plastic bag, or a protective pouch or by insulating exposed terminals with tape
Do not allow spare batteries to come in contact with metal objects, such as coins, keys, or jewelry
Take steps to prevent objects from crushing, puncturing, or applying pressure on the battery

Source: SAFO 15010, Carriage of Spare Lithium Batteries in Carry-On and Checked Baggage

Preventing Battery Fires: Preflight
When preparing to conduct small UAS operations, do not use any battery with signs of damage or defect. For example, check carefully for small nicks in the battery casing and be alert for signs of bubbling or warping during charging.

Once the battery is installed and the small UAS takes flight, the Remote PIC or ground crew may not observe a battery fire until it is too late to land the aircraft safely.

If a battery fire occurs, follow any manufacturer guidance for response procedures.

Accident Reporting
The Remote PIC must report any small UAS accident to the FAA, within 10 calendar days of the operation, if any of the following thresholds are met:

Serious injury to any person or any loss of consciousness
Damage to any property, other than the small unmanned aircraft, if the cost is greater than $500 to repair or replace the property (whichever is lower)
File the report:

Electronically, via the FAA online small UAS accident reporting website
By phone to:
The appropriate FAA Regional Operations Center
The nearest Flight Standards District Office (FSDO)
Visit the Resources page to access the accident reporting website or contact information for the FSDOs and Regional Operations Centers (listed in AC 107-2, Small Unmanned Aircraft Systems (sUAS) (as amended))

Accident Reporting: Serious Injury Threshold
Under 14 CFR part 107, a serious injury qualifies as Level 3 or higher on the Abbreviated Injury Scale (AIS) of the Association for the Advancement of Automotive Medicine. This scale is an anatomical scoring system that is widely used by emergency medical personnel.

It would be considered a serious injury if a person requires hospitalization, but the injury is fully reversible including, but not limited to:

Head trauma
Broken bone(s)
Laceration(s) to the skin that requires suturing

Sources: 14 CFR part 107(III)(I)(2); AC 107-2, Small Unmanned Aircraft Systems (sUAS) (as amended)

Accident Reporting: Required Information
If the accident meets the previously described thresholds, report the following key information to FAA.

Category Required Information
Remote PIC Information
Name
Contact Information
FAA Airman Certificate Number
Aircraft Information

Registration Number (N-number or unique identifier issued in accordance with 14 CFR part 48)
Accident Information
Location of the Accident
Date and Time of the Accident
Person(s) Injured and Extent of Injury (if any or known)
Property Damaged and Extent of Damage (if any or known)
Description of What Happened
In addition to this FAA report, and in accordance with the criteria established by the National Transportation Safety Board (NTSB), certain small UAS accidents must also be reported to the NTSB.

Review Questions
Scenario 1: During your preflight inspection, you discover a small nick in the casing of your small UAS battery.
What action should you take?

A Throw it away with your household trash.
B Use it as long as it will still hold a charge.
*C Follow the manufacturer’s guidance.

Scenario 2: You are part of a news crew, operating a small UAS to cover a breaking story. You experience a flyaway during landing. The unmanned aircraft strikes a vehicle, causing approximately $800 worth of damage.

When must you report the accident to the FAA?
Within 10 days

Summary
This lesson examined abnormal and emergency small UAS situations and accident reporting requirements.

In summary, the Remote PIC must take swift and appropriate action to avoid hazards and respond to equipment malfunction or failure. Should these response efforts fail and result in serious injury or damages, the Remote PIC must report the accident to FAA within 10 calendar days.

You should now be able to:

Identify common abnormal and emergency situations during small UAS operations
Identify requirements for reporting small UAS accidents

Section 336 of Public Law 112-95:

October 26th, 2017

This law is referenced in CFR 107 and at various other points in the sUAS documents so I thought it would be good to look it up. It is short, so I copied it.

SEC. 336. SPECIAL RULE FOR MODEL AIRCRAFT.

(a) IN GENERAL.—Notwithstanding any other provision of law relating to the incorporation of unmanned aircraft systems into Federal Aviation Administration plans and policies, including this subtitle, the Administrator of the Federal Aviation Administration may not promulgate any rule or regulation regarding a model aircraft, or an aircraft being developed as a model aircraft, if—

(1) the aircraft is flown strictly for hobby or recreational use;
(2) the aircraft is operated in accordance with a community-based set of safety guidelines and within the programming of a nationwide community-based organization;
(3) the aircraft is limited to not more than 55 pounds unless otherwise certified through a design, construction, inspection, flight test, and operational safety program adminis- tered by a community-based organization;
(4) the aircraft is operated in a manner that does not interfere with and gives way to any manned aircraft; and
(5) when flown within 5 miles of an airport, the operator of the aircraft provides the airport operator and the airport air traffic control tower (when an air traffic facility is located at the airport) with prior notice of the operation (model aircraft operators flying from a permanent location within 5 miles of an airport should establish a mutually-agreed upon operating procedure with the airport operator and the airport air traffic control tower (when an air traffic facility is located at the airport)).

(b) STATUTORY CONSTRUCTION.—Nothing in this section shall be construed to limit the authority of the Administrator to pursue enforcement action against persons operating model aircraft who endanger the safety of the national airspace system.

(c) MODEL AIRCRAFT DEFINED.—In this section, the term ‘‘model aircraft’’ means an unmanned aircraft that is—

(1) capable of sustained flight in the atmosphere;
(2) flown within visual line of sight of the person operating the aircraft; and
(3) flown for hobby or recreational purposes.

sUAS Sample Exam 2017-06-12

October 26th, 2017

Date effective: June 12, 2017 The following sample exam for Unmanned Aircraft General (UAG) is suitable study material for the Remote Pilot Certificate with a small UAS Rating. These questions are a representation of questions that can be found on all Unmanned Aircraft General tests. The applicant must realize that these questions are to be used as a study guide, and are not necessarily actual test questions. The full UAG test contains 60 questions.

Matching the learning statement codes with the codes listed on your Airman Knowledge Test Report assists in the evaluation of knowledge areas missed on your exam. It is available at http://www.faa.gov/training_testing/testing/media/LearningStatementReferenceGuide.pdf.

To see the answer, highlight the area next to the — as if you were going to copy it.

Sample UAG Exam with ACS Codes:

1. PLT064 UA.V.B.K6a (Refer to FAA-CT-8080-2G, Figure 21.) What airport is located approximately 47 (degrees) 40 (minutes) N latitude and 101 (degrees) 26 (minutes) W longitude?
A) Mercer County Regional Airport.
B) Semshenko Airport.
C) Garrison Airport.

Aeronautical Chart Users Guide 12th Edition A quadrant on Sectionals is the area bounded by ticked lines dividing each 30 minutes of latitude and each 30 minutes of longitude. On the chart we see 48° North, so the line of latitude below it is 47° 30′. We also see 101° longitude on the chart so the quadrant containing the red-circled 2 must be the one containing the airport. Each of the long hash marks is 10′ so if we count up from 47° 30′ by one hash mark, we are just above the Lake Nettie Refuge. Count over from the 101° longitude line and we are near the power plant. Garrison is the airport near those coordinates. If we look up Garrison in the Chart Supplement we can verify that its coordinates are N47°39.36′ W101°26.21′.

2. PLT064 UA.V.B.K6a (Refer to FAA-CT-8080-2G, Figure 26.) What does the line of latitude at area 4 measure?
A) The degrees of latitude east and west of the Prime Meridian.
B) The degrees of latitude north and south of the equator.
C) The degrees of latitude east and west of the line that passes through Greenwich, England.

Remote Pilot Study Guide Latitude and Longitude (Meridians and Parallels) The equator is an imaginary circle equidistant from the poles of the Earth. Circles parallel to the equator (lines running east and west) are parallels of latitude. They are used to measure degrees of latitude north (N) or south (S) of the equator. The angular distance from the equator to the pole is one-fourth of a circle or 90°. The 48 conterminous states of the United States are located between 25° and 49° N latitude. The arrows in Figure 11-3 labeled “Latitude” point to lines of latitude. Meridians of longitude are drawn from the North Pole to the South Pole and are at right angles to the Equator. The “Prime Meridian,” which passes through Greenwich, England, is used as the zero line from which measurements are made in degrees east (E) and west (W) to 180°. The 48 conterminous states of the United States are between 67° and 125° W longitude.

3. PLT040 UA.II.A.K1b (Refer to FAA-CT-8080-2G, Figure 23, area 3.) What is the floor of the Savannah Class C airspace at the shelf area (outer circle)?
A) 1,300 feet AGL.
B) 1,300 feet MSL.
C) 1,700 feet MSL.

Aeronautical Chart Users Guide 12th Edition The MSL ceiling and floor altitudes of each sector are shown in solid magenta figures with the last two zeros omitted. Savannah is on the lower left of the chart and the outer ring is labeled 41/13 so the floor is 1,300′ MSL.

4. PLT064 UA.II.A.K2 (Refer to FAA-CT-8080-2G, Figure 59, area 2.) The chart shows a gray line with “VR1667, VR1617, VR1638, and VR1668.” Could this area present a hazard to the operations of a small UA?
A) No, all operations will be above 400 feet.
B) Yes, this is a Military Training Route from the surface to 1,500 feet AGL.
C) Yes, the defined route provides traffic separation to manned aircraft.

Aeronautical Chart Users Guide 12th Edition MTRs are identified by designators (IR-107, VR-134) which are shown in brown on the route centerline. Arrows are shown to indicate the direction of ight along the route. The width of the route determines the width of the line that is plotted on the chart:
There are IFR (IR) and VFR (VR) routes as follows: Route identification:
a. Routes at or below 1500’ AGL (with no segment above 1500’) are identified by four-digit numbers; e.g., VR1007, etc. These routes are generally developed for flight under Visual Flight Rules.
b. Routes above 1500’ AGL (some segments of these routes may be below 1500’) are identified by three or fewer digit numbers; e.g., IR21, VR302, etc. These routes are developed for flight under Instrument Flight Rules.

5. PLT161 UA.II.A.K1b According to 14 CFR part 107 the remote pilot in command (PIC) of a small unmanned aircraft planning to operate within Class C airspace
A) must use a visual observer.
B) is required to file a flight plan.
C) is required to receive ATC authorization.

CFR §107.41 Operation in certain airspace.
No person may operate a small unmanned aircraft in Class B, Class C, or Class D airspace or within the lateral boundaries of the surface area of Class E airspace designated for an airport unless that person has prior authorization from Air Traffic Control (ATC).

6. PLT064 UA.II.A.K2 (Refer to FAA-CT-8080-2G, Figure 21.) You have been hired by a farmer to use your small UA to inspect his crops. The area that you are to survey is in the Devil’s Lake West MOA, east of area 2. How would you find out if the MOA is active?
A) Refer to the chart legend.
B) This information is available in the Small UAS database.
C) Refer to the Military Operations Directory.

Aeronautical Chart Users Guide 12th Edition CHART TABULATIONS Special Use Airspace (SUA): Prohibited, Restricted and Warning Areas are presented in blue and listed numerically for U.S. and other countries. Restricted, Danger and Advisory Areas outside the U.S. are tabulated separately in blue. A tabulation of Alert Areas (listed numerically) and Military Operations Areas (MOA) (listed alphabetically) appear on the chart in magenta. All are supplemented with altitude, time of use and the controlling agency/contact facility, and its frequency when available. The controlling agency will be shown when the contact facility and frequency data is unavailable.

7. PLT037 UA.II.B.K5 (Refer to FAA-CT-8080-2G, Figure 20, area 5.) How would a remote PIC “CHECK NOTAMS” as noted in the CAUTION box regarding the unmarked balloon?
A) By utilizing the B4UFLY mobile application.
B) By contacting the FAA district office.
C) By obtaining a briefing via an online source such as: 1800WXBrief.com.

Remote Pilot Study Guide NOTAMs are available… online at PilotWeb, which provides access to current NOTAM information. Local airport NOTAMs can be obtained online from various websites.

AIM 5−1−1. Preflight Preparation
a. Every pilot is urged to receive a preflight briefing and to file a flight plan. This briefing should consist of the latest or most current weather, airport, and en route NAVAID information.
d. FSSs are required to advise of pertinent NOTAMs if a standard briefing is requested, but if they are overlooked, don’t hesitate to remind the specialist that you have not received NOTAM information.
Lockheed Martin Flight Services
Internet Access: http://www.1800wxbrief.com For customer service: (866) 936−6826

AC 107-2 Notices to Airmen (NOTAM). Information on how to obtain NOTAMs can be found at PilotWeb.

8. PLT313 UA.IV.A.K1b To ensure that the unmanned aircraft center of gravity (CG) limits are not exceeded, follow the aircraft loading instructions specified in the
A) Pilot`s Operating Handbook or UAS Flight Manual.
B) Aeronautical Information Manual (AIM).
C) Aircraft Weight and Balance Handbook.

Remote Pilot Study Guide Adverse balance conditions (i.e., weight distribution) may affect flight characteristics in much the same manner as those mentioned for an excess weight condition. Limits for the location of the center of gravity (CG) may be established by the manufacturer.

From the course materials, As with any aircraft, compliance with weight and balance limits is critical to the safety of flight for a small UAS. An unmanned aircraft that is loaded out of balance may exhibit unexpected and unsafe flight characteristics.

Before any flight, verify that the unmanned aircraft is correctly loaded by determining the weight and balance condition.

Review any available manufacturer weight and balance data and follow all warnings, cautions, notes, and limitations.

If the manufacturer does not provide specific weight and balance data, apply general weight and balance principals to determine limits for a given flight. For example, add weight to the unmanned aircraft in a manner that does not adversely affect the aircraft’s center of gravity (CG) location—a point at which the unmanned aircraft would balance if it were suspended at that point.

From AC 107-2 1. Weight and Balance (W&B).
Before any flight, the remote PIC should verify the aircraft is correctly loaded by determining the W&B condition of the aircraft. An aircraft’s W&B restrictions established by the manufacturer or the builder should be closely followed. Compliance with the manufacturer’s W&B limits is critical to flight safety. The remote PIC must consider the consequences of an overweight aircraft if an emergency condition arises.

9. PLT310 UA.IV.A.K1a When operating an unmanned airplane, the remote pilot should consider that the load factor on the wings may be increased any time
A) the CG is shifted rearward to the aft CG limit.
B) the airplane is subjected to maneuvers other than straight-and-level flight.
C) the gross weight is reduced.

The Remote Pilot Study Guide Chapter 4 Load Factors in Steep Turns has a detailed description of how the load factor is affected by turns.

FAA-H-8083-25, Pilot’s Handbook of Aeronautical Knowledge, Chapter 5 Critical load factors apply to all flight maneuvers except unaccelerated straight flight where a load factor of 1 G is always present.

10. PLT312 UA.IV.A.K1b A stall occurs when the smooth airflow over the unmanned airplane`s wing is disrupted and the lift degenerates rapidly. This is caused when the wing
A) exceeds the maximum speed.
B) exceeds maximum allowable operating weight.
C) exceeds its critical angle of attack.

FAA-H-8083-3, Airplane Flying Handbook, Chapter 4 A stall is an aerodynamic condition which occurs when smooth airflow over the airplane’s wings is disrupted, resulting in loss of lift. Specifically, a stall occurs when the AOA—the angle between the chord line of the wing and the relative wind—exceeds the wing’s critical AOA. It is possible to exceed the critical AOA at any airspeed, at any attitude, and at any power setting.

11. PLT309 UA.IV.A.K1a (Refer to FAA-CT-8080-2G, Figure 2.) If an unmanned airplane weighs 33 pounds, what approximate weight would the airplane structure be required to support during a 30° banked turn while maintaining altitude?
A) 34 pounds.
B) 47 pounds.
C) 38 pounds.

This figure is taken from Remote Pilot Study Guide Chapter 4 Load Factors in Steep Turns and is also found in FAA-H-8083-25, Pilot’s Handbook of Aeronautical Knowledge, Chapter 5 Load Factors and Stalling Speeds. We can see from the chart that the load factor increase is small, on the side of the chart we see that it is 1.154. Multiply the load factor by the original weight to get 38 lbs.

12. PLT205 UA.V.E.K2 Which is true regarding the presence of alcohol within the human body?
A) A small amount of alcohol increases vision acuity.
B) Consuming an equal amount of water will increase the destruction of alcohol and alleviate a hangover.
C) Judgment and decision-making abilities can be adversely affected by even small amounts of alcohol.

Remote Pilot Study Guide Alcohol impairs the efficiency of the human body. Studies have shown that consuming alcohol is closely linked to performance deterioration. Pilots must make hundreds of decisions, some of them time-critical, during the course of a flight. The safe outcome of any flight depends on the ability to make the correct decisions and take the appropriate actions during routine occurrences, as well as abnormal situations. The influence of alcohol drastically reduces the chances of completing a flight without incident. Even in small amounts, alcohol can impair judgment, decrease sense of responsibility, affect coordination, constrict visual field, diminish memory, reduce reasoning ability, and lower attention span. As little as one ounce of alcohol can decrease the speed and strength of muscular reflexes, lessen the efficiency of eye movements while reading, and increase the frequency at which errors are committed. Impairments in vision and hearing can occur from consuming as little as one drink.

AC 107-2 It is the remote PIC’s responsibility to ensure all crewmembers are not participating in the operation while impaired. While drug and alcohol use are known to impair judgment, certain over-the-counter medications and medical conditions could also affect the ability to safely operate a small UA.

13. PLT441 UA.V.C.K1 When using a small UA in a commercial operation, who is responsible for briefing the participants about emergency procedures?
A) The FAA inspector-in-charge.
B) The lead visual observer.
C) The remote PIC.

CFR §107.49 Preflight familiarization, inspection, and actions for aircraft operation.
Prior to flight, the remote pilot in command must:
(b) Ensure that all persons directly participating in the small unmanned aircraft operation are informed about the operating conditions, emergency procedures, contingency procedures, roles and responsibilities, and potential hazards;

14. PLT403 UA.V.C.K1 To avoid a possible collision with a manned airplane, you estimate that your small UA climbed to an altitude greater than 600 feet AGL. To whom must you report the deviation?
A) Air Traffic Control.
B) The National Transportation Safety Board.
C) Upon request of the Federal Aviation Administration.

CFR §107.21 In-flight emergency.
(b) Each remote pilot in command who deviates from a rule under paragraph (a) of this section must, upon request of the Administrator, send a written report of that deviation to the Administrator.

15. PLT146 UA.V.A.K3 (Refer to FAA-CT-8080-2G, Figure 26, area 2.) While monitoring the Cooperstown CTAF you hear an aircraft announce that they are midfield left downwind to RWY 13. Where would the aircraft be relative to the runway?
A) The aircraft is East.
B) The aircraft is South.
C) The aircraft is West.

Runways are depicted on the charts in a blue or magenta circle. The orientation of the white line is the orientation of the runway with respect to true north. AIM 2-3-3 Runway Designators. Runway numbers and letters are determined from the approach direction. The runway number is the whole number nearest one-tenth the magnetic azimuth of the centerline of the runway, measured clockwise from the magnetic north. So depending on the magnetic deviation, there might be a difference in the orientation on the chart versus the name. In this case, there is one runway. The runway oriented northwest to southeast is RWY 13. Refer to AIM FIG 4−3−2 Traffic Pattern Operations Single Runway for a description of the labels of a traffic pattern. An aircraft on the left downwind would be to the East of the runway.

16. PLT446 UA.V.F.K1 Under what condition should the operator of a small UA establish scheduled maintenance protocol?
A) When the manufacturer does not provide a maintenance schedule.
B) UAS does not need a required maintenance schedule.
C) When the FAA requires you to, following an accident.

AC 107-2 7.2 Maintenance. sUAS maintenance includes scheduled and unscheduled overhaul, repair, inspection, modification, replacement, and system software upgrades of the sUAS and its components necessary for flight. Whenever possible, the operator should maintain the sUAS and its components in accordance with manufacturer’s instructions. The aircraft manufacturer may provide the maintenance program, or, if one is not provided, the applicant may choose to develop one.

17. PLT372 UA.V.F.K2 According to 14 CFR part 107, the responsibility to inspect the small UAS to ensure it is in a safe operating condition rests with the
A) remote pilot-in-command.
B) visual observer.
C) owner of the small UAS.

CFR §107.15 Condition for safe operation.
(a) No person may operate a civil small unmanned aircraft system unless it is in a condition for safe operation. Prior to each flight, the remote pilot in command must check the small unmanned aircraft system to determine whether it is in a condition for safe operation.

18. PLT103 UA.V.D.K4 Identify the hazardous attitude or characteristic a remote pilot displays while taking risks in order to impress others?
A) Impulsivity.
B) Invulnerability.
C) Macho.

Remote Pilot Study Guide Figure 10-2 and FAA-H-8083-25, Pilot’s Handbook of Aeronautical Knowledge, Figure 2-4 The five hazardous attitudes identified through past and contemporary study. Invulnerability “It won’t happen to me.” Many people falsely believe that accidents happen to others, but never to them. They know accidents can happen, and they know that anyone can be affected. However, they never really feel or believe that they will be personally involved. Pilots who think this way are more likely to take chances and increase risk.

19. PLT272 UA.V.E.K5 You are a remote pilot for a co-op energy service provider. You are to use your UA to inspect power lines in a remote area 15 hours away from your home office. After the drive, fatigue impacts your abilities to complete your assignment on time. Fatigue can be recognized
A) easily by an experienced pilot.
B) as being in an impaired state.
C) by an ability to overcome sleep deprivation.

Remote Pilot Study Guide Fatigue is frequently associated with pilot error. Some of the effects of fatigue include degradation of attention and concentration, impaired coordination, and decreased ability to communicate. These factors seriously influence the ability to make effective decisions.

Remote Pilot Study Guide Fatigue continues to be one of the most insidious hazards to flight safety, as it may not be apparent to a pilot until serious errors are made.

FAA-H-8083-25, Pilot’s Handbook of Aeronautical Knowledge Exhaustion Pilots who become fatigued during a night flight will not be mentally alert and will respond more slowly to situations requiring immediate action. Exhausted pilots tend to concentrate on one aspect of a situation without considering the total requirement. Their performance may become a safety hazard depending on the degree of fatigue and instead of using proper scanning techniques may get fixated on the instruments or stare off rather than multitask.

20. PLT104 UA.V.D.K1 Safety is an important element for a remote pilot to consider prior to operating an unmanned aircraft system. To prevent the final “link” in the accident chain, a remote pilot must consider which methodology?
A) Crew Resource Management.
B) Safety Management System.
C) Risk Management.

The answer to this question is most likely A. Risk Assessment is a term that is used in AC 107-2 but Risk Management is not mentioned. Safety Management System is a term that the FAA uses in relation to airports, but not pilots. Crew Resource Management is mentioned in the FAR as one of the areas tested in the Knowledge Test so it is probably the answer.

21. PLT104 UA.V.D.K2 When adapting crew resource management (CRM) concepts to the operation of a small UA, CRM must be integrated into
A) the flight portion only.
B) all phases of the operation.
C) the communications only.

AC 107-2 A.2.5 A characteristic of CRM is creating an environment where open communication is encouraged and expected, and involves the entire crew to maximize team performance. Many of the same resources that are available to manned aircraft operations are available to UAS operations. For example, remote PICs can take advantage of traditional CRM. These crewmembers can provide information about traffic, airspace, weather, equipment, and aircraft loading and performance.

22. PLT103 UA.V.D.K4 You have been hired as a remote pilot by a local TV news station to film breaking news with a small UA. You expressed a safety concern and the station manager has instructed you to `fly first, ask questions later.` What type of hazardous attitude does this attitude represent?
A) Machismo.
B) Invulnerability.
C) Impulsivity.

Remote Pilot Study Guide Figure 10-2 and FAA-H-8083-25, Pilot’s Handbook of Aeronautical Knowledge, Figure 2-4 The five hazardous attitudes identified through past and contemporary study. Impulsivity “Do it quickly.” This is the attitude of people who frequently feel the need to do something, anything, immediately. They do not stop to think about what they are about to do, they do not select the best alternative, and they do the first thing that comes to mind.

23. PLT271 UA.V.D.K1 A local TV station has hired a remote pilot to operate their small UA to cover news stories. The remote pilot has had multiple near misses with obstacles on the ground and two small UAS accidents. What would be a solution for the news station to improve their operating safety culture?
A) The news station should implement a policy of no more than five crashes/incidents within 6 months.
B) The news station does not need to make any changes; there are times that an accident is unavoidable.
C) The news station should recognize hazardous attitudes and situations and develop standard operating procedures that emphasize safety.

Remote Pilot Study Guide During each flight, the single pilot makes many decisions under hazardous conditions. To fly safely, the pilot needs to assess the degree of risk and determine the best course of action to mitigate the risk.

FAA-H-8083-25, Pilot’s Handbook of Aeronautical Knowledge, Chapter 2 Aeronautical Decision Making Being fit to fly depends on more than just a pilot’s physical condition and recent experience. For example, attitude affects the quality of decisions. Attitude is a motivational predisposition to respond to people, situations, or events in a given manner.

It is not clear from the question whether the incidents are due to hazardous attitudes, lack of skill, or a combination of both. In any case, the remote pilot in command has not adequately assessed the risk of flying in these situations.

24. PLT064 UA.V.B.K6a (Refer to FAA-CT-8080-2G, Figure 22, area 2.) At Coeur D`Alene which frequency should be used as a Common Traffic Advisory Frequency (CTAF) to monitor airport traffic?
A) 122.05 MHz.
B) 135.075 MHz.
C) 122.8 MHz.

Aeronautical Chart Users Guide 12th Edition A ‘C’ in a dark circle follows the Common Traffic Advisory Frequency (CTAF). In this case it shows 122.8.

You could also purchase or download a copy of the Chart Supplement for your area. Remote Pilot Study Guide The Chart Supplement U.S. (formerly Airport/Facility Directory) provides the most comprehensive information on a given airport. It contains information on airports, heliports, and seaplane bases that are open to the public.

25. PLT101 UA.V.B.K6a (Refer to FAA-CT-8080-2G, Figure 26, area 4.) You have been hired to inspect the tower under construction at 46.9N and 98.6W, near Jamestown Regional (JMS). What must you receive prior to flying your unmanned aircraft in this area?
A) Authorization from the military.
B) Authorization from ATC.
C) Authorization from the National Park Service.

This is an odd way of writing the coordinates of the tower, but if we assume that they mean N46°54″ W98°36″ then it lies in the airspace for JMS and according to CFR §107.41 Operation in certain airspace. No person may operate a small unmanned aircraft in Class B, Class C, or Class D airspace or within the lateral boundaries of the surface area of Class E airspace designated for an airport unless that person has prior authorization from Air Traffic Control (ATC).

26. PLT064 UA.V.B.K6a (Refer to FAA-CT-8080-2G, Figure 20, area 3.) With ATC authorization, you are operating your small unmanned aircraft approximately 4 SM southeast of Elizabeth City Regional Airport (ECG). What hazard is indicated to be in that area?
A) High density military operations in the vicinity.
B) Unmarked balloon on a cable up to 3,008 feet AGL.
C) Unmarked balloon on a cable up to 3,008 feet MSL.

The charted box clearly indicates that there is an unmarked balloon up to 3,008 feet MSL.

27. PLT281 UA.V.B.K6b The most comprehensive information on a given airport is provided by
A) the Chart Supplements U.S. (formerly Airport Facility Directory).
B) Notices to Airmen (NOTAMS).
C) Terminal Area Chart (TAC).

Remote Pilot Study Guide The Chart Supplement U.S. (formerly Airport/Facility Directory) provides the most comprehensive information on a given airport. It contains information on airports, heliports, and seaplane bases that are open to the public.

28. PLT454 UA.I.B.K20 According to 14 CFR part 107, who is responsible for determining the performance of a small unmanned aircraft?
A) Remote pilot-in-command.
B) Manufacturer.
C) Owner or operator.

CFR §107.15 Condition for safe operation.
(a) No person may operate a civil small unmanned aircraft system unless it is in a condition for safe operation. Prior to each flight, the remote pilot in command must check the small unmanned aircraft system to determine whether it is in a condition for safe operation.

29. PLT194 UA.I.B.K14a Which technique should a remote pilot use to scan for traffic? A remote pilot should
A) systematically focus on different segments of the sky for short intervals.
B) concentrate on relative movement detected in the peripheral vision area.
C) continuously scan the sky from right to left.

Remote Pilot Study Guide To scan effectively, pilots must look from right to left or left to right. They should begin scanning at the greatest distance an object can be perceived (top) and move inward toward the position of the aircraft (bottom). For each stop, an area approximately 30° wide should be scanned. The duration of each stop is based on the degree of detail that is required, but no stop should last longer than 2 to 3 seconds. When moving from one viewing point to the next, pilots should overlap the previous field of view by 10°.

FAA-H-8083-25, Pilot’s Handbook of Aeronautical Knowledge Effective scanning is accomplished with a series of short, regularly spaced eye movements that bring successive areas of the sky into the central visual field. Each movement should not exceed 10°, and each should be observed for at least 1 second to enable detection. Although back and forth eye movements seem preferred by most pilots, each pilot should develop a scanning pattern that is most comfortable and then adhere to it to assure optimum scanning.

30. PLT530 UA.I.B.K1 Under what condition would a small UA not have to be registered before it is operated in the United States?
A) When the aircraft weighs less than .55 pounds on takeoff, including everything that is on-board or attached to the aircraft.
B) When the aircraft has a takeoff weight that is more than .55 pounds, but less than 55 pounds, not including fuel and necessary attachments.
C) All small UAS need to be registered regardless of the weight of the aircraft before, during, or after the flight.

You need to register your aircraft if it weighs between 0.55 lbs. (250 grams) and up to 55 lbs. (25 kg) and you are not flying under the Special Rule for Model Aircraft. Register My sUAS

The lower limit is contained in CFR §48.15 Requirement to register. CFR §48.15 Requirement to register.
No person may operate a small unmanned aircraft that is eligible for registration under 49 U.S.C. 44101-44103 unless one of the following criteria has been satisfied:
(a) The owner has registered and marked the aircraft in accordance with this part;
(b) The aircraft weighs 0.55 pounds or less on takeoff, including everything that is on board or otherwise attached to the aircraft;

The upper limit is defined by a combination of FARs.
CFR §107.3 Definitions.
Small unmanned aircraft means an unmanned aircraft weighing less than 55 pounds on takeoff, including everything that is on board or otherwise attached to the aircraft.

and

CFR §48.5 Compliance dates.
a) Small unmanned aircraft used exclusively as model aircraft.… (b) Small unmanned aircraft used as other than model aircraft. Small unmanned aircraft owners authorized to conduct operations other than model aircraft operations must register the small unmanned aircraft in accordance with part 47 of this chapter…

31. PLT530 UA.I.B.K1 According to 14 CFR part 48, when must a person register a small UA with the Federal Aviation Administration?
A) All civilian small UAs weighing greater than .55 pounds must be registered regardless of its intended use.
B) When the small UA is used for any purpose other than as a model aircraft.
C) Only when the operator will be paid for commercial services.

CFR §48.15 Requirement to register.
No person may operate a small unmanned aircraft that is eligible for registration under 49 U.S.C. 44101-44103 unless one of the following criteria has been satisfied:
(a) The owner has registered and marked the aircraft in accordance with this part;
(b) The aircraft weighs 0.55 pounds or less on takeoff, including everything that is on board or otherwise attached to the aircraft; or
(c) The aircraft is an aircraft of the Armed Forces of the United States.

32. PLT530 UA.I.B.K1 According to 14 CFR part 48, when would a small UA owner not be permitted to register it?
A) If the owner is less than 13 years of age.
B) All persons must register their small UA.
C) If the owner does not have a valid United States driver`s license.

CFR §48.25 Applicants.
(b) A small unmanned aircraft must be registered by its owner using the legal name of its owner, unless the owner is less than 13 years of age. If the owner is less than 13 years of age, then the small unmanned aircraft must be registered by a person who is at least 13 years of age.

33. PLT161 UA.I.B.K16 According to 14 CFR part 107, how may a remote pilot operate an unmanned aircraft in Class C airspace?
A) The remote pilot must have prior authorization from the Air Traffic Control (ATC) facility having jurisdiction over that airspace.
B) The remote pilot must monitor the Air Traffic Control (ATC) frequency from launch to recovery.
C) The remote pilot must contact the Air Traffic Control (ATC) facility after launching the unmanned aircraft.

CFR §107.41 Operation in certain airspace.
No person may operate a small unmanned aircraft in Class B, Class C, or Class D airspace or within the lateral boundaries of the surface area of Class E airspace designated for an airport unless that person has prior authorization from Air Traffic Control (ATC).

34. PLT119 UA.I.B.K9 According to 14 CFR part 107, what is required to operate a small UA within 30 minutes after official sunset?
A) Use of anti-collision lights.
B) Must be operated in a rural area.
C) Use of a transponder.

CFR §1
Night means the time between the end of evening civil twilight and the beginning of morning civil twilight, as published in the Air Almanac, converted to local time.

CFR §107.29 Daylight operation.
(a) No person may operate a small unmanned aircraft system during night.
(b) No person may operate a small unmanned aircraft system during periods of civil twilight unless the small unmanned aircraft has lighted anti-collision lighting visible for at least 3 statute miles. The remote pilot in command may reduce the intensity of the anti-collision lighting if he or she determines that, because of operating conditions, it would be in the interest of safety to do so.

Note that civil twilight is about a half hour after sunset and before sunrise so anti-collision lights are required if operating in that period.

35. PLT301 UA.III.B.K1j You have received an outlook briefing from flight service through 1800wxbrief.com. The briefing indicates you can expect a low-level temperature inversion with high relative humidity. What weather conditions would you expect?
A) Smooth air, poor visibility, fog, haze, or low clouds.
B) Light wind shear, poor visibility, haze, and light rain.
C) Turbulent air, poor visibility, fog, low stratus type clouds, and showery precipitation.

Remote Pilot Study Guide When the temperature of the air rises with altitude, a temperature inversion exists. Inversion layers are commonly shallow layers of smooth, stable air close to the ground. The temperature of the air increases with altitude to a certain point, which is the top of the inversion. The air at the top of the layer acts as a lid, keeping weather and pollutants trapped below. If the relative humidity of the air is high, it can contribute to the formation of clouds, fog, haze, or smoke resulting in diminished visibility in the inversion layer.

FAA-H-8083-25, Pilot’s Handbook of Aeronautical Knowledge Inversion As air rises and expands in the atmosphere, the temperature decreases. There is an atmospheric anomaly that can occur; however, that changes this typical pattern of atmospheric behavior. When the temperature of the air rises with altitude, a temperature inversion exists. Inversion layers are commonly shallow layers of smooth, stable air close to the ground. The temperature of the air increases with altitude to a certain point, which is the top of the inversion. The air at the top of the layer acts as a lid, keeping weather and pollutants trapped below. If the relative humidity of the air is high, it can contribute to the formation of clouds, fog, haze, or smoke resulting in diminished visibility in the inversion layer.

36. PLT351 UA.III.B.K1a What effect does high density altitude have on the efficiency of a UA propeller?
A) Propeller efficiency is increased.
B) Propeller efficiency is decreased.
C) Density altitude does not affect propeller efficiency.

FAA-H-8083-25, Pilot’s Handbook of Aeronautical Knowledge The density of the air, of course, has a pronounced effect on aircraft and engine performance. Regardless of the actual altitude at which the aircraft is operating, it will perform as though it were operating at an altitude equal to the existing density altitude.… High density altitude refers to thin air while low density altitude refers to dense air. The conditions that result in a high density altitude are high elevations, low atmospheric pressures, high temperatures, high humidity, or some combination of these factors.

37. PLT511 UA.III.B.K1d What are characteristics of a moist, unstable air mass?
A) Turbulence and showery precipitation.
B) Poor visibility and smooth air.
C) Haze and smoke.

Remote Pilot Study Guide Stability of an air mass determines its typical weather characteristics. When one type of air mass overlies another, conditions change with height. Characteristics typical of an unstable and a stable air mass are as follows:
Unstable Air: Cumuliform clouds, Showery precipitation, Rough air (turbulence), Good visibility (except in blowing obstructions).
Stable Air: Stratiform clouds and fog, Continuous precipitation, Smooth air, Fair to poor visibility in haze and smoke.

FAA-H-8083-25, Pilot’s Handbook of Aeronautical Knowledge Moist, unstable air causes cumulus clouds, showers, and turbulence to form.

38. PLT173 UA.III.B.K1c What are the characteristics of stable air?
A) Good visibility and steady precipitation.
B) Poor visibility and steady precipitation.
C) Poor visibility and intermittent precipitation.

Remote Pilot Study Guide Stability of an air mass determines its typical weather characteristics. When one type of air mass overlies another, conditions change with height. Characteristics typical of an unstable and a stable air mass are as follows:
Unstable Air: Cumuliform clouds, Showery precipitation, Rough air (turbulence), Good visibility (except in blowing obstructions).
Stable Air: Stratiform clouds and fog, Continuous precipitation, Smooth air, Fair to poor visibility in haze and smoke.

FAA-H-8083-25, Pilot’s Handbook of Aeronautical Knowledge …a stable air mass with poor surface visibility. The poor surface visibility is due to the fact that smoke, dust, and other particles cannot rise out of the air mass and are instead trapped near the surface. A stable air mass can produce low stratus clouds and fog.

AC 00-6B Aviation Weather Stability of an air mass determines its typical weather characteristics. When one type of air mass overlies another, conditions change with height. Characteristics typical of an unstable and a stable air mass are as follows:
Unstable air: Cumuliform clouds, Showery precipitation, Rough air (turbulence), Good Visibility.
Stable air; Stratiform clouds and fog, Continuous precipitation, Smooth air, Fair to poor visibility in haze and smoke.

39. PLT059 UA.III.A.K2 (Refer to FAA-CT-8080-2G, Figure 12.) The wind direction and velocity at KJFK is from
A) 180° true at 4 knots.
B) 180° magnetic at 4 knots.
C) 040° true at 18 knots.

Remote Pilot Study Guide Chapter 3a: Aviation Weather Sources has a good description of how to decode METARs and TAFs.

Aviation Weather also does a good job of explaining how to decode. The METARs site lets you see the decoded weather next to the coded weather so you can test yourself. They are produced by NOAA and WIND DIRECTION AND SPEED: Direction in tens of degrees from true north (first three digits); next two digits: speed in whole knots; as needed Gusts (character) followed by maximum observed speed; always followed by KT to indicate knots; 00000KT for calm>

18004kt translates to 180° true and 4 kts.

40. PLT059 UA.III.A.K2 (Refer to FAA-CT-8080-2G, Figure 12.) What are the current conditions for Chicago Midway Airport (KMDW)?
A) Sky 700 feet overcast, visibility 1-1/2SM, rain.
B) Sky 7,000 feet overcast, visibility 1-1/2SM, heavy rain.
C) Sky 700 feet overcast, visibility 11, occasionally 2SM, with rain.

Remote Pilot Study Guide Chapter 3a: Aviation Weather Sources has a good description of how to decode METARs and TAFs.

Aviation Weather also does a good job of explaining how to decode. Aviation Weather does a good job of explaining it. The METARs site lets you see the decoded weather next to the coded weather so you can test yourself. They are produced by NOAA and
VISIBILITY: Prevailing visibility in statue miles and fractions (space between whole miles and fractions); always followed by SM to indicate statute miles; values less than 1/4 reported as M1/4SM.

OVC 007 is overcast at 700 feet, 1 1/2SM translates to visibilty of 1-1/2SM, RA is rain.

Exam: Part 107 small Unmanned Aircraft Systems (sUAS)

October 26th, 2017

Welcome to the exam for Part 107 small Unmanned Aircraft Systems (sUAS).
You must complete the entire exam in one session. If the exam is not completed and graded in 90 minutes or less, you will need to retake the entire exam when you log into the course again.
You may review course material as you take the test. Many questions have references links available. (A separate window will open. Close that window when ready to continue with this exam.)
When complete, press the “Grade Exam” button at the bottom.
If you get wrong answers you will be brought back to the exam with the incorrect answers marked.
You must get 100% to pass the exam.

1. Which of the following individuals may process an application for a part 107 remote pilot certificate with a small UAS rating? [Sources: 14 CFR parts 107.63 and 61.56]
A) Commercial Balloon pilot
B) Remote Pilot in Command
C) Designated Pilot Examiner

CFR §107.63 Issuance of a remote pilot certificate with a small UAS rating.
(1) The application must be submitted to a Flight Standards District Office, a designated pilot examiner, an airman certification representative for a pilot school, a certificated flight instructor, or other person authorized by the Administrator;

2. After receiving a part 107 remote pilot certificate with a small UAS rating, how often must you satisfy recurrent training requirements? [Sources: 14 CFR part 107.63 and 107.65; AC 107-2, Small Unmanned Aircraft Systems (sUAS) (as amended)]
A) Every 24 months
B) Every 12 months
C) Every 6 months

CFR §107.65 Aeronautical knowledge recency.
A person may not operate a small unmanned aircraft system unless that person has completed one of the following, within the previous 24 calendar months:
(b) Passed a recurrent aeronautical knowledge test covering the areas of knowledge specified in §107.73(b); or
(c) If a person holds a pilot certificate (other than a student pilot certificate) issued under part 61 of this chapter and meets the flight review requirements specified in §§61.56, passed either an initial or recurrent training course covering the areas of knowledge specified in §107.74(a) or (b) in a manner acceptable to the Administrator.

3. According to 14 CFR part 107, a small UAS is a unmanned aircraft system weighing: [Sources: 14 CFR parts 107.1 and 107.3; AC 107-2, Small UAS (sUAS)(as amended)]
A) 55 lbs
B) Less than 55 lbs
C) 55kg or less

CFR §107.3 Definitions.
Small unmanned aircraft means an unmanned aircraft weighing less than 55 pounds on takeoff, including everything that is on board or otherwise attached to the aircraft.

4. Unmanned aircraft means an aircraft operated: [Sources: 14 CFR parts 107.1 and 107.3; AC 107-2, Small Unmanned Aircraft Systems(sUAS)(as amended)]
A) Autonomously by onboard computers
B) During search and rescue operations other than public
C) Without the possibility of direct human intervention from within or on the aircraft

CFR §107.3 Definitions.
Small unmanned aircraft means an unmanned aircraft weighing less than 55 pounds on takeoff, including everything that is on board or otherwise attached to the aircraft.

5. Which of the following types of operations are excluded from the requirements in part 107? [Sources: 14 CFR parts 101.41 and 107.1]
A) Model aircraft for hobby use flown in accordance with 14 CFR part 101
B) Quadcopter capturing aerial imagery for crop monitoring
C) UAS used for motion picture filming

CFR §107.1 Applicability.
(a) Except as provided in paragraph (b) of this section, this part applies to the registration, airman certification, and operation of civil small unmanned aircraft systems within the United States.

CFR §101.41 Applicability.
This subpart prescribes rules governing the operation of a model aircraft (or an aircraft being developed as a model aircraft) that meets all of the following conditions as set forth in section 336 of Public Law 112-95:
(a) The aircraft is flown strictly for hobby or recreational use;

6. Which of the following operations require adherence to 14 CFR 107? [Sources: 14 CFR parts 101.41 and 107.1]
A) Flying for enjoyment with family and friends
B) Operating your small UAS for an imagery company
C) Conducting public operations during a search mission

CFR §107.1 Applicability.
(3) Any operation that a remote pilot in command elects to conduct pursuant to an exemption issued under section 333 of Public Law 112-95, unless otherwise specified in the exemption.

7. According to 14 CFR part 48, when would a small unmanned aircraft owner not be permitted to register it? [Source: 14 CFR 48.25(b)]
A) If the owner does not have a valid United States driver`s license
B) All persons are eligible to register a small unmanned aircraft
C) If the owner is less than 13 years of age

CFR §48.25 Applicants.
(b) A small unmanned aircraft must be registered by its owner using the legal name of its owner, unless the owner is less than 13 years of age. If the owner is less than 13 years of age, then the small unmanned aircraft must be registered by a person who is at least 13 years of age.

8. Under what condition would a small unmanned aircraft not have to be registered before it is operated in the United States? [Source: 14 CFR 48.15]
A) When the aircraft has a takeoff weight that is more than 0.55 pounds, but less than 55 pounds, not including fuel and necessary attachments
B) When the aircraft weighs less than 0.55 pounds on takeoff, including everything that is on-board or attached to the aircraft
C) All small unmanned aircraft need to be registered regardless of the weight of the aircraft before, during, or after the flight

You need to register your aircraft if it weighs between 0.55 lbs. (250 grams) and up to 55 lbs. (25 kg) and you are not flying under the Special Rule for Model Aircraft. Register My sUAS

The lower limit is contained in CFR §48.15 Requirement to register.
No person may operate a small unmanned aircraft that is eligible for registration under 49 U.S.C. 44101-44103 unless one of the following criteria has been satisfied:
(a) The owner has registered and marked the aircraft in accordance with this part;
(b) The aircraft weighs 0.55 pounds or less on takeoff, including everything that is on board or otherwise attached to the aircraft;

The upper limit is defined by a combination of FARs.
CFR §107.3 Definitions.
Small unmanned aircraft means an unmanned aircraft weighing less than 55 pounds on takeoff, including everything that is on board or otherwise attached to the aircraft.

and

CFR §48.5 Compliance dates.
a) Small unmanned aircraft used exclusively as model aircraft.… (b) Small unmanned aircraft used as other than model aircraft. Small unmanned aircraft owners authorized to conduct operations other than model aircraft operations must register the small unmanned aircraft in accordance with part 47 of this chapter…

9. When using a small unmanned aircraft in a commercial operation, who is responsible for informing the participants about emergency procedures? [Source: AC 107-2, Small Unmanned Aircraft Systems (sUAS)(as amended)]
A) The FAA Inspector-in-Charge
B) The Remote Pilot in Command
C) The lead visual observer

CFR §107.49 Preflight familiarization, inspection, and actions for aircraft operation.
Prior to flight, the remote pilot in command must:
(b) Ensure that all persons directly participating in the small unmanned aircraft operation are informed about the operating conditions, emergency procedures, contingency procedures, roles and responsibilities, and potential hazards;

10. A person without a part 107 remote pilot certificate may operate a small UAS for commercial operations: [Source: AC-107-2, Small Unmanned Aircraft Systems (sUAS) (as amended)]
A) Only when visual observers participate in the operation
B) Under the direct supervision of a Remote PIC
C) Alone, if operating during daylight hours

CFR §107.12 Requirement for a remote pilot certificate with a small UAS rating.
(a) Except as provided in paragraph (c) of this section, no person may manipulate the flight controls of a small unmanned aircraft system unless:
(1) That person has a remote pilot certificate with a small UAS rating issued pursuant to subpart C of this part and satisfies the requirements of §107.65; or
(2) That person is under the direct supervision of a remote pilot in command and the remote pilot in command has the ability to immediately take direct control of the flight of the small unmanned aircraft.

11. A person whose sole task is watching the small UAS to report hazards to the rest of the crew is called: [Sources: 14 CFR part 107.3; AC 107-2, Small Unmanned Aircraft Systems (sUAS)(as amended)]
A) Remote PIC
B) Visual observer
C) Person manipulating the controls

§107.3 Definitions.
Visual observer means a person who is designated by the remote pilot in command to assist the remote pilot in command and the person manipulating the flight controls of the small UAS to see and avoid other air traffic or objects aloft or on the ground.

12. When adapting crew resource management (CRM) concepts to the operation of a small unmanned aircraft, CRM must be integrated into: [Source: FAA-H-8083-25, Pilot’s Handbook of Aeronautical Knowledge (PHAK), 17-2]
A) All phases of the operation
B) The communications only
C) The flight portion only

AC 107-2 A.2.5 A characteristic of CRM is creating an environment where open communication is encouraged and expected, and involves the entire crew to maximize team performance. Many of the same resources that are available to manned aircraft operations are available to UAS operations. For example, remote PICs can take advantage of traditional CRM. These crewmembers can provide information about traffic, airspace, weather, equipment, and aircraft loading and performance.

13. You have been hired as a Remote Pilot in Command by a local TV news station to film breaking news with a small unmanned aircraft. You expressed a safety concern and the station manager has instructed you to “hurry up and get it done.” What type of hazardous attitude does this attitude represent? [Source: FAA-H-8083-25, Pilot’s Handbook of Aeronautical Knowledge (PHAK), 17-4]
A) Impulsivity
B) Invulnerability
C) Machoism

Remote Pilot Study Guide Figure 10-2 and FAA-H-8083-25, Pilot’s Handbook of Aeronautical Knowledge, Figure 2-4 The five hazardous attitudes identified through past and contemporary study. Invulnerability “It won’t happen to me.” Many people falsely believe that accidents happen to others, but never to them. They know accidents can happen, and they know that anyone can be affected. However, they never really feel or believe that they will be personally involved. Pilots who think this way are more likely to take chances and increase risk.

14. Under what condition should the Remote Pilot in Command of a small unmanned aircraft establish a scheduled maintenance protocol? [Source: AC 107-2, Small Unmanned Aircraft Systems (sUAS)(as amended)]
A) Small unmanned aircraft systems do not require maintenance
B) When the manufacturer does not provide a maintenance schedule
C) When the FAA requires you to, following an accident

AC 107-2 7.2 Maintenance. sUAS maintenance includes scheduled and unscheduled overhaul, repair, inspection, modification, replacement, and system software upgrades of the sUAS and its components necessary for flight. Whenever possible, the operator should maintain the sUAS and its components in accordance with manufacturer’s instructions. The aircraft manufacturer may provide the maintenance program, or, if one is not provided, the applicant may choose to develop one.

15. Scheduled maintenance should be performed in accordance with the: [Source: AC-107-2, Small Unmanned Aircraft Systems (sUAS)(as amended)]
A) Stipulations in 14 CFR part 43
B) Manufacturer’s suggested procedures
C) Contractor requirements

AC 107-2 7.2 Maintenance. sUAS maintenance includes scheduled and unscheduled overhaul, repair, inspection, modification, replacement, and system software upgrades of the sUAS and its components necessary for flight. Whenever possible, the operator should maintain the sUAS and its components in accordance with manufacturer’s instructions. The aircraft manufacturer may provide the maintenance program, or, if one is not provided, the applicant may choose to develop one.

16. According to 14 CFR part 107, the responsibility to inspect the small unmanned aircraft system (small UAS) to ensure it is in a safe operating condition rests with the: [Source: 14 CFR 107.49(a)]
A) Visual observer
B) Remote Pilot in Command
C) Owner of the small UAS

AC 107-2 7.3 Preflight Inspection. Before each flight, the remote PIC must inspect the sUAS to ensure that it is in a condition for safe operation, such as inspecting for equipment damage or malfunction(s). The preflight inspection should be conducted in accordance with the sUAS manufacturer’s inspection procedures when available (usually found in the manufacturer’s owner or maintenance manual) and/or an inspection procedure developed by the sUAS owner or operator.

CFR 107.49 Preflight familiarization, inspection, and actions for aircraft operation.
Prior to flight, the remote pilot in command must: (c) Ensure that all control links between ground control station and the small unmanned aircraft are working properly;
(d) If the small unmanned aircraft is powered, ensure that there is enough available power for the small unmanned aircraft system to operate for the intended operational time; and
(e) Ensure that any object attached or carried by the small unmanned aircraft is secure and does not adversely affect the flight characteristics or controllability of the aircraft.

17. Before each flight, the Remote PIC must ensure that: [Source: AC-107-2, Small Unmanned Aircraft Systems (sUAS)(as amended)]
A) ATC has granted clearance
B) The site supervisor has approved the flight
C) Objects carried on the small UAS are secure

CFR 107.49 Preflight familiarization, inspection, and actions for aircraft operation.
(e) Ensure that any object attached or carried by the small unmanned aircraft is secure and does not adversely affect the flight characteristics or controllability of the aircraft.

18. When operating an unmanned aircraft, the Remote Pilot in Command should consider that the load factor on the wings or rotors may be increased anytime when: [Source: FAA-H-8083-25, Pilot’s Handbook of Aeronautical Knowledge, Chapter 4-12]
A) The gross weight is reduced
B) The center of gravity (CG) is shifted rearward to the aft CG limit
C) The aircraft is subjected to maneuvers other than straight and level flight.

The Remote Pilot Study Guide Chapter 4 Load Factors in Steep Turns has a detailed description of how the load factor is affected by turns.

FAA-H-8083-25, Pilot’s Handbook of Aeronautical Knowledge, Chapter 5 Critical load factors apply to all flight maneuvers except unaccelerated straight flight where a load factor of 1 G is always present.

19. A stall occurs when the smooth airflow over the unmanned airplane`s wing is disrupted, and the lift degenerates rapidly. This is caused when the wing: [Source: FAA-H-8083-3, Airplane Flying Handbook, 4-3]
A) Exceeds maximum allowable operating weight
B) Exceeds its critical angle of attack
C) Exceeds the maximum speed

FAA-H-8083-3, Airplane Flying Handbook, Chapter 4 A stall is an aerodynamic condition which occurs when smooth airflow over the airplane’s wings is disrupted, resulting in loss of lift. Specifically, a stall occurs when the AOA—the angle between the chord line of the wing and the relative wind—exceeds the wing’s critical AOA. It is possible to exceed the critical AOA at any airspeed, at any attitude, and at any power setting.

20. What could be a consequence of operating a small unmanned aircraft above its maximum allowable weight? [Source: Pilot’s Handbook of Aeronautical Knowledge (PHAK), 9-2]
A) Faster speed
B) Increased maneuverability
C) Shorter endurance

Remote Pilot Study Guide
Important performance deficiencies of an overloaded aircraft are:
• Higher takeoff speed
• Longer takeoff run
• Reduced rate and angle of climb
• Lower maximum altitude
• Shorter range
• Reduced cruising speed
• Reduced maneuverability
• Higher stalling speed
• Higher approach and landing speed
• Longer landing roll

For purposes of this question I think we can interpret “Shorter range” as “Shorter endurance”.

21. According to 14 CFR part 107, who is responsible for ensuring that all control links between the ground control station and the small unmanned aircraft are working properly? [Source: 14 CFR 107.49]
A) Manufacturer
B) Remote Pilot in Command
C) Owner or operator

CFR 107.49 Preflight familiarization, inspection, and actions for aircraft operation.
Prior to flight, the remote pilot in command must: (c) Ensure that all control links between ground control station and the small unmanned aircraft are working properly;

22. To ensure that the unmanned aircraft center of gravity (CG) limits are not exceeded, follow the aircraft loading instructions specified in the: [Source: FAA-H-8083-1, Weight & Balance Handbook, 4-4-5]
A) Pilot’s Operating Handbook or UAS Flight Manual
B) Aircraft Weight and Balance Handbook
C) Aeronautical Information Manual (AIM)

Remote Pilot Study Guide Adverse balance conditions (i.e., weight distribution) may affect flight characteristics in much the same manner as those mentioned for an excess weight condition. Limits for the location of the center of gravity (CG) may be established by the manufacturer.

From the course materials, As with any aircraft, compliance with weight and balance limits is critical to the safety of flight for a small UAS. An unmanned aircraft that is loaded out of balance may exhibit unexpected and unsafe flight characteristics.

Before any flight, verify that the unmanned aircraft is correctly loaded by determining the weight and balance condition.

Review any available manufacturer weight and balance data and follow all warnings, cautions, notes, and limitations.

If the manufacturer does not provide specific weight and balance data, apply general weight and balance principals to determine limits for a given flight. For example, add weight to the unmanned aircraft in a manner that does not adversely affect the aircraft’s center of gravity (CG) location—a point at which the unmanned aircraft would balance if it were suspended at that point.

From AC 107-2 1. Weight and Balance (W&B).
Before any flight, the remote PIC should verify the aircraft is correctly loaded by determining the W&B condition of the aircraft. An aircraft’s W&B restrictions established by the manufacturer or the builder should be closely followed. Compliance with the manufacturer’s W&B limits is critical to flight safety. The remote PIC must consider the consequences of an overweight aircraft if an emergency condition arises.

23. How would high density altitude affect the performance of a small unmanned aircraft? [Source: Pilot’s Handbook of Aeronautical Knowledge (PHAK), Chapter 10]
A) Increased performance
B) Decreased performance
C) No change in performance

FAA-H-8083-25, Pilot’s Handbook of Aeronautical Knowledge The density of the air, of course, has a pronounced effect on aircraft and engine performance. Regardless of the actual altitude at which the aircraft is operating, it will perform as though it were operating at an altitude equal to the existing density altitude.… High density altitude refers to thin air while low density altitude refers to dense air. The conditions that result in a high density altitude are high elevations, low atmospheric pressures, high temperatures, high humidity, or some combination of these factors.

24. While operating around buildings, the Remote Pilot in Command should be aware of the creation of wind gusts that: [Source: Pilot’s Handbook of Aeronautical Knowledge (PHAK), Chapter 11]
A) Increase performance of the aircraft
B) Change rapidly in direction and speed causing turbulence
C) Enhance stability and imagery

Remote Pilot Study Guide Another atmospheric hazard exists that can create problems for pilots. Obstructions on the ground affect the flow of wind and can be an unseen danger. Ground topography and large buildings can break up the flow of the wind and create wind gusts that change rapidly in direction and speed. These obstructions range from man-made structures, like hangars, to large natural obstructions, such as mountains, bluffs, or canyons.

25. According to 14 CFR part 107, what is required to operate a small unmanned aircraft within 30 minutes after official sunset? [Source: 14 CFR 107.29(b)]
A) Must be operated in a rural area
B) Use of lighted anti-collision lights
C) Use of a transponder

CFR §1
Night means the time between the end of evening civil twilight and the beginning of morning civil twilight, as published in the Air Almanac, converted to local time.

CFR §107.29 Daylight operation.
(a) No person may operate a small unmanned aircraft system during night.
(b) No person may operate a small unmanned aircraft system during periods of civil twilight unless the small unmanned aircraft has lighted anti-collision lighting visible for at least 3 statute miles. The remote pilot in command may reduce the intensity of the anti-collision lighting if he or she determines that, because of operating conditions, it would be in the interest of safety to do so.

Note that civil twilight is about a half hour after sunset and before sunrise so anti-collision lights are required if operating in that period.

26. According to 14 CFR part 107, how may a Remote Pilot in Command (Remote PIC) operate an unmanned aircraft in class C airspace? [Source: Aeronautical Information Manual (AIM), 3-2-6]
A) The Remote PIC must contact the Air Traffic Control (ATC) facility after launching the unmanned aircraft
B) The Remote PIC must monitor the Air Traffic Control (ATC) frequency from launch to recovery
C) The Remote PIC must have prior authorization from Air Traffic Control (ATC)

CFR §107.41 Operation in certain airspace.
No person may operate a small unmanned aircraft in Class B, Class C, or Class D airspace or within the lateral boundaries of the surface area of Class E airspace designated for an airport unless that person has prior authorization from Air Traffic Control (ATC).

27. In accordance with 14 CFR part 107, you may operate a small UAS from a moving vehicle when no property is carried for compensation or hire: [Sources: 14 CFR part 107.25; AC 107-2, Small Unmanned Aircraft Systems (sUAS)(as amended)]
A) Over suburban areas
B) Over a parade or other social events
C) Over a sparsely populated area

CFR §107.25 Operation from a moving vehicle or aircraft.
No person may operate a small unmanned aircraft system—
(a) From a moving aircraft; or
(b) From a moving land or water-borne vehicle unless the small unmanned aircraft is flown over a sparsely populated area and is not transporting another person’s property for compensation or hire.

28. In accordance with 14 CFR part 107, except when within a 400’ radius of a structure, at what maximum altitude can you operate a small UAS? [Source: 14 CFR part 107.51]
A) 600 feet AGL
B) 400 feet AGL
C) 500 feet AGL

CFR §107.51 Operating limitations for small unmanned aircraft.
(b) The altitude of the small unmanned aircraft cannot be higher than 400 feet above ground level, unless the small unmanned aircraft:

29. The FAA may approve your application for a waiver of provisions in part 107 only when it has been determined that the proposed operation: [Sources: 14 CFR Parts 101.41. 107.1, 107.200, and 107.205; AC 107-2, Small Unmanned Aircraft Systems (sUAS)(as amended)]
A) Will be conducted outside of the United States
B) Can be safely conducted under the terms of that certificate of waiver
C) Involves public aircraft or air carrier operations

§107.200 Waiver policy and requirements.
(a) The Administrator may issue a certificate of waiver authorizing a deviation from any regulation specified in §107.205 if the Administrator finds that a proposed small UAS operation can safely be conducted under the terms of that certificate of waiver.

30. When requesting a waiver, the required documents should be presented to the FAA at least how many calendar days prior to the planned operation? [Source: AC 107-2, Small Unmanned Aircraft Systems (sUAS)(as amended)]
A) 90 days
B) 10 days
C) 30 days

AC 107-2 Although not required by part 107, the FAA encourages applicants to submit their application at least 90 days prior to the start of the proposed operation. The FAA will strive to complete review and adjudication of waivers within 90 days; however, the time required for the FAA to make a determination regarding waiver requests will vary based on the complexity of the request.

31. To avoid a possible collision with a manned airplane, you climb your unmanned aircraft to yield the right of way. In doing so, your unmanned aircraft reached an altitude greater than 600 feet AGL. To whom must you report the deviation? [Source: 14 CFR 107.21(b)]
A) The Federal Aviation Administration, upon request
B) Air Traffic Control
C) The National Transportation Safety Board

CFR §107.21 In-flight emergency.
(b) Each remote pilot in command who deviates from a rule under paragraph (a) of this section must, upon request of the Administrator, send a written report of that deviation to the Administrator.

32. Damaged lithium batteries can cause: [Source: Safety Alert for Operators (SAFO) 10017, Risks in Transporting Lithium Batteries in Cargo by Aircraft]
A) A change in aircraft center of gravity
B) An inflight fire
C) Increased endurance

AC 107-2 A battery fire could cause an in-flight emergency by causing a LOC of the small UA. Lithium battery fires can be caused when a battery short circuits, is improperly charged, is heated to extreme temperatures, is damaged as a result of a crash, is mishandled, or is simply defective. The remote PIC should consider following the manufacturer’s recommendations, when available, to help ensure safe battery handling and usage.

33. While operating a small unmanned aircraft system (small UAS), you experience a flyaway and several people suffer injuries. Which of the following injuries requires reporting to the FAA? [Source: 14 CFR 107.9 and 107(III)(I)(2); AC-107-2, Small Unmanned Aircraft Systems (sUAS)(as amended)]
A) Minor bruises
B) Scrapes and cuts bandaged on site
C) An injury requiring an overnight hospital stay

§107.9 Accident reporting.
No later than 10 calendar days after an operation that meets the criteria of either paragraph (a) or (b) of this section, a remote pilot in command must report to the FAA, in a manner acceptable to the Administrator, any operation of the small unmanned aircraft involving at least:
(a) Serious injury to any person or any loss of consciousness; or
(b) Damage to any property, other than the small unmanned aircraft, unless one of the following conditions is satisfied:
  (1) The cost of repair (including materials and labor) does not exceed $500; or
  (2) The fair market value of the property does not exceed $500 in the event of total loss.

34. Within how many calendar days must a small UAS accident be reported to the FAA? [Sources: 14 CFR part 107.9; AC 107-2, Small Unmanned Aircraft Systems (sUAS)(as amended)]
A) 30 days
B) 10 days
C) 90 days

§107.9 Accident reporting.
No later than 10 calendar days after an operation that meets the criteria of either paragraph (a) or (b) of this section, a remote pilot in command must report to the FAA, in a manner acceptable to the Administrator, any operation of the small unmanned aircraft involving at least:
(a) Serious injury to any person or any loss of consciousness; or
(b) Damage to any property, other than the small unmanned aircraft, unless one of the following conditions is satisfied:
  (1) The cost of repair (including materials and labor) does not exceed $500; or
  (2) The fair market value of the property does not exceed $500 in the event of total loss.

35. The effective use of all available resources—human, hardware, and information — prior to and during flight – to ensure the successful outcome of the operation is called: [Source: AC-107-2, Small Unmanned Aircraft Sysems (sUAS)(as amended)]
A) Safety Management System
B) Crew Resource Management
C) Risk Management

AC 107-2 5.3 Aeronautical Decision-Making (ADM) and Crew Resource Management (CRM). manage these resources effectively. CRM is a component of ADM, where the pilot of sUAS makes effective use of all available resources: human resources, hardware, and information. Many remote pilots operating under part 107 may use a VO, oversee other persons manipulating the controls of the small UA, or any other person who the remote PIC may interact with to ensure safe operations. Therefore, a remote PIC must be able to function in a team environment and maximize team performance.

FAASTeam Course: Part 107 small Unmanned Aircraft Systems (sUAS)

October 26th, 2017

Regulations for small Unmanned Aircraft Systems went into effect in the summer of 2016 and the FAA has developed a UAS page. I read the FAR carefully and took the course and got all of the questions right the first time. A non-pilot might have more trouble, but if they really studied the Remote Pilot Study Guide and AC then they should have no trouble with the Knowledge Test. There are other FARs and FAA publications that are relevant but I think that they are covered enough in these documents that you could easily pass the test.

The rest of this post is the description of the course and the review section at the end of the course. It is a good overview of the things you need to know. Other posts in this series cover the exam I took at the end of the course and sample knowledge tests.

Description
The part 107 small Unmanned Aircraft Systems (sUAS) course describes the certification and operational requirements to operate sUAS in the National Airspace System (NAS) under Title 14 of the Code of Federal Regulations (14 CFR) part 107, small Unmanned Aircraft Systems. For part 61 pilot certificate holders with a current flight review, successful completion of this online course satisfies the training requirement before applying for a part 107 remote pilot certificate with an sUAS rating. All other interested individuals may complete this online course as a self-study resource. Individuals without a part 61 pilot certificate or current flight review are required to take the FAA Unmanned Aircraft General (UAG) Knowledge Test at an FAA-approved Knowledge Testing Center before applying for a part 107 certificate.

Review Introduction
The Federal Aviation Administration (FAA) has adopted specific rules to allow the operation of civil small unmanned aircraft systems (small UAS) in the National Airspace System (NAS) for purposes other than hobby and recreation. The rules are specified in Title 14 of the Code of Federal Regulations (14 CFR) part 107, Small Unmanned Aircraft Systems. 14 CFR part 107 addresses small UAS classification, certification, and operating rules.

Remote Pilot in Command Eligibility Requirements
To apply for a part 107 remote pilot certificate with a small UAS rating, you must be at least 16 years old; able to read, speak, write, and understand the English language (FAA may make exceptions for medical reasons); and in a physical and mental condition that would not interfere with the safe operation of a small UAS.

Training and Testing Requirements
A part 61 pilot certificate holder with a current flight review (per 14 CFR part 61.56) may complete this initial online course or the initial FAA Unmanned Aircraft General (UAG) Knowledge Test at a Knowledge Testing Center (KTC). Every 24 months, such individuals may then take the recurrent online course or the recurrent FAA UAG Knowledge Test at a KTC.

Any other applicant is required to take the initial FAA UAG Knowledge Test at a KTC, followed by the recurrent FAA UAG Knowledge Test at a KTC (every 24 months).

Application Process for a Remote Pilot Certificate
After satisfying the applicable initial training or testing requirements, apply for a part 107 remote pilot certificate with a small UAS rating through an online or paper process. Apply online through the Integrated Airman Certificate and/or Rating Application (IACRA) website whenever possible. Or submit a paper FAA Form 8710-13, Remote Pilot Certificate and/or Rating Application. You may be required to meet with an FAA-authorized individual, such as a Certificated Flight Instructor (CFI), Airman Certification Representative (ACR) for a pilot school, a person designated by a Flight Standards District Office (FSDO), or Designated Pilot Examiner (DPE).

Small Unmanned Aircraft System (small UAS) Characteristics

Small unmanned aircraft:
Weigh less than 55 pounds (25 kg), including everything that is onboard or otherwise attached to the aircraft.
Are operated without the possibility of direct human intervention from within or on the aircraft.
A small unmanned aircraft system includes the unmanned aircraft itself and its associated elements that are required for safe operation, such as communication links and components that control the aircraft.

Exclusions
14 CFR part 107 does not apply to model aircraft that meet the criteria in 14 CFR part 101.41, amateur rockets, moored balloons or unmanned free balloons, kites, operations conducted outside the United States, public aircraft operations, and air carrier operations.

Registration Requirements
Owners must register small UAS with the FAA prior to operating in the NAS if the aircraft is greater than 0.55 lbs and operated under part 107. If the owner is less than 13 years of age, then the small unmanned aircraft must be registered by a person who is at least 13 years of age.

Obtain a Foreign Aircraft Permit before conducting any operation that involves a civil aircraft that is registered in a foreign country or owned, controlled, or operated by someone who is not a U.S. citizen or permanent resident.

Marking Requirements
Before operation, mark the small UAS to identify that it is registered with the FAA. The registration marking must be a unique identifier number, legible and durable, and visible or accessible without tools.

Crew Resource Management
A small UAS operation may involve one individual or a team of crewmembers:

The Remote Pilot in Command (Remote PIC) holds a current remote pilot certificate with a small UAS rating and has the final authority and responsibility for the operation and safety of the small UAS
A person manipulating the controls operates the small UAS under direct supervision of the Remote PIC
A visual observer acts as a flight crewmember to help see and avoid air traffic or other objects in the sky, overhead, or on the ground
Many techniques from manned aircraft operations apply to the operation of unmanned aircraft. Examples include situational awareness, risk-based aeronautical decision making, and crew resource management.

Maintenance and Inspection
Follow all manufacturer recommendations for scheduled and unscheduled overhaul, repair, inspection, modification, replacement, and system software upgrades for the unmanned aircraft itself and all components necessary for flight.

Before beginning any small UAS flight operation, inspect the small UAS to ensure that it is in a condition for safe operation.

Loading and Performance
Prior to each flight, ensure that any object attached to or carried by the small unmanned aircraft is secure and does not adversely affect the flight characteristics or controllability of the aircraft.

Follow all manufacturer recommendations for evaluating performance to ensure safe and efficient operation. Check weather conditions prior to and during every small UAS flight and consider the effects of weather on aircraft performance.

Operating Rules
The Remote PIC must ensure that the small UAS operation complies with all operational requirements and limitations described in 14 CFR part 107. All crewmembers must comply with part 107 requirements by operating at appropriate times, in approved locations, and in a manner that protects the safety of the persons, property, and the NAS.

Certificates of Waiver
If the operation cannot be conducted within the regulatory structure of part 107, the Remote PIC is responsible for submitting an application for a Certificate of Waiver and proposing a safe alternative. Only certain provisions of part 107 are waivable. FAA will determine if the proposed operation can be safely conducted under the terms of that Certificate of Waiver.

Abnormal and Emergency Situations
Follow any manufacturer guidance for appropriate response procedures in abnormal or emergency situations. In case of an in-flight emergency, the Remote PIC is permitted to deviate from any rule of part 107 to the extent necessary to meet that emergency. FAA may request a written report explaining the deviation.

Accident Reporting
Report any small UAS accident to the FAA, within 10 days of the operation, if any of the following thresholds are met:
Serious injury to any person or any loss of consciousness
Damage to any property, other than the small unmanned aircraft, if the cost is greater than $500 to repair or replace the property (whichever is lower).

Aircraft Equipment Suffixes for Flight Following

October 14th, 2017

When asking for flight following you need to give your N-Number, type of aircraft, destination, and initial altitude. These codes are the old domestic IFR flight plans. They are intended for IFR navigation so they assume that the aircraft has at least one VOR receiver.

IFR flights now require FAA Flight Plan Form 7233−4 which has more detailed equipment codes. Codes for the new ICAO compliant flight plans can be found in the AIM TBL 5−1−4 and TBL 5−1−5. If you use an EFB, like ForeFlight or WingsX, it can guide you through filing with the right code.

Aircraft Classification

RVSM stands for Reduced Vertical Separation Minimums. RVSM airspace is where air traffic control separates aircraft by a minimum of 1,000 feet vertically between flight level (FL) 290 and FL 410 inclusive. Global Navigation Satellite System (GNSS) is a generic term for satellite-based navigation systems like GPS/WAAS, Russia’s GLONASS, and Europe’s Galileo systems.

My Cherokee has a Mode C transponder and one VOR so it is a /U
An Arrow I fly has a non-WAAS Garmin 530 so it is /G.
My Cessna 210 has a WAAS Garmin 430 and it is also /G.

Antennas

October 10th, 2017

The VOR/Localizer/Glide Slope antenna is usually V-shaped and points backwards on the vertical stabilizer of the plane. On some aircraft, they point forward. They look like old-fashioned wire clothes hanger wire. Newer versions can be blade shaped. sometimes they are boomerang shaped like the old car phone antennas of the 90s.) They usually have a splitter that allows two radios to use the same antenna.

COM Antennas are usually a white rod or blade. You can’t split the signal for a COM radio using a rod antenna so one antenna is required for each radio. Not sure about blade antennas.

A GPS antenna is usually somewhat teardrop or ovoid shaped. You can’t split the GPS signal, so one antenna per device is required. Some ADSB systems have an embedded WAAS chip and need their own GPS antenna.

GPS and COM antennas can be combined.

IFR capable airplanes also have a marker beacon antenna. These can be embedded in the fuselage or external.

A transponder is required in most airplanes and they have their own antenna. These are a small 2″ or so stick with a ball on the end. They are mounted on the bottom of the fuselage.

Airplanes in the US at least are required to have an ELT. Old style ELTs and some of the newer ones have a piece of steel with a spring-like coil in them. Newer versions are more aerodynamic.

ADF antennas come in a wide variety of shapes. Some of the newer ones look like GPS antennas.

DME antennas usually look like a shark’s fin.

Composite aircraft often have the antenna embedded in the frame so it is not visible.

This Cessna video describes all of the antennas on a modern C182. I don’t have most of them on my airplanes and the ones I do have look a lot different.

Here are some images of different antennas. These are all from aluminum bodied planes. Composite planes often have the antenna embedded in the skin, except for GPS and I don’t have any images of them.

In general, VOR, ELT, and GPS antennas are on the top of the plane. Marker beacon, DME, and transponder antennas are on the bottom.

Here’s a picture of the most common arrangement. VOR antenna on the tail pointing backward. Two COM antennas. If you look closely at the bump on the right, that’s a GPS antenna. The long wire is an ADF antenna.

All 1

Here’s another aircraft with a common arrangement. In this one you can see the ELT antenna just behind the cover. ELTs are always on the top.

All 2

Here’s a good picture of an ELT antenna.

ELT

Beechcraft often have a boomerang antenna for Com/ Nav/ Loc/ GS antenna.

Boomerang

Another type of ADF and a good view of VOR and one COMM.

ADF

I don’t have a good picture of GPS antennas. The bump on the right is the antenna.

GPS

VOR blade antenna and an antenna on the stabilizer that I can’t identify.

VOR Blade

I haven’t seen many VOR antennas like this one.

VOR Odd

Replaced a Cylinder.

September 24th, 2017

We have one cylinder that has always had a lower pressure reading when we checked it at annual. This year it was 35/80 so we pulled it. If you look at the cylinder, it has a ton of corrosion that caused the rings to no longer seal. I suspect the rest of the cylinders are the same. When we borescoped them last year, we were looking for valves that weren’t seating properly and I don’t remember looking for corrosion.

Mike Busch has an article at AVWeb that talks about proper operation of big bore Continentals (ours is a TSIO 520-H4). I think I’ve read all of his articles. I don’t know if you do these things, but I always do. (Except for avoiding shock cooling.)

Lean Until the Engine Stumbles if you give it more than Taxi Power
One the engines are running and it’s time to taxi out, perhaps the most common mistake I see pilots make is failing to lean the engine properly. The Continental fuel-injection system is set up with a very rich idle mixture in order to facilitate cold starts. Taxiing with the mixture full-rich is like driving a car with the choke on…the engine is literally awash in excess fuel. This often results in fouled plugs, contaminated exhaust valves, and fuel dilution of the oil film on the cylinder walls.

Taxi
Incidentally, big-bore Continentals are supposed to idle smoothly at 600 RPM

I don’t think ours will. I usually need taxi at 800 or more and use a little braking from time to time.

Runup – Leaned Out
My preference is to leave the mixture leaned for idle when I do my runup. Others prefer to go full-rich for runup, and then re-lean for idle if there’s a delay.

I also lean for runup and when I make my call to the tower, I keep my hand on the mixture until cleared for takeoff. The plugs don’t have much in the way of lead deposits—especially compared to the Cherokee.

Takeoff – Slow increase in power. I don’t take as long as he does, maybe 5 seconds.
For optimum engine longevity, we want to minimize the gradient of this thermal event by throttling up as slowly as possible. If the runway is relatively long, I’ll try to advance the throttles just fast enough to reach full takeoff power as the airplane achieves rotation speed.

Climb
Normal cruise-climb in a Continental-powered airplane is normally 75% power. In many aircraft (including most Cessna singles and twins), this occurs at top-of-the-green manifold pressure and top-of-the-green RPM.

I normally pull the prop to 2300 at the end of the runway to reduce noise and then MP to 1” below the top of the green at pattern altitude. Our MP gauge reads 1″ high. I generally climb at 130 MPH.

You should be leaned for climb, cruise, descent, landing, and taxi. The only time you should be full-rich is for start, takeoff, and go-around.

If I am just burning gas or doing manuevers, I pull the power (usually a lot) and lean while climbing. CHTs are usually in the 250 range and TIT below 1200.

Cruise
I recommend reducing to a more conservative cruise power setting between 55% and 65% power. 55% provides near-optimum fuel economy, while 65% provides a good compromise between fuel economy and airspeed.

If I am flying with with passengers who want to get somewhere fast, I set the cruise to top of the green and 20-21 gph. If I flying to pick someone up, (or on the rare occasions when I am going somewhere) I do the more conservative power settings.

If your engines don’t feel smooth at bottom-of-the-green RPM, experiment to find the lowest RPM at which they do feel smooth, and cruise at that.

There is a Continental Service Bulletin (CSB09-11A) that says to never run below 2300 RPM in cruise. When we checked the prop, 2300 RPM was actually 2453. 2200 RPM is 2352. So I suppose I could cruise at 2200 RPM.

Descent
The one I use is to allow one minute for each 1″ of MP that I need to reduce to get from cruise power to approach power. …I recommend not enrichening the mixtures for descent unless the engines start running rough.

I do this if I am in cruise, but since I am usually running at much lower power settings, I don’t have to lose many inches. The consensus on AVweb is that shock-cooling is a myth. I think even Mike Busch no longer holds with this recommendation. Mike Busch, … has found it unusual to have them [CHTs] drop at a rate of even 30 degrees per minute with aggressive power reductions when ATC gives a slam dunk approach.

Landing
Every POH I’ve ever read instructs you to advance the mixture to full-rich on final prior to landing. Don’t do it! Pouring cold fuel on a hot cylinder head simply can’t be a good thing for cylinder longevity. I recommend leaving the mixture leaned out for landing and taxi. I also recommend setting the props to top-of-the-green RPM, not shoving them full forward the way the POH instructs.

I don’t do this. I normally set the prop to full when I am in the pattern and it is already at the stops. I also go full rich on final.

Shutdown
A turbocharger should be given the opportunity to cool down (and spin down) at idle for 3 to 5 minutes before shutting down the engine and thereby cutting off the flow of oil to the turbocharger. In many cases, the landing roll and taxi-in take enough time that no additional cool-down is required.

If the TIT is below 1000, I just shut down, if not I wait a couple of minutes.

Interesting Winds Today

September 19th, 2017

Checking out the hurricanes on Earth Wind Map I happened to notice two lows in the Pacific.

Earth Wind Map

Communications with ATC

September 17th, 2017

A recent forum post asked about the correct way to contact ATC. Specifically, they had heard that pilots use their aircraft type and color. I think they were confusing communication with ATC and talking to other pilots at a non-towered airport.

The AIM discusses communications with ATC in Chapter 4.

4−2−3. Contact Procedures
a. Initial Contact.
1. The terms initial contact or initial callup means the first radio call you make to a given facility or the first call to a different controller or FSS specialist within a facility. Use the following format:

(a) Name of the facility being called;
(b) Your full aircraft identification as filed in the flight plan or as discussed in paragraph 4−2−4, Aircraft Call Signs (below);
(c) When operating on an airport surface, state your position.
(d) The type of message to follow or your request if it is short; and
(e) The word “Over” if required.

c. Subsequent Contacts and Responses to Callup from a Ground Facility. Use the same format as used for the initial contact except you should state your message or request with the callup in one transmission.

4−2−4. Aircraft Call Signs
2. ATC specialists may initiate abbreviated call signs of other aircraft by using the prefix and the last three digits/letters of the aircraft identification after communications are established.

…The pilot may use the abbreviated call sign in subsequent contacts with the ATC specialist.

When operating at a non-towered field, many pilots have begun to identify their aircraft by type and color. I’m not sure when this started, but if you listen to popular YouTube flyers like steveo1kinevo, you will notice that when they are flying into non-towered fields, they just give the aircraft type, e.g. Caravan or TBM. From a practical standpoint, it makes a lot of sense. You can’t see the numbers but you can see the type and color. And unless there are lots of white Cessnas in the pattern, it is easy to keep track of who is where. What you want to know in the pattern is where are the other planes and how fast are they going.

Type and color gives you the necessary information. For me at least, it is easier to keep track of type and color than it is to keep track of N numbers.


The content on this web site is provided for your information only and does not purport to provide or imply legal advice.
Should opinions, explanations, or discussions conflict with current FARs, other rules, regulations, or laws, then appropriate provisions of those rules, regulations, or laws prevail.
Navigation charts are provided for illustrative purposes only and are Not for Navigation.
TouringMachine.com is not responsible or liable for any errors, omissions, or incorrect information contained within this site.
Use at your own risk.
Copyright © 2002-2024 Touring Machine Company. All Rights Reserved.